You are on page 1of 217

CBSE Bharat Sahodaya Complex

Model
Question Papers 2020-21
Grade - 12

In collaboration with
ENGLISH CORE (CODE NO. 301)
CLASS – XII 2020-21
Time allowed: 3 hours MaximumMarks: 80
SECTION A
READING SKILLS
I. Read the following passage carefully: 10
1 It is an eraof automation. Nearly every year several inventions and
innovations swell up the markets making the consumers vow on knowing
about each one. One of the commonest factors behind those inventions is our
consistent quest for making things easier for ourselves to work upon.
2 One of the greatest advances in modern technology has been the invention
of computers. Now there is hardly any sphere of human life where computers
have not been pressed into service of man.
3 Computers are capable of doing extremely complicated work in all branches
of learning. These machines can be put to varied uses. Because they work
accurately and at high speeds, they save research workers’ years of hard work.
This whole process by which machines can be used to work for us has been
called ‘automation’.In the future, automation may enable human beings far
more leisure than they dotoday. The coming of automation is bound to have
important social consequences.
4 Some years ago an expert on automation, Sir Leon Bagrit, pointed out that it
was a mistake to believe that these machines could ‘think’. There is no
possibility that human beings will be ‘controlled by machines.’ Though
computers are capable of learning from their mistakes and improving on their

1
performance, they need detailed instructions from human beings to be able to
operate. Theycan neverlead independent lives by making decisions of their
own.
5 Sir Leon said that in the future, computers would be developed which would
be small enough tocarry in the pocket. Computers could be plugged into a
national network and be used like radios. It would also be possible to make
tiny translating machines. This will enable people who do not share a common
language, to talk to one another without any difficulty.
6 Computers are used in ordinary public hospitals. By providing a machine with
a patient's systems, a doctor is to diagnose the nature of his illness.
Bookkeepers and accountants, too, can be relieved of dull clerical work, for the
tedious task of compiling and checking lists of figures can be done entirely by
machines.
7 Computers are so much in every sphere of life that they are speedily
becoming our masters and we like dumbs are allowing them to easily play
upon our minds. Things have changed so fast after theintroduction of
computers that hardly we sit to think upon concepts, believing computers to
be there for that task too. Getting everything on a single click has made our
other fingers inactive.
8 The advantage of computers is as vivid as its aftermath. It is an inevitable fact
the computers have enhanced technology but jammed common minds.
On the basis of your reading of the above passage, answer any ten out of the
following questions by choosing the best answers from the given options:
(110 = 10)
(a) Inventions are promoted due to ______.
(i) our interest in technology
(ii) our interest in consumerism

2
(iii) our interest in making things easier
(iv) our interest in doing works
(b) One of the greatest advances in human technology is _________.
(i) the era of automation
(ii) the invention of computers
(iii) machines that can think
(iv) the need for instructions from human beings
(c) The process by which machines can work for human beings is called _____.
(i) controlling by machines
(ii) bookkeepers and accountants
(iii) computers
(iv) automation
(d) How does automation help human beings?
(i) Enjoy more leisure
(ii) Machines do things without giving instructions
(iii) They are controlled by machines
(iv) Learn from mistakes
(e) Though computers are capable of learning from their mistakes and
improving on performance, they will never be able to replace man. Why?
(i) Computers can take decisions on their own.
(ii) Computers learn from their mistakes.
(iii) Computers are able to improve their performance.
(iv) Computers need detailed instructions from human beings.
(f) Sir Leon Bragit had predicted that ____.
(i) computers could be carried in one's pocket
(ii) weather forecasts would be made by computers
(iii) people could diagnose their own diseases

3
(iv) computers can lead independent lives
(g) How, according to Sir Leon, will computers break the language barrier?
(i) By using computers as radios.
(ii) By creating tiny translating machines.
(iii) By providing a machine with a patient’s system.
(iv) By using computer for clerical works.
(h) Choose the option that was not a change after the introduction of
computers?
(i) Humans think and work.
(ii) We allow computers to easily play upon our minds.
(iii) We have completely turned to believe on computers.
(iv) Information made available on the finger tips.
(i) It is an inevitable fact that computers ____.
(i) can take decisions by themselves
(ii) make our concepts clear
(iii) have jammed common minds
(iv) allow us to analyse the questions
(j) Find words from the passage which mean the same as ‘recognize’. (para 3)
(i) record
(ii) diagnose
(iii) access
(iv) relieved
(k) Find words from the passage which mean the same as ‘outcomes’. (para 6)
(i) consequences
(ii) branches
(iii) leisure
(iv) process

4
II. Read the following passage carefully: 10
1 No treatment on the market today has been proved to slow human aging.
But one intervention, consumption of a low-calorie yet nutritionally balanced
diet, works incredibly well in a broad range of animals, increasing longevity and
prolonging good health. Those findings suggest that caloric restriction could
delay aging and increase longevity in humans, too. But what if someone could
create a pill that mimicked the physiological effects of eating less without
actually forcing people to eat less, a 'caloricrestriction mimetic'?
2 The best-studied candidate for a caloric-restriction mimetic, 2DG (2-deoxy-D-
glucose), works by interfering with the way cells process glucose. It has proved
toxic at some doses in animals and so cannot be used in humans. But it has
demonstrated that chemicals can replicate the effects of caloric restriction; the
trick is finding the right one.
3 Cells use the glucose from food to generate ATP (adenosine triphosphate),
the molecule that powers many activities in the body. By limiting food intake,
caloric restriction minimizes the amount of glucose entering cells and
decreases ATP generation. When 2DG is administered to animals that eat
normally, glucose reaches cells in abundance but the drug prevents most of it
from being processed and thus reduces ATP synthesis. Researchers have
proposed several explanations for why interruption of glucose processing and
ATP production might retard aging. One possibility relates to the ATP-making
machinery’s emission of free radicals, which are thought to contribute to aging
and to such age-related diseases as cancer by damaging cells. Reduced
operation of the machinery should limit their production and thereby constrain
the damage. Another hypothesis suggests that decreased processing of glucose
could indicate to cells that food is scarce (even if it isn’t) and induce them to

5
shift into an anti-aging mode that emphasizes preservation of the organism
over such ‘luxuries’ as growth and reproduction.
(Adapted from ‘The Serious Search for an Anti-Aging Pill’. Copyright © 2006
Scientific American, a division of Nature America, Inc. All rights reserved.)
On the basis of your reading of the above passage, answer any ten out of the
following questions by choosing the best answers from the given options:
(110 = 10)
(a) Consumption of a low-calorie and balanced diet worked well in a broad
range of animals’ _______.
(i) fleshy body growth
(ii) long life and good health
(iii) nutritionally balanced diet
(iv) food habits
(b) What can suspend aging and increase longevity in humans?
(i) caloric restriction
(ii) unbalanced diet
(iii) high nutritious food
(iv) physiological effects
(c) Name the phenomena of mimicking the physiological effects of eating
less without actually forcing people to eat less.
(i) Balanced diet
(ii) Longevity in humans
(iii) pill
(iv) caloric-restriction mimetic
(d) Which is considered as the best-studied candidate for a caloric-
restriction mimetic?
(i) Deoxy -D- Glucose

6
(ii) ATP
(iii) 2DG
(iv) Glucose
(e) Why 2DG cannot be used in humans?
(i) It was created only for animals
(ii) It has proved toxic in animals
(iii) It has proved toxic in humans
(iv) It was created for non-humans
(f) ATP is generated from glucose by ____.
(i) Food
(ii) Adenosine
(iii) Cells
(iv) molecule
(g) _____ is reduced when 2DG is administered to animals that eat
normally.
(i) Glucose
(ii) ATP synthesis
(iii) Drug
(iv) Cell
(h) The second hypothesis proposes that lessened processing of glucose
could direct cells that food is scarce and give indication to change into
_______ to preserve the organism fromfurther growth.
(i) conservation
(ii) damaging cells
(iii) an anti-aging mode
(iv) ATP production

7
(i) Find the word from the passage which means the same as ‘depletion’.
(para 1)
(i) Treatment
(ii) Consumption
(iii) Longevity
(iv) Restriction
(j) Find the word from the passage which means the same as ‘duplicate’.
(para 2)
(i) mimetic
(ii) process
(iii) demonstrate
(iv) replicate
(k) Find the word from the passage which means the same as ‘interference’.
(para 3)
(i) restriction
(ii)abundance
(iii) interruption
(iv) indicate

SECTION B
WRITING SKILLS
III. The International Book Fair is being held at Pragruthy Maidan between 7
June and 14 June, 2021. As the librarian, write a notice informing the students
about it. (Word limit: 50 words)
3
OR

8
You are Kashyap/ Kasha, Secretary, Seasons Enterprises Ltd., Trivandrum – 24.
Your MD has asked you to draft an advertisement for a local daily under the
classified columns for the vacant posts of one Receptionist and two HR
Assistants. Draft the advertisement in not more than 50 words.

IV. On the occasion of Van Mahotsav function in ABC International School, a


tree plantation drive has been organised. As the Secretary of Eco Club, draft an
invitation to a noted environmentalist to grace the occasion. Word limit: 50
words. 3
OR
On behalf of your parents, draft an informal invitation card to be sent to your
friends and relatives on the occasion of your sibling’s tenth birthday.

V. You are Anila/Anand, staying at Arathy Apartments, Bandhur Street,


Mumbai. Write a letter to the Editor, The Deccan Chronicle registering your
protest against government/local body’s apathy towards cleanliness in the city
during the days of reported outbreak of plague in the city and migration of
people to the neighbouring centres/State. Look at the picture below and write
your letter. Word limit: 120-150 words.
5

9
OR
You are Ajay of 12, Jayanthi House, Himachal Pradesh. You recently read about
the post of a chemist being advertised by Alphine Research Ltd, Delhi. You wish
to apply for it. Apply with full details to the Managers of the production unit.
Word limit: 120-150 words.

VI. You are Mehnaz/ Muhammad Mehboob. You recently visited Kerala and
felt that there is vast potential for tourism in India that remains untapped.
Write the article in 120-150 words on ‘The Importance of Tourism’.
5
OR
You attended a meeting of Manakkal Residents’ Welfare Association, wherein
the police personnel explained various ways adopted by the burglars to make
an entry into the houses and suggested the precautions to be taken to prevent
such incidents. Write a report in 120-150 words to be published in The Hindu
Daily. You are Sachin/ Sashe, a reporter.

SECTION C
LITERATURE

10
VII. Read the extracts given below and answer the questions that follow. Any
two extracts need to be answered: (8x1=8)
(a) How it must have broken his heart to leave it all, poor man; to hear his
sister moving about in the room above, packing their trunks! For they must
leave the country next day.
(i) Who are ‘they’ here?
(a) Franz and his sister
(b) M. Hamel and his sister.
(c) Franz and M. Hamel
(d) None of the above
(ii) Why is M. Hamel’s heartbroken?
(a) Because no one is interested in his class.
(b) Because he has to leave the country the next day.
(c) Because he had no money.
(d) Because his parents were no more.
(iii) Why do they have to leave the country?
(a) Prussians had announced that German would be taught instead of
French.
(b) Nobody in the districts of Alsace and Lorraine learn German.
(c) Prussians had announced that French would be taught instead of
German.
(d) Nobody in the districts of Alsace and Lorraine learn French.
(iv) Who is packing the trunks?
(a) Hamel’s sister
(b) Franz’s sister
(c) Hamel’s wife
(d) Franz’s cousin

11
(b) “The world had, of course, never been very kind to him, so it gave him
unwonted joy to think ill of it in this way. It became a cherished pastime of his,
during many dreary ploddings, to think of people he knew who had let
themselves be caught in the dangerous snare, and of others who were still
circling around the bait.”
(i) Whom does ‘him’ refer to in the extract?
(a) The peddler.
(b) The crofter
(c) The ironmaster
(d) EdlaWillmansson
(ii) Why did he think ill of the world?
(a) Because he robbed people.
(b) Because the world showed kindness to him.
(c) Because the world had never been kind to him.
(d) Because he trapped people.
(iii) What is the ‘dangerous snare’?
(a) The rattrap.
(b) Deep pit
(c) Maze
(d) None of the above
(iv) What is the ‘bait’?
(a)scarecrow
(b) prohibiting things
(c) uninteresting things of the world
(d) worldly things like riches, joys, shelter, food etc.

12
(c) "So the wizard said, 'Oh, very well. Go to the end of the lane and turn
around three times and look down the magic well and there you will find three
pennies. Hurry up!”
(i) To whom is above extract addressed to?
(a) Jack
(b) Jo
(c) Roger Skunk
(d) Bobby
(ii) Why three pennies were required?
(a) There was a shortage of three pennies to pay the wizard.
(b) He missed three pennies on his way.
(c) He wanted it for doing magic.
(d) The wizard asked for three pennies.
(iii) What magic wizard had done?
(a) Changed the smell of Roger to that of roses.
(b) Changed Roger to roses.
(c) Changed the smell of Roger’s mother to that of roses.
(d) Changed Roger’s mother to roses.
(iv) Which word in the extract is synonym of street?
(a) wizard
(b) Lane
(c) turn
(d) pennies

VIII. Read the extracts given below and answer the questions that follow.
Any one extract needs to be answered:
(4x1=4)

13
(i) “Far far from gusty waves these children’s faces.
Like rootless weeds, the hair torn around their pallor.
The tall girl with her weighed-down head.
The paper-seeming boy, with rat’s eyes.”
(i) What are the children compared to?
(a) gusty waves
(b) rootless weeds
(c) pallor
(d)weighed-down head
(ii) Why do you think the tall girl is sitting with a weighed down head?
(a) She is depressed due to abject poverty or family tussles.
(b)She is having physical illness.
(c) She is not having any friends.
(d) She feels the class boring.
(iii) Choose two phrases which tell us that the children are under-
nourished.
(a)‘far from gusty waves’ and ‘the hair torn around their pallor’
(b) ‘far from gusty waves’ and ‘like rootless weeds’
(c) ‘the hair torn around their pallor’ and ‘the paper-seeming boy
with rat’s eyes’
(d) ‘like rootless weeds’ and ‘the paper-seeming boy with rat’s
eyes’.
(iv) What does the phrase, ‘paper-seeming boy’ convey?
(a) The boy likes to draw.
(b) The boy plays with paper.
(c) The boy is as thin as paper.
(d) The boy is an expert in origami.

14
(ii) When Aunt is dead, her terrified hands will lie
Still ringed with ordeals she was mastered by.
The tigers in the panel that she made
Will go on prancing, proud and unafraid.
(i) Why are Aunt Jennifer’s hands ‘terrified’?
(a) Because she couldn’t complete theembroidery.
(b)Because of the unhappy and harsh experience of her married
life.
(c) Because the tigers she made was not perfect.
(d) Because she didn’t finish her work.
(ii) What is Aunt Jennifer’s death symbolic of?
(a) Natural death
(b) Suicide
(c)Life in heaven after death.
(d) Complete submission to the suppression of her husband.
(iii) How will the tigers behave after her death?
(a) Will go on prancing proud and unafraid.
(b) Will die with her.
(c) Will be afraid of men.
(d) Will be tamed by men.
(iv) What is the literary device used in the last line?
(a) Assonance
(b) Alliteration
(c) Enjambment
(d) Anaphora

IX. Answer any eight of the following questions. (8x1=8)

15
(a) Why did the narrator talk to his psychiatrist friend and what did he say
about the third level? (The Third Level)
(b) How has Mukesh’s father sustained his life all through? (Lost Spring)
(c) Which two incidents in Douglas’ early life made him scared of water? (Deep
Water)
(d) Why did the General not order immediate arrest of Dr Sadao who had
sheltered the white man? (The Enemy)
(e) Why did Gandhi object to C. F. Andrew’s stay in Champaran? (Indigo)
(f) Having looked her mother, why did Kamala Das look at the young children?
(My Mother at Sixty Six)
(g)What does Neruda want the war mongers to do? (Keeping Quiet)
(h) How does Keats define a thing of beauty? (A Thing of Beauty)
(i) Why does Derry’s mother not want him to go back to Mr. Lamb? (On The
Face Of It)
(j) Who is Carter? Why was he summoned? (Evans Tries An O-Level)

X. Answer any five of the following questions in 30 – 40 words each: (5x2=10)


(a) Why does Aunt create the tigers opposite to her character on a panel?
(b) How will keeping quiet protect our environment?
(c) Why does Kamala Das smile in the poem?
(d) Why did Gandhi think that taking the Champaran case to the court was
useless?
(e) Why did Douglas want to overcome his fear of water?
(f) Why does the author say, “ Mukesh’s dream of being a motor mechanic
looms like a mirage amidst the dust of streets that fill his town
Firozabad”?

16
XI. Answer any two of the following questions in 30 – 40 words each: (2x2=4)
(a) Why did Charley run away from the third level?
(b) Why did Mr. Lamb help Derry?
(c) Why was Evans reluctant to remove his hat?

XII. Answer any one of the following questions in 120-150 words: (1x5=5)
(a) The bangle makers of Firozabad make beautiful bangles and make
everyone happy but they live and die in squalor. Elaborate.
(b) In the poem, ‘Keeping Quiet’how is the poet’s appeal for keeping quiet
different from absolute sluggishness?

XIII. Answer any one of the following questions in 120-150 words: (1x5=5)
(a)How did Mr. Lamb’s meeting with Derry become a turning point in Derry’s
life?
(b)Dr Sadao was a patriotic Japanese as well as a dedicated surgeon. How
could he honour both the values?

17
ENGLISH CORE (CODE NO. 301)
CLASS – XII 2020-21
Time allowed: 3 hours Maximum Marks: 80
MARKING SCHEME

SUGGESTED VALUE POINTS


SECTION A: READING SKILLS
1 COMPREHENSION PASSAGE 10 marks
NOTE: No mark(s) should be deducted for mistakes in usage and
grammar, spelling, or word limit. Full marks may be awarded if a
student has been able to identify the core ideas. If a student literally
lifts a portion of the given passage as an answer to a question, no
mark(s) to be deducted for this as long as it is relevant.

I. MCQ (Any Ten)


(a) Inventions are promoted due to ______.
Ans. (iii) our interest in making things easier (1
mark)
(b) One of the greatest advances in human technology is
_________.
Ans. (ii) the invention of computers (1 mark)
(c) The process by which machines can work for human
beings is called _____.
Ans. (iv) automation (1 mark)
(d) How does automation help human beings?
Ans. (i) Enjoy more leisure (1 mark)
(e) Though computers are capable of learning from their
mistakes and improving on performance, they will never
be able to replace man. Why?
Ans. (iv) Computers need detailed instructions from
human beings. (1 mark)
(f) Sir Leon Bragit had predicted that ____.
Ans. (i) computers could be carried in one's pocket (1
mark)
(g) How, according to Sir Leon, will computers break the
language barrier?
Ans. (ii) By creating tiny translating machines. (1 mark)
(h) Choose the option that was not a change after the
introduction of computers?
Ans. (i) Humans think and work. (1 mark)
(i) It is an inevitable fact that computers ____.
Ans. (iii) have jammed common minds (1 mark)
(j) Find words from the passage which mean the same
as ‘recognize’. (para 3)
Ans. (ii) diagnose (1 mark)
(k) Find words from the passage which mean the same
as ‘outcomes’. (para 6)
Ans. (i) consequences (1 mark)
SECTION A: READING SKILLS
II COMPREHENSION PASSAGE 10 marks
MCQ (Any Ten)
(a) Consumption of a low-calorie and balanced diet
worked well in a broad range of animals’
_______.
Ans. (ii) long life and good health (1 mark)
(b) What can suspend aging and increase longevity in
humans?
Ans. (i) caloric restriction (1 mark)
(c) Name the phenomena of mimicking the
physiological effects of eating less without
actually forcing people to eat less.
Ans. (iv) caloric-restriction mimetic (1 mark)
(d) Which is considered as the best-studied candidate
for a caloric-restriction mimetic?
Ans. (iii) 2DG (1 mark)
(e) Why 2DG cannot be used in humans?
Ans. (ii) It has proved toxic in animals (1 mark)
(f) ATP is generated from glucose by ____.
Ans. (iii) Cells (1 mark)
(g) _____ is reduced when 2DG is administered to
animals that eat normally.
Ans. (ii) ATP synthesis (1 mark)
(h) The second hypothesis proposes that lessened
processing of glucose could direct cells that food
is scarce and give indication to change into
_______ to preserve the organism from further
growth.
Ans. (iii) an anti-aging mode (1 mark)
(i) Find the word from the passage which means the
same as ‘depletion’. (para 1)
Ans. (i) Consumption (1 mark)
(j) Find the word from the passage which means the
same as ‘duplicate’. (para 2)
Ans. (iv) Replicate (1 mark)
(k) Find the word from the passage which means the
same as ‘interference’. (para 3)
Ans. (iii) interruption (1 mark)
SECTION B (WRITING SKILLS) 16 MARKS
NOTE: The objective of the section on Writing Skills is to test a
candidate’s writing ability. Hence, expression assumes as much
importance as the content of the answer.

III. NOTICE: 3 marks


The format should include: ISSUING AUTHORITY / NAME
OF THE INSTITUTION, the word ‘NOTICE’, HEADING,
DATE, and WRITER’S NAME WITH DESIGNATION
The candidate should not be penalized if he / she has
used capital letters for writing a notice within or without
a box. 1 mark
Content 1 mark
Expression 1 mark
Qn: The International Book Fair is being held at
Pragruthy Maidan between 7 June and 14 June, 2021. As
the librarian, write a notice informing the students
about it. (Word limit: 50 words)
HEADING: International Book Fair/ Any other suitable
title
Suggested value points:
 objective  what  when  where  duration  request
children to join  any other relevant details (due credit
should be given to economy of words)
OR
III. ADVERTISEMENT 3 marks
Format 1 mark
Content 1 mark
Expression 1 mark
Qn: You are Kashyap/ Kasha, Secretary, Seasons
Enterprises Ltd., Trivandrum – 24. Your MD has asked
you to draft an advertisement for a local daily under the
classified columns for the vacant posts of one
Receptionist and two HR Assistants. Draft the
advertisement in not more than 50 words.
SITUATION VACANT
Suggested value points:
 requirement  nature of job  qualifications - academic
and professional  experience  proficiency/skills 
attractive salary  last date of application (any other
relevant point)
IV. INVITATION 3 Marks
Note: - No marks are to be awarded if only the format is
given. Credit should be given for the candidate's
creativity in presentation of ideas.
Format: 1 mark
Content (coherence and relevance of ideas and style): 1
mark
Expression (grammatical accuracy, appropriate words
and spelling): 1 mark
Qn. On the occasion of Van Mahotsav function in ABC
International School, a tree plantation drive has been
organised. As the Secretary of Eco Club, draft an
invitation to a noted environmentalist to grace the
occasion. Word limit: 50 words.
3
Formal Invitation Format:
(i) Host
(ii) Words of Invitation
(i) Occasion
(ii) Day, date, time, venue
(iii) Chief guest
(iv) RSVP
(v) Sender’s designation
(vi) Address/ Ph. No.

Sub: - Invitation for a tree plantation drive / any other


relevant subject
Suggested Value Points:  tree plantation drive  date,
time, venue  invite the environmentalist (any other
relevant point).
OR

Qn. On behalf of your parents, draft an informal


invitation card to be sent to your friends and relatives
on the occasion of your sibling’s tenth birthday.

Informal Invitation Format:


(i) Name of the host
(ii) Words of Invitation
(iii) Occasion
(i) Day, date, time, venue
(ii) RSVP
(iii) Sender’s name
(iv) Contact details (Ph. No.)

Sub: - Tenth Birthday / any other relevant subject


Suggested Value Points:  date, time, venue  invitation
(any other relevant point).
V. LETTER WRITING (5 marks)
LETTER TO THE EDITOR
Note: - No marks are to be awarded if only the format is
given. Credit should be given for the candidate's
creativity in presentation of ideas. Use of both the
traditional and the new format is permitted. Mixing of
the formats is not permitted.
Format: 1 mark
1. sender's address
2. date
3. receiver's address
4. subject /heading
5. salutation
6. closing
Content (coherence and relevance of ideas and style): 2
marks
Expression (grammatical accuracy, appropriate words
and spelling): 2 marks
Qn. You are Anila/Anand, staying at Arathy Apartments,
Bandhur Street, Mumbai. Write a letter to the Editor,
The Deccan Chronicle registering your protest against
government/local body’s apathy towards cleanliness in
the city during the days of reported outbreak of plague
in the city and migration of people to the neighbouring
centres/State. Look at the picture below and write your
letter. Word limit: 120-150 words.
Sub: - Importance of Cleanliness / Significance of
Cleanliness / Cleanliness drive and its appreciation / any
other relevant subject
Suggested value points: Draw attention protest
against government/local body’s apathy towards
cleanliness in the city importance of cleanliness 
present condition  what is required to be done 
contribution of people  its wholesome effect on nation
(any other relevant point)

OR
JOB APPLICATION:
Qn: You are Ajay of 12, Jayanthi House, Himachal
Pradesh. You recently read about the post of a chemist
being advertised by Alphine Research Ltd, Delhi. You
wish to apply for it. Apply with full details to the
Managers of the production unit. Word limit: 120-150
words.
Sub: - Job Application/ Application for the position of
Chemist / any other relevant subject
Suggested Value Points:  From where did you know
about the vacancy  what you want to do  your present
qualifications (all the details)  experiences  company’s
benefits of choosing you for the job (any other relevant
point)
VI. ARTICLE 5 marks
Format (Title and Writer’s Name) 1 mark
Content (coherence and relevance of ideas and style) 2
marks
Expression grammatical accuracy, appropriate words
and spelling [2 marks]
Qn. You are Mehnaz/ Muhammad Mehboob. You
recently visited Kerala and felt that there is vast
potential for tourism in India that remains untapped.
Write the article in 120-150 words on ‘The Importance
of Tourism’.
Heading: The Importance of Tourism/ any other suitable
title
Suggested Value Points:  Increases government
revenues.  Can be developed locally.  Improving the
quality of life of locals.  Provides recreational facilities
to locals too.  Promotes globalization.  Environment
protection and conservation.  Creates a favourable
image.  Enhancing cultural heritage.  Helps in
diversification of economy.  (any other relevant point).
OR
REPORT: 5 marks
Format –Title & By line (writer’s name) 1 mark
Expression grammatical accuracy, appropriate words
and spelling [2 marks] Content (coherence and
relevance of ideas and style) 2 marks
Qn. You attended a meeting of Manakkal Residents’
Welfare Association, wherein the police personnel
explained various ways adopted by the burglars to make
an entry into the houses and suggested the precautions
to be taken to prevent such incidents. Write a report in
120-150 words to be published in The Hindu Daily. You
are Sachin/ Sashe, a reporter.
Suggested Value Points:  what – meeting  purpose 
where  when  what - observation – ways adopted by
the burglars to make an entry into the houses 
precautions  any other relevant point
Note: Past Tense should be used.
SECTION C: LITERATURE (TEXT BOOKS) 44 Marks
NOTE: The objective of the section on Literature is to test a candidate’s
ability to understand and interpret the prescribed text through short
and long answer type questions. Hence, both content and expression in
answers to the given questions deserve equal importance while
awarding marks.

VII. Extract based questions. Any two. (8x1=8)


(a) How it must have broken his heart to leave it all,
poor man; to hear his sister moving about in the room
above, packing their trunks! For they must leave the
country next day.
(i) Who are ‘they’ here? 1
(b) Ans: M. Hamel and his sister.
(ii) Why is M. Hamel’s heartbroken? 1
(b) Ans: Because he has to leave the country the
next day.
(iii) Why do they have to leave the country? 1
(a) Ans: Prussians had announced that German
would be taught instead of French
(iv) Who is packing the trunks? 1
(a) Ans: Hamel’s sister

(b) “The world had, of course, never been very kind to


him, so it gave him unwonted joy to think ill of it in this
way. It became a cherished pastime of his, during many
dreary ploddings, to think of people he knew who had
let themselves be caught in the dangerous snare, and of
others who were still circling around the bait.”
(i) Whom does ‘him’ refer to in the extract? 1
(a) Ans: The peddler.
(ii) Why did he think ill of the world? 1
(c) Ans: Because the world had never been kind to
him.
(iii) What is the ‘dangerous snare’? 1
(a) Ans: The rattrap.
(iv) What is the ‘bait’? 1
(d) Ans: worldly things like riches, joys, shelter,
food etc.

(c) "So the wizard said, 'Oh, very well. Go to the end of
the lane and turn around three times and look down the
magic well and there you will find three pennies Hurry
up!”
(i) To whom is above extract addressed to? 1
(c) Ans: Roger Skunk
(ii) Why three pennies were required? 1
(a) Ans: There was a shortage of three pennies to pay
the wizard.
(iii) What magic wizard had done? 1
(a) Ans: Changed smell of Roger to that of roses.
(iv) Which word in the extract is synonym of street?
Ans: (b) Lane1
Extract Based Question. Any one. (4x1=4) (4x1=4)
(i) “Far far from gusty waves these children’s faces.
Like rootless weeds, the hair torn around their
pallor.
The tall girl with her weighed-down head.
The paper-seeming boy, with rat’s eyes.”
(i) What are the children compared to? 1
Ans.(b) rootless weeds
(ii) Why do you think the tall girl is sitting with a
weighed down head? 1

Ans. (a) She is depressed due to abject


poverty or family tussles.
(iii) Give two phrases which tell us that the
children are under-nourished. 1
Ans: (d) ‘like rootless weeds, and ‘the paper-
seeming boy with rat’s eyes’.
(iv) What does the phrase, ‘paper-seeming
boy’ convey? 1

Ans: (c) The boy is as thin as paper.


(iii) When Aunt is dead, her terrified hands will lie
Still ringed with ordeals she was mastered by.
The tigers in the panel that she made
Will go on prancing, proud and unafraid.
(i) Why are Aunt Jennifer’s hands ‘terrified’?
1
(b) Ans: Because of the unhappy and harsh
experience of her married life.
(ii) What is Aunt Jennifer’s death symbolic of?
1
(d) Ans: Complete submission to the
suppression of her husband.
(iii) How will the tigers behave after her death?
1
(a) Ans: will go on prancing proud and
unafraid.
(iv) What is the literary device used in the last
line? 1
(b) Ans: Alliteration.
IX. Short Answer Questions. Any eight. (8x1=8) (8x1=8)
(a) Why did the narrator talk to his psychiatrist friend
and what did he say about the third level? (The Third
Level)
Ans: -Because no one believed him- he said it was a
waking-dream wish fulfilment.
(b) How has Mukesh’s father sustained his life all
through? (Lost Spring)
Ans: - First as a tailor, then as a bangle maker – failed to
renovate his house and send his two sons to school –
only taught them bangle-making.
(c) Which two incidents in Douglas’ early life made him
scared of water? (Deep Water)
Ans: - Incident at California beach and the misadventure
at the Y.M.C.A.
(d) Why did the General not order immediate arrest of
Dr Sadao who had sheltered the white man? (The
Enemy)
Ans: -Because he was in need of Sadao’s help as he was
ill – if Sadao was arrested, there would be no one to
treat him.
(e) Why did Gandhi object to C. F. Andrew’s stay in
Champaran? (Indigo)
Ans: - Because he wanted the lawyers to be self-reliant
to fight for a just cause – they should not expect any
help from a Britisher.
(f) Having looked her mother, why did Kamala Das look
at the young children? (My Mother at Sixty-Six)
Ans: -To put away the thoughts of her ageing mother.
(g) What does Neruda want the war mongers to do?
(Keeping Quiet)
Ans: - Stop fighting and put-on clean clothes and live
with others with understanding.
(h) How does Keats define a thing of beauty? (A Thing of
Beauty)
Ans: -It is eternal- does not fade away- has soothing
effect on humans.
(i) Why does Derry’s mother not want him to go back to
Mr. Lamb? (On the Face of It)
Ans: - She claims to have heard many things about Mr.
Lamb – she has been told by people that he is not a
good person.
(j) Who is Carter? Why was he summoned? (Evans Tries
An O-Level)
Ans: -Detective Superintendent- To take the injured
parson with him as only he knew what was happening
and could help to catch Evans.
X. Answer any five in 30 – 40 words each: (5x2=10)
(a) Why does Aunt create the tigers opposite to her
character on a panel?
Ans: -To express her inner longing for
independence and free life.
(b) How will keeping quiet protect our environment?
Ans: - People will introspect not to harm nature
and understand the necessity of harmony with
nature.
(c) Why does Kamala Das smile in the poem?
Ans: - Because she does not want to show her
sadness to her mother- to hide her fear.
(d) Why did Gandhi think that taking the Champaran
case to the court was useless?
Ans: - Because the peasants were fear stricken – it
involved lot of expenses – wanted the farmers to
be free from fear.
(e) Why did Douglas want to overcome his fear of
water?
Ans: - Because it ruined all his fishing, boating and
canoeing trips – to enjoy all the water body
activities.
(f) Why does the author say, “ Mukesh’s dream of
being a motor mechanic looms like a mirage
amidst the dust of streets that fill his town
Firozabad”?
Ans: - Because people of Firozabad are poverty-stricken
– they never think differently.
XI. Answer any two in 30 – 40 words each: (2x2=4)
(a) Why did Charley run away from the third level?
Ans: - Because the clerk at the ticket counter charged
him that the money he was carrying was fake- he might
be put behind the bars for that.
(b) Why did Mr. Lamb help Derry?
Ans: - As a handicapped person, he knew the difficulties
faced by people like him- Derry was very pessimistic
which would spoil his life- wanted Derry to look at
things from different perspective.
(c) Why was Evans reluctant to remove his hat?
Ans:- Because he had kept his cut hair in it – used the
hair to look like McLerry – reluctant as it was his lucky
charm.
XII. Answer any one of the following questions in 120-150 (1x5=5)
words:
(a) The bangle makers of Firozabad make beautiful
bangles and make everyone happy but they live
and die in squalor. Elaborate.
Answer: Firozabad is the hub of India’s glass-
blowing industry - where families have spent
generations making bangles to adorn married
women. - in spite of the back breaking hard work
that they put in, they cannot have two square
meals a day. - They work in deplorable conditions
- many lose their eyesight early- live in unhygienic
conditions- lack of basic amenities – the workers
cannot organise themselves into a cooperative -
devoid of all enthusiasm and do not dare to
dream of anything better -The fear of the police -
lack of leadership among themselves - poverty,
indifference and greed- their own life is steeped
in poverty and squalor.
(b) In the poem, ‘Keeping Quiet’ how is the poet’s
appeal for keeping quiet different from absolute
sluggishness?
Answer:The poet- emphasizes the importance of
self-introspection in a man’s life- seek a moment
to understand ourselves - analyse our actions as
we experience the sadness of death - a moment
of silence or stillness required - as a moment of
inactivity or absolute sluggishness - He seeks
silence where people are not obsessed with work
and give a personal moment of time to think
about their actions - to help them overcome their
woes and defects - the Earth will enlighten men in
this process -as there is activity under apparent
stillness.
XIII. Answer any one of the following questions in 120-150 (1x5=5)
words:
(a) How did Mr. Lamb’s meeting with Derry become a
turning point in Derry’s life?
Answer: Derry, a complex-stricken lad of fourteen -
victim of inferiority complex - out of a misinterpretation
of himself and the world - suffers from an acute sense of
self-hatred and rejection due to his burnt face - leads
him to total alienation -Mr. Lamb has a tin leg - but
never allows his handicap to interfere with the pace and
pleasure of his life - turning point - Mr. Lamb’s meeting
with Derry - gives Derry confidence and persuades him
to develop a positive attitude towards life - His message
to Derry is very clear - Alienation and withdrawal is not
the solution -Derry believes in running away from
people but Mr. Lamb lives among people - Derry’s brief
association with Mr. Lamb proves to be quite rewarding
- Mr. Lamb teaches him a new perspective of looking at
life, people and things - decides never to get back to his
old seclusion - His burnt face will no longer interfere
with his self-respect, poise and confidence - gradually
learns to find his place in the mainstream of life - Mr.
Lamb helps Derry rediscover himself and gives a new
meaning to his life- Derry wants to leave his handicap
behind, forge ahead in life and live life to the fullest.
(b) Dr Sadao was a patriotic Japanese as well as a
dedicated surgeon. How could he honour both the
values?
Answer: Dr Sadao- a patriotic Japanese - a dedicated
surgeon. - An American run-away prisoner in a badly
wounded condition - washed ashore on Sadao’s
doorstep. The doctor - initially a little hesitant to save
the prisoner’s life when the discovers the latter’s real
identity - decides to treat the enemy soldier. - save the
prisoner’s life - but being a patriotic Japanese- informs a
senior Japanese Army General - agrees to have him
killed by the General’s assassins. - waits for the assassins
to carry out their job but when they do not arrive, he
gives the prisoner his boat and helps him to escape
safely - honour both the values - a patriotic Japanese as
well as a dedicated surgeon.
MODEL EXAMINATION 2020 – 21
CLASS- XII ACCOUNTANCY (055) TIME- 3 Hours
Marks:80
General Instructions:
1. This question paper comprises two Parts – A and B. There are 32 questions in the
question paper. All questions are compulsory.
2. Part A is compulsory for all candidates.
3. Part B has two options i.e. (1) Analysis of Financial Statements and (2)
Computerized Accounting. You have to attempt only one of the given
options.
4. Question nos. 1 to 13 and 23 to 29 are very short answer type questions
carrying 1 mark each.
5. Question nos. 14 and 30 are short answer type–I questions carrying 3
Marks each.
6. Question nos. 15 to 18 and 31 are short answer type–II questions
carrying 4 marks each.
7. Question nos. 19, 20 and 32 are long answer type–I questions carrying 6
marks each.
8. Question nos. 21 and 22 are long answer type–II questions carrying 8
marks each.
9. There is no overall choice. However, an internal choice has been provided
in 2 questions of three marks, 2 questions of four marks and 2 questions of
eight marks.
Part- A
(Accounting for Not for Profit organizations, Partnership firms and
Companies)
1. Receipts and payment account is based on _____. 1
a) cash principle
b) accrual principle
c) credit principle
d) none of these
2 In the absence of partnership deed, partners are entitled to 1
a) Salary b) Interest on loan & advances c) Commission d) Profit share in
capital ratio
3 A, B and C were partners in a firm sharing profits and losses in the ratio of 4: 1
3:2. B retired and his share was taken over by A and C equally. Calculate the
gaining ratio. (1) A. 4:2 B. 1:1 C. 3:2 D. 4:1
4 Give any one difference between reconstitution of a firm and dissolution of a 1
firm.
5 Reserve Capital is not a part of: (a) Authorized Capital (b) Subscribed capital 1
(c) Unsubscribed capital (d) Issued Share Capital
6 The profits for the last years are Rs. 60,000; Rs. 40,000 and Rs. 66,500. The 1
total assets of the firm are Rs. 10,00,000 and outside liabilities are Rs.
5,42,500. The rate of interest expected from capital invested is 10%. The value
of goodwill on capitalisation basis is:
a) Rs. 97,000 b) Rs. 97,250 c) Rs. 97,500 d) Rs. 97,750
7 Fixed Assets (Book Value Rs. 45,000) is undervalued by 10%. Calculate the 1
value (Dr./Cr.) to be shown in Revaluation account. (a) Rs. 4,500, Dr. (b) Rs.
4,500, Cr. (c) Rs. 5,000, Dr. (d) Rs. 5,000, Cr
8 A, B and C are partners sharing profits in the ratio 5:3:2. They decided to 1
share future profits in the ratio 2:3:5. What will be the accounting treatment
of Workmen compensation Reserve appearing in the balance sheet on that
date when no other information is available. (a) Distributed among partners in
capital ratio (b) Distributed among partners in old ratio (c) Distributed among
partners in new ratio (d) Carried forward to new balance sheet
9 State with reasons whether the following statement is ​TRUE​ or ​FALSE​: Receipt 1
and Payment Account records the receipts and payments of revenue nature
only
10 P/L Suspense A/c debited in the deceased partner’s capital account. 1
What does it indicate?
11 Which of the following statements is not correct? 1
a) A partnership business may include trade, vocation and profession but not
Joint ownership.
b) As an agent a partner represents other partners and binds them through
his act.
c) As a principal, a partner is bound by the act of other partners.
d) Interest on Partner’s Loan is 6% p.a. is fixed in all circumstances.

12 According to Accounting Standard -26 (AS-26) ___. 1


a) Self-generated goodwill is not accounted as an asset or
goodwill should not be raised in the books of accounts
b) Purchased goodwill should not be written off
c) Goodwill can be recorded in the books whether money or money’s worth
paid for it or not
d) Self-generated goodwill is always shown under the fixed assets
13 P and Q are partners having capitals of 15,000 each. R is admitted for 1/3rd
share for which he has to bring 20,000 for his share of capital. The amount of
1
goodwill will be
(a) 8,000 (b) 10,000 (c) 9,000 (d) 11,000
14 Calculate the amount of sports material to be debited to the I & E a/c of sports 3
club for the year ended 31.03.2020 on the basis of the following information:
PARTICULARS 01.04.2019 01.04.2020
Stock of sports material 7,500 6,400
Creditors for sports material 2,000 2,600
Amount paid for sports material during the year was ₹ 19,000
OR
On the basis of the following information given below, calculate the amount
of stationery to be debited to the income and expenditure account of Good
Health Sports Club for the year ended 31​st​ March, 2020

PARTICULARS 01.04.2019 01.04.2020


Stock of stationery 8,000 6,000
Creditors for stationery 9,000 11,000
Stationery purchased during the year ended 31​st​ March, 2020 was Rs 47,000

15 The firm of R, K and S was dissolved on 31​st March 2019. Pass necessary 4
journal entries for the following after various assets (other than cash and
Bank) and the third-party liabilities had been transferred to realisation
account.
(i) K agreed to pay off his wife’s loan of ₹ 6,000.
(ii) Total Creditors of the firm were ₹ 40,000. Creditors worth ₹
10,000 were given a piece of furniture costing ₹ 8,000 in full and
final settlement. Remaining creditors allowed a discount of 10%.
(iii) A machine that was not recorded in the books was taken over by
K at ₹ 3,000 whereas its expected value was ₹ 5,000.
The firm had a debit balance of ₹ 15,000 in the profit and loss a/c on the date
of dissolution.
OR
Ravi and Mukesh were partners in a firm sharing profits and losses equally.
On 31​st​ March, 2019 their firm was dissolved. On the date of dissolution
their balance sheet showed stock of ₹ 60,000 and creditors of ₹ 70,000. After
transferring stock and creditors to realization account the following
transactions took place.
i) took over 40% of total stock at 20% discount.
ii) 30% of total stock was taken over by creditors of ₹ 20,000 in full
settlement.
iii) Remaining stock was sold for cash at a profit of 25%.
iv) Remaining creditors were paid in cash at a discount of 10%.
Pass necessary journal entries for the above transactions in the books of
the firm.
16 X, Y and Z are partners sharing profits and losses in the ratio of 5:3:2. With 4
effect from 1st April, 2019, they decided to share profits and losses in the ratio
of 2:3:5.
On that date, their balance sheet showed following balances:
General reserve ₹ 90,000
Workmen compensation reserve ₹ 10,000
Advertisement suspense a/c ₹ 40,000
They decided to carry these balances in the balance sheet.
Pass the adjusting entry. Show working notes clearly.

17 Tanmay Ltd. had an authorized capital of 2,00,000 equity shares of Rs. 10 4


each. The company offered to the public for subscription 1,00,000 shares.
Applications were received for 97,000 shares. The amount was payable as
follows on application was Rs. 2 per share, Rs. 4 was payable each on
allotment and first and final call. A shareholder holding 600 shares failed to
pay the allotment money. His shares were forfeited. The company did not
make the first and final call. Present the share capital in the Balance Sheet of
the company as per Schedule III of the Companies Act, 2013. Also prepare
Notes to accounts
18 Dona, Anu and Prem are partners in a firm sharing profit and losses in the 4
ratio of 5:3:2. Their books closed on March 31​st​ every year.
Dona died on 30​th​ September 2019. The executors of Dona are entitled
to :
(i) Her share of capital Rs.5,00,000 along with her share of goodwill.
The total goodwill of the firm was valued at Rs.60,000.
(ii) Her share of profit up to her date of death on the basis of sales
till the date of death. Sales for the year ended 31, 2019 was
Rs.2,00,000 and profit for the same year was 10% on sales. Sales
shows a growth trend of 20% and percentage of profit earning is
reduced by 1%.
(iii) Amount payable to Dona was transferred to her executors.
Pass necessary journal entries and show the workings clearly.
19 VK Ltd issued 5,000, 9% debentures of Rs.10 each at par for cash which 6
redeemable at 10% premium and also raised a loan of Rs. 80,000 from
America Bank for which the company placed with the bank Rs. 1,00,000, 9%
debentures as collateral security. As per the terms, the bank is obliged and
bound to immediately release the debentures, as soon as the loan is repaid.
How will you show the debentures in the balance sheet of the company
assuming that the company has recorded the issue of debentures as collateral
in the books? Also pass the journal entries.
20 From the following Receipts and Payments Account, prepare Income and 6
Expenditure Account of Patel Education Society for the year ending 31st
March, 2019 and Balance Sheet as on that date:
RECEIPTS AND PAYMENTS ACCOUNT
for the year ended 31st March, 2019
Receipts Amount Payments Amount (Rs.)
(Rs.)
To Balance b/d 33,500 By Salaries 12000
To Entrance Fees 3,000 By Electricity 1200
To Subscription: Charges
Arrears 500 By Other Expenses 5250
Current Year 35,000 By Fixed Deposits 25000
Advance 7,50 By Utensils 2000
To Surplus on Sale of By Creditors 10000
Refreshments 1,000 By Balance c/d 25500
To Sale of Old Printer 4,000
(Book value – Nil)
To Miscellaneous Income 3,200

80,950 80,950
Additional Information:
((i) Following were the assets and liabilities as on 1st April, 2018: Utensils
Rs.8,000, Furniture Rs.25,000, Consumable stores Rs.3,500, Creditors
Rs.12,000.
(ii) On 31st March, 2019 Stock of Consumable Stores was Rs.7,000; Creditors
were Rs.5,500; Outstanding Subscription Rs.750; and Accrued Interest on
fixed deposits was Rs 250.
(iii) Charge depreciation on the closing balance of Furniture and Utensils @
10% and 15% respectively...
21 A, B & C were partners in a firm sharing profits & losses in proportion to their 8
fixed capitals. Their Balance Sheet as at March 31, 2017 was as follows:
Balance Sheet as at March 31, 2017
​LIABILITIES (​₹) ASSETS ​(₹)
Capitals Bank 21,000
A ₹ 5,00,000 Stock 9,000
B ₹ 3,00,000 Debtors ₹ 15,000
C ​₹ 2,00,000 10,00,000 Less: Provision ​₹ 1,500 13,500
General reserve 75,000 A’s loan 35,500
Creditors 23,000 Plant and machinery 2,00,000
Outstanding salary 7,000 Land and building 6,00,000
B’s Loan 15,000 P & L for the year ending
31​st​ March 2017 2,41,000
11,20,000 11,20,000

On the date of above Balance Sheet, C retired from the firm on the
following terms:
i. Goodwill of the firm will be valued at two years purchase of the Average
profit of the last three years. The Profits for the year ended March 31,
2015 & March 31, 2016 were ₹ 4,00,000 and ₹ 3,00,000 respectively.
ii. Provision for Bad Debts will be maintained at 5% of the Debtors.
iii. Land & Building will be appreciated by Rs. 90,000 and Plant & Machinery
will be reduced to ₹ 1,80,000.
iv. A agreed to repay his Loan.
v. The loan repaid by A was to be utilized to pay C.

The balance of the amount payable to C was transferred to his Loan


Account bearing interest @ 12% per annum.
Prepare Revaluation Account, Partners’ Capital Accounts, Partners’ Current
Accounts and the Balance Sheet of the reconstituted firm.
OR
Raman and Rohit were partners in a firm sharing profits and losses in the ratio
of 2:1. On 31st March, 2018, their Balance Sheet was as follows:
Balance Sheet of Raman and Rohit as at 31st March, 2018
Liabilities Amount Assets Amount
(Rs.) (Rs.)
Capital: Plant and Machinery 1,75,000
Ramana 1,40,000
Rohit ​ ,00,000
1 Furniture and Fixtures 65,000
2,40,000
Workmen Stock 47,000
Compensation Fund 40,000
Debtors 1,10,000
Creditors 1,60,000 Less: Provision
for doubtful debts ​7,000 1,03,000

Bank Balance 50,000

4,40,000 4,40,000

On the above date, Saloni was admitted to the partnership firm.


Raman surrendered 2/5th of his share and Rohit surrendered 1/5th of his
share in favour of Saloni.
It was agreed that:
(i) Plant and machinery will be reduced by Rs. 35,000 and furniture and
fixtures will be reduced to Rs. 58,500.
(ii) Provision for bad and doubtful debts will be increased by Rs. 3,000.
(iii) A claim for Rs. 16,000 for workmen’s compensation was admitted.
(iv) A liability of Rs. 2,500 included in creditors is not likely to arise.
(v) Saloni will bring Rs. 42,000 as her share of goodwill premium and capital
of Rs 50,000
Prepare Revaluation Account, Partners’ Capital Accounts and show the
working clearly.
22 Sam Paints Ltd. was registered with an authorised capital of ₹ 50,00,000 divided 8
in 5,00,000 equity shares of ₹ 10 each.
Company issued 2,00,000 equity shares at a premium of ₹ 3 per share,
payable as follows:
₹ 4 on application
₹ 5 on allotment (including premium)
₹ 2 on first call
₹ 2 on second and final call
All shares were subscribed and all the money was duly received.
Share issue expenses amounted to ₹ 75,000 which were fully written off
against securities premium.
Pass necessary journal entries.
OR
Krithik Ltd. issued 10,000, 10% debentures of ₹ 100 each, payable as follows:
₹ 10 on application
₹ 20 on allotment
₹ 30 on first call and
₹ 40 on second and final call
Aparna, who holds 500 debentures failed to pay the amount due on
allotment.
She however, pays this amount with the first call money.
Kala, who holds 800 debentures paid all the calls in advance on allotment.
Pass necessary journal entries.
Part – B
(Analysis of Financial statements)
23 Debt Equity Ratio is: 1
a) Solvency Ratio b) Liquidity Ratio c) Operating Ratio d) None of the above
24 Under which tool of Financial Statement Analysis, 100% is taken as a base and 1
all other related items are expressed as a percentage of base?
(a) Comparative Statement (b) Common Size Statement (c) Ratio analysis (d)
Cash Flow Statement
25 List the objective of calculating proprietary ratio 1
26 While preparing common-size balance sheet each item of Balance sheet is 1
expressed as % of
a. Current assets b. Non-current assets c. Non-current liabilities d. Total
assets.

27 What is meant by ‘financing activities’ for preparing a cash flow statement? 1


28 Why are loose tools and stores and spares not included in current assets while 1
calculating current ratio?
29 Match the following 1
i) Earning capacity a) Solvency ratio
ii) Short term credibility b) Profitability ratio
c) Liquidity ratio

30 i) Net profit after interest and tax of M Ltd. was ₹ 1,00,000. 3


Its current assets were ₹ 4,00,000 and current liabilities were ₹ 2,00,000.
Tax rate was 50%.
Its total assets were ₹ 10,00,000 and 10% long term debt was ₹ 4,00,000
Calculate return on investment.
ii) Rate of gross profit on revenue from operations of a company is 25%.
Its gross profit is ₹ 5,00,000.
Its shareholders’ funds are ₹ 25,00,000; Non-current liabilities are ₹
8,00,000 and non-current assets are ₹ 23,00,000
Calculate its working capital turnover ratio.
OR
From the following data, calculate Quick ratio:
Working capital Rs.2,50,000; Total Debts Rs.4,00,000; Long-term debt
Rs.3,20,000; Inventory Rs.2,00,000; Prepaid expenses Rs.10,000.

31 Given below is the information extracted from the books of Shyam Ltd: 4

Particulars 31-03-2019 31-03-2020

Revenue from Operations 25,00,000 40,00,000

Purchase of stock in trade 7,00,000 10,00,000

Change in inventories of stock in trade 2,75,000 1,50,000

Other expenses 25,000 50,000

Income Tax 40% 40%

Prepare Comparative Statement of Profit and Loss on the basis of the above
information.
OR
Prepare a common-size statement of profit and loss of ‘Hari Darshan Ltd.’ from
the following information:

Particulars Note 2018-19 2019-20


No.
Revenue from operations 20,00,000 10,00,000
Purchase of stock in trade 7,70,000 4,20,000
Change in inventories 1,20,000 80,000
Other expenses 52,000 30,000
Other income 60,000 50,000
Tax rate 50%
32 Calculate values of opening and closing inventories from the following 6

information:
Revenue from operations Rs.6,00,000; Gross profit ratio- 25 %
Inventory Turnover ratio -5times; Closing inventory is Rs.12,000 more than
the opening inventory
(b)Rate of gross profit on Revenue from operation of a company is 25%. Its
Gross profit is Rs.5,00,000; its Shareholders fund are Rs.25,00,000; Long term
Loan Rs.8,00,000 and Non-Current Assets were Rs23,00,000.
Compute Working Capital Turnover Ratio of the company.
Part – B (Computerised Accounting)

23 List any two attributes of information to be stored in Payroll data base. 1


24 Which of the following is not an advantage of computerised accounting 1
system?
a) Efficient record keeping
b) Ensures effective control over the system.
c) Generation of reports and information in fixed format
d) Economy in the processing of accounting data.
25 Differentiate between Generic software and Specific Software on basis of cost 1
of installation and maintenance
26 The data is classified for creating groups of accounts in the heads of: 1
(A) Assets, Liabilities and Capital (B) Assets, Owners’ equity, Revenue and
Expenses (C) Assets, Capital, Liabilities, Revenue and Expenses (D) Capital,
Revenue and Expenses
27 A1: E2 in Excel refers to: 1
(A) Column on Excel sheet (B) Row on Excel sheet (C) Column between start
and end points of Excel sheet (D) Alphabets between A to E on Excel sheet.
28 Name the data element in the accounting transaction. 1
29 What is the activity sequence of the basic information processing mode 1
30 Explain adjusting entries. 3
OR
Explain ‘Null Values’ and ‘Complex Attributes’.
31 Name and explain the function which returns the future value of an 4
investment which has constant payment and interest.
OR
Explain ‘Sequential’ and ‘Mnemonic’ codes.
32 Identify the error that appears when there are invalid numeric values in a 6
formula or function. How can this error be rectified? Explain.
​ACCOUNTANCY (055)
Qn NO. MARKING SCHEME Marks
Part- A
(Accounting for Not for Profit organizations, Partnership firms and
Companies)
1 a) cash principle 1

2 b) Interest on loan & advances 1

3 1:1 1
4 Reconstitution would result into reforming of Partnership firm with 1
certain changes and Dissolution would result into its closure of Business.
5 (c) Unsubscribed capital 1
6 c) Rs.97,500 1
7 5000 Cr 1
8 (b) Distributed among partners in old ratio 1
9 False. It records receipts and payments of revenue as well as capital 1
nature. (1/2 mark for answer and ½ mark for reason)

10 His share of loss transferred to his account. 1

11 a) A partnership business may include trade, vocation and profession 1


but not Joint ownership.

12 a) Self-generated goodwill is not accounted as an asset OR goodwill 1


should not be raised in the books of accounts.

13 (b) 10,000 1
14 3
​ ARTICULARS
P Amount
Amount paid for sports materials 19,000
Add: i) Creditors at the end of the year 2,600
ii) Opening stock 7,500
29,100
Less: i) Closing stock (6,400)
ii) Creditors at the beginning of the year (2,000)
Sports material consumed ​20,700

OR
Stock of stationery on 01.04.2019 ----------------- ₹ 8,000
Stationery purchased during 2019-20------------- ₹ 47,000
Less: Stock of stationery on 31.03.2020 --------- ₹​ (6,000)
Stationery consumed during 2019-20 -------- ​₹ 49,000

Amount to be debited to I & E a/c ₹ ​49,000

15 4
Date Particulars L.F Dr Cr

Realization A/c Dr 6000


To K’s Capital A/c 6000

Realization A/c Dr 27000


To Bank A/c 27000

K’s Capital Account Dr 3000


To Realization A/c 3000

R’s Capital A/c Dr 5000


K’s Capital A/c Dr 5000
S’s Capital A/c Dr 5000
To Profit and Loss A/c 15000

OR

Date Particulars L.F. Dr. ₹ Cr. ₹


i) Ravi’s Capital a/c Dr. 19,200
To Realisation a/c 19,200
(40% of the total stock taken over
by Ravi at 20% discount)
ii) No entry
iii) Cash a/c 22,500
To Realisation a/c 22,500
(stock sold for cash)
iv) Realisation a/c 45,000
To cash a/c 45,000
(Creditors paid in cash at a
discount of 10%)

16 4
Date Particulars L.F. Dr. Cr.
2019 Z’s Capital a/c Dr 18,000
April To X’s Capital a/c 18,000
(Being the adjustment entry)

Working notes:
1. Net effect of the balances of accumulated profits and losses
General reserve ₹ 90,000
Workmen compensation reserve ₹ 10,000
Advertisement suspense a/c ​ ₹ (40,000)
Net amount to be adjusted ​ ₹ 60,000
2. Calculation of sacrifice / gain due to change in profit-sharing ratio
X’s Sacrifice / Gain = 5/10 – 2/10 = 3/10 (sacrifice)
Y’s Sacrifice / Gain = 3/10 – 3/10 = 0 (no change)
Z’s Sacrifice / Gain = 2/10 – 5/10 = (3/10) (gain)
In this case Z has gained 3/10 in share whereas, X has sacrificed
3/10​th​ share.
Thus, Z will compensate x for his gained share
i.e., ₹ 60,000 X 3/10 = ₹ 18,000

17 Balance Sheet of Tanmay Ltd. 4


As at ....................(As per revised schedule VI)

Particulars Not Amoun Amount


e t (Rs.) (Rs.)
No. Current Previous
year year

EQUITY & LIABILITIES 1 5,79,60 --------------- 1


Shareholder’s fund : 0 -
a) Share Capital

Notes to Accounts

1. Share Capital

Authorised Capital​: 2,00,000 equity 20,00,000


shares of Rs.10 each
10,00,000
Issued Capital​ 1,00,000 equity shares 1
of Rs.10 each
5,78,400
Subscribed Capital
Subscribed but not fully paid 96,400 1,200 1
shares of Rs.10 each, Rs.6 called-up 5,79,600
Add: Forfeited shares A/c

18 Dona’s share of profit = 2,40,000 x9/100x5/10x6/12 =5,400 4


(Each entry carries one mark)
19 6
Date Particulars L. F Debit Credit
Bank A/c……………….Dr 50000
Loss on issue of Debenture 5000
A/C…...Dr 50000
To 9% Debenture A/C 5000
To Premium on Redemption
A/C
Bank A/C………………Dr 80000
To Bank Loan A/C 80000
(Being Loan raised from bank)
Debenture SuspensA/C…………..Dr 100000
To 9% Debenture A/C 100000
(debenture issues as collateral
Securities)
Profit & Loss A/C………………Dr 5000
To Loss on issue of 5000
Debenture A/C
(Being Loss on issue charged from
Profit & loss A/C)

Extract of Balance Sheet As on ………………………


Particulars Note No Amount

A. Equities & Liabilities


a. Equities
b. Non-Current Liabilities
i. 9% Debenture 150000
(-) Debenture Suspense ​100000 50000
ii. Bank Loan 80000
B. Assets
a. Non-Current Assets
b. Current Assets
i. Bank 13000

20 Surplus = 24750 Opening capital fund = 58500 6


Total of balance sheet =89500
21 8
OR

Loss of revaluation Rs. 42,000 (​3 marks)


BAL of capital account Raman 161600
Rohit 102400 ​(3 marks)
Saloni 132000
Balance sheet Total 569500 ​(2 marks)

22 8
Date Particulars L.F. Dr. (₹) Cr. (₹)
i Bank a/c 8,00,000
Dr. 8,00.000
To Equity share application a/c
(Money received on application)
ii Equity share application a/c 8,00,000
Dr. 8,00,000
To Equity share capital a/c
(Application money transferred to
share capital a/c)
iii Equity share allotment a/c 10,00,000
Dr. 4,00,000
To Equity share capital a/c 6,00,000
To Securities premium reserve a/c
(allotment, including premium due)
iv Banka/c 10,00,000
Dr. 10,00,000
To Equity share allotment a/c
(Allotment money received including
premium)
v Equity share first call a/c Dr. 4,00,000
To Equity share capital a/c 4,00,000
(money due on first call)
vi Bank a/c 4,00,000
Dr. 4,00,000
To Equity share first call a/c
(money received on first call)
vii Equity share second and final call a/c 4,00,000
Dr. 4,00,000
To Equity share capital a/c
(Money due on second and final call)
viii Bank a/c Dr. 4,00,000
To Equity share second and final call 4,00,000
a/c
(Money received on second and final
call)
ix Share issue expenses a/c Dr. 75,000 .
To Bank a/c 75,000
(Expenses incurred on issue of shares)
x Securities premium reserve a/c 75,000 .
To Share issue expenses a/c 75,000
(Share issue expenses written off
against securities premium reserve a/c)

OR

Date Particulars Dr. (₹) Cr. (₹)


L.F
.
i Bank a/c 8,00,000
Dr. 8,00,000
To Equity share application a/c
(Money received on application)
ii Equity share application a/c 8,00,000
Dr. 8,00,000
To Equity share capital a/c
(Application money transferred to
share capital a/c)
iii Equity share allotment a/c 10,00,000
Dr. 4,00,000
To Equity share capital a/c 6,00,000
To Securities premium reserve a/c
(allotment, including premium due)
iv Bank a/c 10,00,000
Dr. 10,00,000
To Equity share allotment a/c
(Allotment money received including
premium)
v Equity share first call a/c 4,00,000
Dr. 4,00,000
To Equity share capital a/c
(money due on first call)
vi Bank a/c 4,00,000
Dr. 4,00,000
To Equity share first call a/c
(money received on first call)
vii Equity share second and final call a/c 4,00,000
Dr. 4,00,000
To Equity share capital a/c
(Money due on second and final call)
viii Bank a/c Dr. 4,00,000 .
To Equity share second and fina 4,00,000
call a/c
(Money received on second and final
call)
ix Share issue expenses a/c Dr. 75,000 .
To Bank a/c 75,000
(Expenses incurred on issue of
shares)
x Securities premium reserve a/c 75,000 .
To Share issue expenses a/c 75,000
(Share issue expenses written off
against securities premium reserve
a/c)

OR

Date Particulars Dr. (₹) Cr.


L.F. (₹)
Bank a/c Dr. 1,00,000
I To 10% debentures application a/c 1,00,000
(Application money received on 10,000
debentures @ ₹10 per debenture)
ii 10% debentures application a/c Dr. 1,00,000
To 10% debentures a/c 1,00,000
(application money transferred)
10% Debentures allotment a/c Dr. 2,00,000
iii To 10 % debentures a/c 2,00,000
(Allotment due)
Bank a/c Dr. 2,46,000
iv To 10% Debentures allotment a/c 1,90,000
To Calls in advance a/c 56,000
(allotment money received on 9,500
debentures @ ₹ 20 per debenture; plus
call received in advance on 800
debentures @ ₹ 70 per debenture)
10% Debentures first call a/c Dr. 3,00,000
v To Debentures a/c 3,00,000
(First call due on 10,000 debentures @
₹ 30 each)
vi Bank a/c Dr. 2,76,000
Calls in advance a/c Dr. 24,000
To 10% Debenture first call a/c 3,00,000
(first call money received after
adjusting the advance of first call @ ₹
30 per debenture on 800 debentures of
Kala)
vii
Bank a/c 10,000
To 10% Debentures allotment a/c 10,000
(receipt of arrears of allotment in
respect of 500 debentures)
viii 10% debentures second and final call 4,00,000 .
a/c Dr. 4,00,000
To 10% Debentures a/c
(Second and final call due on 10,000
debentures @ ₹ 40 per debenture)
ix Bank a/c Dr. 3,68,000
Calls in advance a/c Dr. 32,000
To 10% debentures second and final 4,00,000
call a/c
(Second call money received after
adjusting the advance of second call @
₹ 40 per debenture on 800 debentures
of Kala)

Part – B
(Analysis of Financial statements)
23 a) Solvency Ratio 1

24 b) Common size
25 Proprietary ratio assesses the solvency of the concern. It shows the extent 1

to which fixed assets are financed by owner's fund.


26 d.Total assets 1

27 Financing activities are the activities that result in change in capital or 1

borrowings of the enterprise.


28 Because they are not held for conversion into cash. 1

29 1
i) Earning capacity b) Profitability ratio
ii) Short term credibility c) Liquidity ratio

i)- b ii) c
30 a) Return on investment = Profit before interest and tax / Capital employed 3

X 100
Profit before tax = 1,00,000 X 100/5 = ₹ 2,00,000
Add: Int. on long term debts = ​₹ 40,000
Profit before interest and tax = ​₹ 2,40,000
Capital employed = total assets – current liabilities
=₹ 10,00,000 – ₹ 2,00,000 = ₹ 8,00,000
Therefore, return on investment = ​2,40,000 / 8,00,000 X 100 = 30%

b) Working capital turnover ratio = Revenue from operations / Working


capital
Gross profit = ₹ 5,00,000
So, Revenue from operations = ₹ 5,00,000 X 100/25 = ₹
20,00,000
Working capital = Shareholder’s funds + Non-current liabilities –
noncurrent assets
= ₹ 25,00,000 + ₹ 8,00,000 – ₹ 23,00,000 = ₹ 10,00,000
Working capital turnover ratio = ₹ 20,00,000/10,00,000 = ​2times
OR
Current liabilities= Rs.80,000
Quick Assets Rs.1,20,000
Quick ratio= 1,20,000/80,000 = ​1.5:1
31 Profit after tax (31-03-2019) – 6,00,000 4

(31-03-2020) – 7,80,000

OR

In the books of ‘Hari Darshan Ltd.’ common size statement of profit and
loss
For the year ended 31​st​ March 2018 and 31​st​ March 2019
Particulars Absolute Absolute Percentage Percentage
amount amount of RFO of RFO
2017 – 18 2018-19 2017-18 % 2017-18 %
Revenue from 10,00,000 20,00,000 100 100
operations
Add: Other income 50,000 60,000 5 5

Total revenue 10,50,000 20,60,000 105 103


Less: Expenses . . . .
Purchase of stock 4,20,000 7,70,000 42 38.5
in trade
Change in 80,000 1,20,000 8 6.0
inventories
Other expenses 30,000 52,000 3 2.6
Total expenses 5,30,000 9,42,000 53 47.1
Profit before tax 5,20,000 11,18,000 52 55.90
Less: Tax @ 50% 2,60,000 5,59,000 26 27.95
Profit after tax 2,60,000 5,59,000 26 27.95

32 Opening inventory 84,000 closing inventory 96,000 6


Working capital turnover ratio- 2times
Part – B (Computerised Accounting)

23 Attributes of information to be stored in Payroll data base: (Any two) 1

(i) Name
(ii) ID
(iii) Designation
(iv) Location
(v) Basic Pay
24 c) Generation of reports and information in fixed format 1

25 Cost of installation and maintenance is generally low with Generic 1

Software and is relatively high with Specific software


26 (C) Assets, Capital, Liabilities, Revenue and Expenses. 1

27 (c) Column between start and end points of Excel sheet 1

28 A data element is the smallest named unit of data in the information 1

system. In accounting, a transaction consists of four data elements, such as


name of the account, accounting code, date of transaction and amount
29 The activity sequence of the basic information mode is collected data, 1

organize and process it and then communicate the information extracted.


30 The Adjusting entry is recorded to relate the figures to the trading period. 3

Suppose, premises have been sublet on March 31, and three months’ rent,
has been received in advance amounting to Rs. 12000. While preparing
accounts up to 31st March, one should taken into account only one month’s
rent for preparing the profit and loss account (accounting period concept);
the rest two month’s rent, already received is for the next year and will be
credited to profit and loss account next year. The adjusting entry will be:
Rent Account Dr
To Advance Rent Account
Rent Received in advance Account is a ‘Liability’ and is shown in the
balance sheet.
OR
Null Values​: Absence of data item is represented by a special value called
null value. There are three situations which may require the use of null
value.
​·​ When particular attribute does not apply to an entity.
​·​ Value of an attribute is unknown although it exists.
​·​ Unknown because it does not exist.
Complex Attributes​:
These are composite and multivalued attributes which may be nested
(or grouped) to constitute complex ones. The parenthesis { } are used for
showing grouping of components of composite attributes. The braces { }
are used for showing the multivalued attributes.
31 PMT :- The PMT function calculates the periodic payment for an annuity 4

assuming equal payments and a constant rate of interest.


The syntax of PMT function is as follows:
= ​PMT (rate, nper, pv, [fv], [type])
where Rate is the interest rate per period,
Nper is the number of periods,
Pv is the present value or the amount the future payments are worth
presently, future value or cash balance that after the last payment is made
(a future value of zero when we omit this optional argument)
Type is the value 0 for payments made at the end of the period or the value
1 for payments made at the beginning of the period. The PMT function is
often used to calculate the payment for mortgage loans that have a fixed
rate of interest
OR
Sequential codes These are the codes in which code numbers and/or letters
are assigned in a consecutive order. These codes are applied primarily to
source documents such as cheques, invoices etc. This facilitates document
searches. This process enables either identification of missing codes
(numbers) relating to a particular document or to trace a relevant document
on the basis of the codes. Mnemonic codes These codes consist of
alphabets or abbreviations as symbols to codify a piece of information. SJ
for sales journal, HQ for Headquarters are examples of mnemonic codes

32 The error is #NUM! Error. The steps to correct it are: 6

1. Optionally, click the cell that displays the error, click the button that
appears and then click show calculation steps.
2. Review the following causes and solutions:
● Using an unacceptable argument in the function that requires a
numeric argument.
● Make sure that the arguments used in the function are numbers.
● Using a worksheet function that iterates, such as IRR or RATE,
and the function cannot find the result.
● Use a different starting value for the worksheet function.
Then click the Microsoft button >Excel option and then click the formulas
category.
MODEL EXAMINATION 2020 – 21
CLASS – XII
SUBJECT – BUSINESS STUDIES

TIME: 3 HOURS MM: 80


General instructions:
1 This question paper contains 34 questions.
2. Marks are indicated against each question.
3. Answer should be brief and to the point.
4. Answers to the questions carrying 3 marks may be from 50 to 75
words.
5. Answers to the questions carrying 4 marks may be about 150
words.
6. Answers to the questions carrying 6 marks may be about 200
words.
7. Attempt all parts of the questions together.
1 Grouping of activities on the basis of functions is a part of 1
(a) Decentralised organisation (b) Divisional organisation
(c) Functional organisation (d) Centralised organisation

2 The Reserve Bank of India reduces CRR (Cash Reserve Ratio) by 1


0.5% to control recession. This is related to which dimension of
Business Environment.
(a) Political Environment (b) Economic Environment
(c) Social Environment (d) Legal Environment

3 The process by which a manager integrates and synchronises the 1


activities of different department is called
(a) Management (b) Controlling
(c) Planning (d) Coordination

4 All the forces of Business Environment are_______________ 1


(a) Relativity (b) Inter related
(c) Uncertain (d) None of the above

5 Management is a process because 1


(a) It involves series of steps (b) It leads to systematic working (c)
It involves one single activity (d) None of the above

6 Introduction of mobile phone have negatively affected the business of 1


watches and cameras "Which factor of business environment is
related with the sentence.
(a) Social environment (b)Technological environment
(c) Political environment (d)Legal environment
7 Name the method of training in which the trainee learns under the 1
guidance of a master worker.
a. Internship Training b. Apprenticeship Programme
c. Vestibule Training d. Campus Recruitment
8 Under which philosophy of marketing, efforts are made to bring down 1
the cost of production to the minimum?
(a)Production concept (b)Product concept
(c)Marketing concept (d)Selling concept
9 ____________ is the step in the planning process where other 1
managerial functions also curve into picture.
(a) Evaluating alternative course of actions (b) Selecting an
alternative
(c) Implementing the plan (d) Following action
10 Controlling function of an organisation is _________________ 1
(a) Forward looking
(b) Backward looking
(c) Forward as well as backward looking
(d) None of the above
11 In order to get feedback about its recently launched immunity 1
boosting ayurvedic medicine, Adarsh limited conducted an online
survey using a questionnaire, to gather customer views and opinions.
Identify the marketing function being used by Adarsh Limited:
(a) Standardisation
(b) Product Designing
(c) Customer support service
(d) Gathering and analysing market information

OR
A bottle of hair oil is an example of
(a) Primary packaging
(b) Secondary packaging
(c) Transportation packaging
(d) Sales promotion

12 Evenly distribution of authority at every level lead to 1


(a) Delegation (b) Decentralisation
(c) Informal Organisation (d) Formal Organisation

OR

_____________ is a process by which the manager brings order out


of chaos, removes conflict among people over work or responsibility
sharing and creates an environment suitable for teamwork.
(a) Planning
(b) Controlling
(c) Organising
(d) Directing

Read the following text and answer question No. 13-16 on the basis
of the same:

„UFT Steel Ltd‟. is a large and creditworthy company manufacturing


steel for the Indian market. It now wants to cater to the Asian market
and decides to invest in new hi-tech machines. Since the investment
is large, it requires long-term finance. It decides to raise funds by
issuing equity shares. The issue of equity shares involves huge
floatation cost. To meet the expenses of floatation cost the company
decides to tap the money-market.

13 Name and explain the money-market instrument the company can 1


use for the above purpose.
(a) Call money
(b) Treasury bill
(c) Commercial paper
(d) Certificate of deposit

14 What is the duration for which the company can get funds through 1
this instrument?
(a) 91 days to one year
(b) 15 days to one year
(c) One day to 15 days
(d) One year

15 State any other purpose for which this instrument can be used. 1
(a) It can be used for seasonal capital need
(b) It can used for working capital need
(c) Both a and b
(d) None of the above.

16 Identify the market UFT Steel Ltd. will look for raising fund. 1
(a) Primary market
(b) Secondary market
(c) Both a and b
(d) None of the above

Read the following text and answer question No.17-20 on the basis of
the same:
Aditya Crackers Ltd., a fire-cracker manufacturing company located in
Hyderabad, launched some new products on the eve of Diwali in the
market which attracted many buyers due to its low prices. The
demand increased and to meet this increased demand, the company
employed people from the nearby villages where there was a lot of
unemployment. Very soon the products were a hit in the market that
initiated the company official to get it registered with a special name
“Charminar”. Their competitors starting to imitate their products but
failed to replicate the success of company. Aditya Ltd. started to send
the products to other states also. They appointed several Distributors
in big cities and offered attractive discounts to them. Company also
used an advertising agency to aware the people that the products
were least harming the environment as compared to their
competitors. The Company was very much keen to cater the interest
of society by focusing on using less toxic material while production
and thus all the preventive measures were taken while production of
fire crackers to secure the environment.

17 …...Very soon the products were a hit in the market that initiated the 1
company official to get it registered with a special name “ Charminar”
With reference to the given text “Charminar” is a ____________ to
protect the product from the competitors‟ imitation.
(a) Brand Value (b) Trade mark
(c) Brand mark (d) None of these.
18 Aditya Crackers Ltd., a fire-cracker manufacturing company located in 1
Hyderabad, launched some new products on the eve of Diwali in the
market which attracted many buyers due to its low prices……...
With reference to the text above, which element of marketing mix has
been discussed here:
(a) Price (b) Place
(c) Both (a) and (b) (d) Neither (a) nor (b)
19 The Company was very much keen to………………………. 1
In this statement, company tried to follow one marketing philosophy.
Identify the same:
(a) Marketing concept (b) Selling concept
(c) Societal Marketing concept (d) Production concept
20 Company also used an advertising agency to aware the people that 1
the products were least harming the environment as compared to
their competitors…………….
With reference to the text above, which element of marketing mix has
been used here:
(a) Price (b) Product
(c) Place (d) Promotion
21 Arnav was working in „Axe Ltd‟, a company manufacturing air 3
purifiers. He found that the profits had started declining from the last
six months. Profit has an implication for the survival of the firm, so he
analysed the business environment to find out the reasons for this
decline.
(a) Identify the level of management at which Arnav was working.
(b) State two other functions being performed by Arnav

22 What do you understand by labelling? State any two functions of 3


labelling in the marketing of the product
23 Romans Ltd. is engaged in manufacturing high end luxury pens. The 3
target production is 700 units daily. The company had been
successfully attaining this target until three months ago. Over the last
few months, it has been observed that daily productions vary
between 600-650 units.
(a) Discuss the steps involved in the process of the function discussed
above.
OR
State any three points of importance of controlling.

24 State any three merits of internal sources of recruitment. 3


OR
State any three limitations of external sources of recruitment.
25 Nishant, the director of a garment company, is planning to 4
manufacture bags for the utilisation of waste material from one of his
garment‟s unit. He has decided that his manufacturing unit will set-up
in a rural area of Odisha where people have very few job
opportunities and labour is available at very low rates. He also
thought of giving equal opportunities to men and women.
For this, he wanted four different heads for sales, accounts, purchase
and production. He gave an advertisement and short listed ten
candidates per post after conducting different selection tests.
Identify and state the next three steps for choosing the best
candidate out of the short-listed candidates.

26 Tijo was supervisor at a „Annapurna Aata‟ factory. The factory was 4


producing 200 quintals of Aata every day. His job was to make sure
that the work went on smoothly and there was no interruption in
production. He was good leader who would give orders only after
consulting his subordinates and work out the policies with the
acceptance of the group.
Identify and describe the leadership style being adopted by Tijo.
27 Sahad was working in enterprise on daily wages basis. It was difficult 4
for him to fulfill the basic needs of his family. His daughter fell ill. He
had no money for his daughter‟s treatment. To meet the expenses of
her treatment, he participated in a cycle race and won the prize
money. The cycle company offered him a permanent pensionable job
which he happily accepted.
(a) Identify the needs of Sahad that are satisfied by offer of cycle
company.
(b) (b) Also, explain two other needs of Sahad followed by above
that are still to be satisfied.
OR

Prem is working in a multinational company in Noida. He was running


a temperature for the last many days. When his blood was tested, he
was found to be positive for malaria. He was admitted in a hospital
and a blood transfusion was advised by the doctors as his condition
was very serious. One of his colleagues sent a text message to his
superior, Mr. B. Chatterjee. Mr. B. Chatterjee immediately sent a text
message to the employees of the organisation requesting them to
donate blood for Prem. When the General Manager came to know
about it, he ordered for fumigation in the company premises and
cleaning the surroundings.
1. From the above paragraph, quote lines that indicate formal and
informal communication.
2. State any two features of informal communication

28 Distinguish between delegation and decentralization (any four basis). 4


OR
Explain any four points of importance of delegation of authority.
29 Explain any four factors that can affect the fixed capital requirement 4
of a company.
OR
Explain any four factors that can affect the dividend decision of a
company.
30 Explain any two rights of consumers provided by Consumer Protection 4
Act, 1986.
31 What is meant by capital structure? Explain any five factors which 6
affect the choice of capital structure of a company.
32 „Failing to plan is planning to fail.‟ In the light of the statement 6
explain any four importance of planning as a function of
management.
OR
„Rapid Ltd.‟ is a well-known automobile manufacturing company in
India. The company plans to increase the sale of its sedan cars by
20% in the next quarter. In order to achieve the desired target, the
marketing team of the company considers the impact of policy of the
government towards diesel vehicles and the level of competition in
this segment of cars. They explore the various available options like
offering more discounts to dealers and customers, providing more
customer friendly finance options, lucky draws on test drive,
increasing advertising, offering more of free accessories on the
purchase of the car, etc. A thorough analysis of the various available
options is done keeping in view the relative viability of each option.
The company decides to pursue the option of offering more discounts
to dealers and customers in order to boost the sale of sedan cars. In
order to implement the plan, they determine the various discount
packages and communicate the same to their product dealers. To
make the prospective consumers aware about the new available
benefits, advertisements are made through various sources of print
and electronic media. The market analysts of the company keep a
close watch on the revenue from the sedan cars to study the effect of
new initiatives by the company to promote its sales.
In context of the above case:
1. Name the function of management described in the above
paragraph.
Identify and explain the various steps involved in process the function
of management as identified in part (a) by quoting lines from the
paragraph.
33 Kushal Ltd. is a leading automobile company in which the various 6
department are setting up their own objective without paying any
interest to the organizational objectives.
I. Which aspects of management of company is lacking? What will be
impact on the organization?
II. Identify the principle of management which has been overlooked
by the organization.
III. State any two value which is neglected by the people of
organization.
34 Various tools of communication are used by the marketers to promote 6
their products.
(a) Why do companies use all tools at the same time?
(b) Name and explain the most commonly used non-personal tool of
promotion which is paid for by the marketer.
(c) Which tool of promotion will primarily be used for the following?
(i) To promote or protect a company‟s image or its individual
products.
(ii) An existing product meant for mass usage by literate people.
(iii) To introduce a new product to a particular class of people through
door to-door visits.
(iii) To attract attention of the people by using incentives
OR
Dayaram‟ is a famous chain selling a large variety of products in the
Indian market. Their products include chips, biscuits, sweets and
squashes. It charges a comparatively higher price than its
competitors as it sells quality products. Besides, it offers regular
discounts to its customers and easy credit terms to its retailers. It has
five of its own retail shops. It also sells its products through various
grocery stores so that the products are made available to customers
at the right place, in the right quantity and at the right time. It
regularly uses different communication tools to increase its sales.
The above paragraph describes the combination of variables used by
Dayaram to prepare its market offering. Identify and explain the
variables.
MARKING SCHEME

BUSINESS STUDIES 2020-21

Question Marks
No.
1 Functional organisation 1
2 Economic environment 1
3 Coordination 1
4 Inter related 1
5 It involves series of steps 1
6 (b)Technological environment 1
7 b. Apprenticeship Programme 1
8 (a) Production concept 1
9 (c) Implementing the plan 1
10 Forward as well as backward looking 1
11 (d) Gathering and analysing market information 1
OR
Primary packaging

12 Decentralisation 1
OR
Organising
13 (a) Commercial paper 1

14 (a) 15 days to one year 1

15 (a) Both a and b 1

16 (a) Primary market 1


17 (b) Trade mark 1
18 (a) Price 1
19 (c) Societal Marketing concept 1
20 (d) Promotion 1
21 (a) Top level 3
Any two functions performed by top level management
22 Meaning (1mark) 3
Any two functions (1 mark each)
● Describe the product
● Identify the product
● Help in grading
● Promote sales
● Provide information

23 1. Setting Performance Standards 3


2. Measurement of actual performance
3. Comparing actual performance with standards
4. Analysing deviations
OR
1. Accomplishing organisational goals
2. Making efficient use of resources
3. Ensuring order and discipline
4. Improves employee motivation
5. Judging accuracy of standards
6. Facilitates coordination in action (any three
points)
24 1 mark for each merit of internal sources of recruitment 3
OR
1 mark for each limitation of external sources of
recruitment
25 4
1. The next three steps to be followed by Nishant in
the selection process after giving an advertising
and short listing ten candidates per post after
conducting different selection tests are as follows:

Employment Interview: It is a face-to-face


interaction between the interviewers and
prospective candidate. It involves a formal, in-
depth conversation that is conducted to evaluate
the applicant‟s suitability for the job.

Reference and Background Checks: At the time of


filling up of the job application form,the
prospective candidates are required to provide
names, addresses, and telephone numbers of
references for the purpose of verifying information
and gaining additional information about him/her.

Selection Decision: The final decision about the


selection is made from among the candidates who
pass the tests and interview.

26 Democratic style of leadership 4


27 1. Physiological and safety/security needs. 4
2. Other needs
a. Affiliation/ Belongingness needs
b. Esteem needs

OR
Informal communication: “One of his colleagues sent a
text message to his superior, Mr. B. Chatterjee. Mr. B.
Chatterjee immediately sent a text message to the
employees of the organisation requesting them to
donate blood for Sahad.”
Formal communication: “When the general manger
came to know about it, he ordered for fumigation in the
company premises and cleaning surroundings.”
The features of informal communication are as follows:
● The grapevine/ informal communication spreads
very fast and sometimes gets distorted.
● It is very difficult to detect the source of such
communication.

28 1 mark for each correct difference between delegation 4


and decentralization
OR
1 mark for each correct importance (4x1)
29 Factors affecting fixed capital requirements: 4
1. Nature/type of business
2. Scale of operations
3.Choice of techniques
4. Technology upgradation
5. Growth prospects
6. Diversification
7. Financing alternatives
8. Level of collaboration
OR
Factors affecting dividend decisions:
1. If the earnings of the company are high, dividends
are paid at a higher rate.
If the earnings of a company are stable, it is likely to
pay higher dividends.
2. A company is more likely to maintain a stable
dividend rate over a period of time, unless there is a
significant change in its earnings.
3. A company planning to pursue a growth opportunity
is likely to pay lower dividends.
4. The dividends are paid in cash, therefore if the cash
flow of the company is good, it is likely to pay higher
dividends.
5. If the shareholders prefer regular income in form of
dividends, the company is likely to maintain a dividend
pay-out rate.
6. If the tax rate is high, the company is likely to pay
less dividend.
7. If a company wants positive reactions at stock
market, It Is likely to pay higher dividends.
8. A large company can access funds easily from capital
market as per its requirements, therefore, it is likely to
retain lesser profits and is likely to pay higher dividends.
9. The legal constraint should be considered at the time
of dividend payment by a company.
10. The contractual constraints may also affect the
dividend payment by a company.

30 Any two rights of consumers 2 marks each 4


31 Meaning of capital structure (1 mark) 6
1 mark for each correct factor (5x1)
32 Importance of planning: 6
1. Planning provides direction as it acts as a guide for
deciding what course of action should be taken to attain
the organisational goals.
2. Planning reduces the risk of uncertainty arising due
to the dynamic nature of business environment as it
enables a manager to anticipate and meet changes
effectively.
3. Planning reduces overlapping & wasteful activities as
it serves as the basis for coordinating the activities and
efforts of different divisions and individuals.
4. Planning promotes innovation as it encourages new
ideas that can take shape of concrete plans.
5. Planning facilitates decision making as it enables a
manager to choose the best alternative course „of action
among the various available alternatives in light of
present and future conditions.
6. Planning establishes standards for
controlling. Planning provides standards against which
the actual performance is measured and timely
corrective actions the taken. (Any Four)
OR

1. Planning is the function of management which is


being described in the above paragraph.
2. The various steps involved in the planning process
are explained below:

● Setting objectives: The planning process is


initiated by setting the objectives in clear,
specific and measurable terms. The objectives
may be set for the organization as a whole
and for each department or unit within the
organization.
“The company plans to increase the sale of its
sedan cars by 20% in next quarter.”
● Developing Premises: Planning process is
carried out keeping in view the assumptions
related to the future, which is uncertain.
These assumptions are called premises and
may relate to government policy, interest
rate, inflation, etc. Accurate forecasts are
therefore essential for successful planning.
“In order to achieve the desired target the
marketing team of the company considers the
impact of policy of the government towards
diesel vehicles and level of competition in this
segment of cars.” .
● Identifying alternative courses of
action: The next step in the planning process
involves identification of the various ways in
which the goals can be achieved.
“They explore the various available options
like offering more discount to dealers and
customers, providing more customer friendly
finance options, lucky draws on test drives,
increasing advertising, offering more of free
accessories on the purchase of the car, etc.”
● Evaluating alternative courses: In order to
select the best option, the relative positive
and negative aspects of each alternative
should be evaluated in the light of their
feasibility and consequences.
“A thorough analysis of the various available
options is done keeping in view the relative
viability of each option.”
● Selecting an alternative: The best plan is
adopted to achieve the desired goals.
Sometimes, a combination of plans may be
selected instead of one best course of action.
“The company decides to pursue the option of
offering more discount to dealers and
customers in order to boost the sale of sedan
cars.”
● Implement the plan: This step is concerned
with putting plans into action.
“In order to implement the plan, they
determine the various discount packages and
communicate the same to their product
dealers. To make the prospective consumers
aware about the new available benefits,
advertisements are made through various
sources of print and electronic media.”
● Follow up action: Monitoring of plans is
equally important to ensure that objectives
are achieved efficiently and effectively.
“The market analysts of the company keep a
close watch on the revenue from the sedan
cars to study the effect of new initiatives by
the company to promote its sales.”

33 I. The company is lacking „coordination‟. Its different 6


department like production, marketing, etc. do not
coordinate this wok in absence of coordination; these
will be overlapping and chaos instead of harmony and
integration of activities. The company will fail to achieve
its objectives.
II. Subordinate of individual interest to general interest.
Values -Mutual cooperation, unity of action, optimum
utilization of resources.
34 (a) The companies use all tools at the same time 6
because of the nature of market, nature of product,
promotion budget and objectives of promotion being
different at different times.
(b) The most commonly used non-personal tool of
promotion which is paid for by the marketer is
advertising. It is a paid and non-personal form having
an identified sponsor to promote an idea.
(c) The tools of promotion used are:
1. Public relations
2. Advertisements
3. Personal selling
4. Sales promotion
OR
The combination of variables used by Haryaramto
prepare its market offerings are described below:

● Product: “Their products include chips, biscuits,


sweets and squashes.”
A product is anything of value i.e. a product or
service offered to a market to satisfy its needs or
wants. A product includes physical product, after
sale service, handling grievances etc. Every
marketer needs to constantly review and revise its
products in order to enhance customer‟s
satisfaction and have a competitive edge.
● Price: “It charges a comparatively higher price
than its competitors.”
Price is the monetary value paid in consideration
for purchase of a product or service by a buyer to
its seller. It is a very crucial decision for the
marketers as consumers are very sensitive to the
pricing. The factors affecting price determination
are cost of product, the utility and demand, extent
of competition in the market, government and legal
regulations, pricing objectives and marketing
methods used.
● Place: “It has five of its own retail shops.”Or “It
also sells its products through various grocery
shops.”
It is considered an important element of marketing
mix because it includes all those activities that help
in making the goods and services available to the
prospective buyers in the right quantity, at the
right time and in right condition. The two main
decisions involved in physical distribution are
physical movement of goods from producers to
consumers and choice of channels of distribution.
● Promotion: “It regularly uses different
communication tools to increase its sales.”
Promotion refers to the set of activities undertaken
by a marketer to inform the prospective buyers
about the product and persuading them to make a
purchase. The various components of promotion
mix are advertising, sales promotion, personal
selling and public relation.”
MODEL EXAMINATION – 2020-2021

ECONOMICS

Grade :12 Time : 3Hrs


Marks : 80

General instructions:
i. All the questions in both the sections are compulsory. Marks for questions are indicated
against each question.
ii. Question number 1 - 10 and 18 - 27 are very short-answer questions carrying 1 mark
each. They are required to be answered in one word or one sentence each.
iii. Question number 11 - 12 and 28 - 29 are short-answer questions caring 3 marks each.
Answers to them should not normally exceed 60-80 words each.
iv. Question number 13 - 15 and 30 - 32 are also short-answer questions carrying 4 marks
each. Answers to them should not normally exceed 80-100 words each.
v. Question number 16 - 17 and 33 - 34 are long answer questions carrying 6 marks each.
Answers to them should not normally exceed 100-150 words each.
vi. Answer should be brief and to the point and the above word limit be adhered to as far
as possible.

SECTION-A (Macro Economics)

1. Higher the legal reserve ratio __________ will be the credit control
( a ) higher ( b) lower ( c ) constant ( d ) none of these (1)

2. D8 Receipts which do not affect asset or liability status of government (1)


(a) capital receipts ( b ) revenue receipts ( c ) budgetary receipts ( d ) none of these

3. MPC being equal to 0.5, --------- will be ∆C, if income increases by 100 rupees.
(choose the correct alternative) (1)
a) 0.5
b) 50
c) 500
d) None of these

4. Name any two quantitative tools to control credit creation in an economy.


OR
Page 1 of 5
Define Fiat money (1)

5. Policy related to revenue and expenditure of the Government is known as ------------


(Choose the correct alternative) (1)
a) Monetary Policy
b) Foreign Trade Policy
c) Fiscal Policy
d) None of these

6. Revenue made by Govt from the property which do not have legal heir is known as-------
-----(Fill up the blank with correct answer) (1)

7. State whether the given statement is true or false: (1)


“There is an inverse relation between rate of foreign exchange and demand for foreign
exchange”.

8. The sum of MPC and MPS is always equal to --------------(Fill up the blank with
correct answer) (1)

9. From the set of statements given in Column I and Column II, choose the correct pair of
statements: (1)
Column I Column II
(a) At the Break Even point (i) Equals National Income
(b) If MPC is equal to one (ii) Value of k is infinity
(c) Value of aggregate demand (iii) Depends upon investment
(d) Consumption in the economy (iv) C > Y

10. The difference between fiscal deficit and interest payment is called________. (1)
(Fill up the blank with correct answer)

11. Households and firms depend on each other in a circular flow of income”. Justify the
statement in case of a Two- sector model. (3)

12. “Economists are generally concerned about rising Marginal Propensity to Save (MPS) in
an economy”. Explain Why?
OR
If Personal disposable income is Rs. 1000 crore and the consumption expenditure is Rs.
750 crore, find out the Average Propensity to save. (3)

13. Discuss any one of the following functions of a central bank:


a) As Government‟s bank b) Open market operations. (4)

14. „„Through its budgetary policy government allocates resources in accordance with the
requirements of the country.‟‟ Do you agree with the given statement? Justify your
answer with Valid reason.
OR

Page 2 of 5
DO you think Non-tax receipts are an important part of the budget? Examine different
sources of Non-tax receipts in the budget. (4)

15. Supply of foreign currency rises in response to a rise in its exchange rate .How? (4)

16. From the following data, calculate National Income by Income Method:
(a) Find Net value added at factor cost: (3)
Items Rs in lakh
i) Durable use producer goods with a life span of 10 years 10
ii)Single use producer goods 5
iii) Sales 20
iv)Unsold output produced during the year 2
v) Taxes on production 1

(b) How should the following be treated in estimated national income of a country? You
must give reason for your answer. (3)
i) Taking care of aged parents.
ii) Payment of corporate tax.
iii) Expenditure on providing police services by the government.
OR
Explain any 6 precautions to be taken while using the Income method. (6)

17. Diagrammatically represent Excess Demand. Also, analyze the monetary methods to
correct the Excess demand. (6)

SECTION-B (Indian Economic Development)


18. National Rural Employment Program me was launched in ___________
( a ) 1965 ( b ) 1980 ( c ) 1970 ( d ) 1981 (1)

19. State whether the following statement is true or false (1)


Urban Poverty is a spill-over of rural poverty.

20. Identify the correct sequence of alternatives given in Column II by matching them with
respective terms in Column I: (1)

Column – I Column – II
A. Land Ceiling i. Increase in production of food grain using high yielding variety
seeds
B. Land reforms ii. Portion of agricultural produce sold in the market
C. Green Revolution iii. Fixing the maximum limit of land holding for an individual.
D. Marketed Surplus iv. Change in the ownership of land (land to tillers)

21. The scheme of Micro-finance is extended through ---------- (Choose the correct
alternative) (1)
(a) Self- help groups
(b) Land development banks
Page 3 of 5
(c) NABARD
(d) Regional rural banks

22. HYV seeds were also called ................ (Fill up the blank with correct answer)
OR
------------------is known as the “Indian Father of Green Revolution” (Fill up the blank
with correct answer) (1)

23. _______ has the highest GDP growth rate as compared to India and Pakistan (1)

24. Choose the Correct alternative (1)


Unemployment is calculated as:
(a) Labour force – Work force
(b) Labour force + Work force
(C) None of these

25. --------------- (1951/ 1921) year is known as the „Year of Great Divide‟
(Fill up the blank with correct alternative). (1)

26. Arrange the following policies in chronological order (based on their launching date)
and choose the correct alternative: (1)
i. Sampoorna Gramin Rozgar Yojana (SGRY)
ii. MUDRA Bank
iii. Swarna Jayanti Shahri Rozgar Yojna (SJSRY)
iv. Twenty Point Programme
Choose the correct alternative:
a) IV, III, I, II
b) I, II, III, IV
c) IV, III, I, II
d) II, III, I, IV

27. ----------------was set to replace LQP in 1991. (Choose the correct alternative) (1)
(a) LPG
(b) PPC
(c) LPE
(d) None of these

28. How will you distinguish Labour Supply from Labour force? (3)
OR
How will you distinguish Open unemployment from Structural unemployment?

29. Describe the path of development initiatives taken by Pakistan for its economic
development ? . (3)

30. Examine the role of On- the job training and migration in enhancing the quality of
human capital. (4)

Page 4 of 5
31. Enumerate any 4 arguments against the NEP,1991 (4)

32. Define the following terms (4)


a) Disinvestment b) Import Substitution
OR
Define the following terms
a) Outsourcing b) Quota

33. Explain the institutional sources of rural credit. (6)

34. Evaluate PAPS and write any 6 reasons why Poverty Alleviation programmes failed to
deliver the desired results in some areas?
OR
Examine the role of education in the economic development of a nation ? (6)

Page 5 of 5
Q.NO MODEL EXAMINATION 2020-21
ANSWER KEY
1. Higher
2. Revenue Receipts
3. 50 (MPC=change in consumption/ Change in income)
4. Bank Rate and Open market operations are the Quantitative
measures.
OR
Fiat money is government-issued currency that is not backed by
. a physical commodity, such as gold or silver, but rather by the
government that issued it.
5. (C)Fiscal Policy
6. Escheat
7. True
8. The sum of MPC and MPS is always equal to one
9. (b) If MPC is equal to one – (ii) Value of K is infinity
10. Primary deficit
11. Circular flow

12. Households dispose of their post-tax income by spending or


saving. Saving is a withdrawal from the circular flow of
income and it has a pivotal role in determining changes in
national income over time. This is known as Micro macro
paradox. What is true at the micro level, may not be true at the
macro level.

Page 1 of 1
OR

Y= C+S, therefore S=Y-C


S= 250 Crores
APS= S/Y= 250/1000= 0.25

13. A)The RBI acts as banker to the government the Central as


well as state governments. It transacts all banking business of the
government, which involves the receipt and payment of money
on behalf of the government and carrying out of itsexchange,
remittance and other banking operations. As government’s
banker, the RBI provides short-term credit to the government to
meet any shortfalls in its receipts over its disbursements.
B) Open Market operations: It is the simultaneous sale and
purchase of securities also known as open market operations – on
behalf of the government. Open market operations enable a
central bank to adjust liquidity or supply of money in the banking
system.

14. Through the budgetary policy, government tries to reallocate


resources in accordance with the economic and social priorities
of the country. Government can influence allocation of resources
through tax concessions or subsidies. Government discourages
the production of harmful consumption goods like liquor,
cigarettes etc. through the heavy taxes and encourages useful
goods by using subsidies.
OR
Non tax receipts The estimates of Non-Tax Revenue
receipts from various sources such as return on assets in form of
dividend and profits, interest, fees, fines and

Page 1 of 1
miscellaneous receipts collected in the exercise of sovereign
functions, regulatory charges and license fees and user charges
for publicly provided goods and services.
Escheat refers to the income of the state which arises out of the
property left by the people Without an legal heir. Grants and
donations also form a part of non- tax receipts.

15. The supply of foreign currency rises in the following


situations: When price of a foreign currency rises, domestic
goods become relatively cheaper. It induces the foreign country
to increase their imports from the domestic country. As a result,
supply of foreign currency rises.

16. a) Net value added at factor cost = 15 lakhs.


(b) i) Taking care of aged parents should be included in the
estimation of national income as it involves generation of
services that are rendered for the parents.

ii) Payment of corporate tax should not be included in the


estimation of national income because it is a transfer payment by
the firm. It is paid out of income and therefore it is not to be
separately added in the national income. 1
iii) Expenditure on providing police services by the government
should be included in the estimation of national income because
expenditure incurred by the government is a part of government’s
final consumption expenditure.
OR
Transfer earnings like old age pensions should not be included.
Income from illegal activities is not be included.
Income from Windfall gains should not be included.

Page 1 of 1
Imputed rent of owner occupied houses is to be treated along
with rent.
Corresponding to production for self- consumption, there should
be generation Of income.

17. During excess demand, central bank increases the bank rate,
which raises the cost of borrowings from the central bank. It
forces the commercial banks to increase their lending rates,
which discourages borrowers from taking loans. During excess
demand, central bank offers securities for sale. Sale of securities
reduces the reserves of commercial banks.
It adversely affects the bank’s ability to create credit and
decreases the level of aggregate demand in the economy. To
correct the excess demand, the central bank increases CRR or/and
SLR. It reduces the amount of effective cash resources of
commercial banks and limits their credit creating power. When
the economy is suffering from excess demand, central bank
increases the margin, which restricts the credit creating power of
banks.

SECTION-B (Indian Economic Development)


18. 1980
19. Urban Poverty is a spill-over of rural poverty : TRUE

Page 1 of 1
20. iii , iv, i , ii
21. a) Self- help groups
22. HYV seeds were also called Miracle Seeds
OR
MS Swaminathan is known as the “Indian Father of Green
Revolution”
23.CHINA
24. (A) Labour force – Work force
25. 1921
26. (C)IV, III, I, II
27. (A)LPG has replaced LQP.
28.Labour Supply refers to supply of labour corresponding to
different wage rates. Supply of labour is measured in terms of
man days of work and is always related to wage rate. Labour
force refers to number of persons actually working or willing to
work.
OR
Structural Unemployment : This form of unemployment is largely defined
as unemployment that results from perceived value and skills that an individual
brings to a job against the needed, different skills required by an employer to
do the job correctly.

29. For economic development Pakistan followed


The different policies
(i) A variety of regulated
(ii) Protection of consumer
(iii) The introduction of
(iv) Public investment
(v) Nationalisation of capital
(vi) In 1980s, denatio
(vii) In 1988, economic reforms were initiated in the economy.

30. On-the-job-training is a source of human capital formation as


it increases the skill and efficiency of the workers and leads to an
increase in production and productivity. Usually, it is

Page 1 of 1
hypothesized that labour migration has a positive impact on
human capital, because money remittances from labour migrants
are spent on education of their household members, which raises
the overall level of education in a country.
31. a (i) Raising the rate of economic growth of India to about 8
per cent at par with other Asian countries like Singapore,
Malaysia, Hong Kong, South Korea etc.
(ii) Economic reforms is helping the country to attain growing
competitiveness in its industrial sector to face global competition.
(iii) New Economic Policy aims to reduce the extent of poverty
and inequality in the distribution of income and wealth.
(iv) The new policy has been taking steps to raise the efficiency
and profitability of public sector enterprises.

32. Disinvestment is when governments or organizations sell or


liquidate assets or subsidiaries. Disinvestments can take the form
of divestment or a reduction of capital expenditures
(CapEx). Disinvestment is carried out for a variety of reasons,
such as strategic, political, or environmental.
Import substitution is a strategy under trade policy that abolishes
the import of foreign products and encourages for the production
in the domestic market. The purpose of this policy is to change
the economic structure of the country by replacing foreign goods
with domestic goods.
until such time as it is can absorb investment more easily and also
trade its own products.
Outsourcing is the business practice of hiring a party outside a
company to perform services and create goods that traditionally
were performed in-house by the company's own employees and
staff.

Page 1 of 1
A quota is a government-imposed trade restriction that limits the
number or monetary value of goods that a country can import or
export during a particular period. Countries use quotas in
international trade to help regulate the volume of trade between
them and other countries.
33. Institutional credit for agriculture has gradually increased.
Cooperative Banks, Regional Rural Banks and Commercial
Banks are the main sources of agricultural credit. The flow of
agricultural credit since 2003-04 has consistently exceeded the
target. The four major sources of institutional credit are co-
operatives, commercial banks, regional rural
banks and government departments.

34. Poverty Alleviation Programmes aims to reduce the rate of


poverty in the country by providing proper access to food,
monetary help, and basic essentials to the households and
families belonging to the below the poverty line. As per the
Planning Commission
of India, the level of poverty in a country can be estimated based
on the consumer expenditure surveys that are conducted by the
National Sample Survey Office (NSSO) under the Ministry of
Statistics and Programme Implementation. The PAPs also were
not fully Effective, Neglect of agriculture, Urban Concentration
of Growth Process, Economic colonization, Spread of
Consumerism, Lopsided Growth Process, Cultural erosion are
some of the issues associated with PAPS.
OR
OR
Education plays a significant role in economic development as
follows: Education increases the accessibility of people to

Page 1 of 1
modern and scientific ideas. It increases the efficiency and ability
of people to absorb new technology. It creates awareness of the
available opportunities and mobility of labour
(i) It improves productivity and efficiency.
(ii) It improves mental horizon of the people by which they try to
improve their health status.
(iii) Common educated masses contribute in the development of
the country.
(iv) It makes them good citizen that helps in choosing an
intelligent government.
(v) It promotes science and technology by which people have a
scientific outlook.
(vi) It produces skilled and trained workers.
Vi In this way education helps in human capital formation.

Page 1 of 1
​MODEL EXAMINATION 2020 – 21
CLASS XII
TIME-3 hours HOME SCIENCE(064) M.M-70
GENERAL INSTRUCTIONS-
1.All questions are compulsory.
2.There are total 36 questions.
3.Question paper is divided into three sections-A, B and C.
4.Section A has question no.1to 14 (objective type questions) and are of 1 mark each.
5.Section B has question no. 15 to 21 (case study based multiple choice questions) and are of 1
mark each.
6.Section C has question no.22 to 27 of 2 marks each, question no.28 and 29 of 3 marks each,
question no.30 to 33 of 4 marks each and question no.34 to 36 of 5 marks each.
7.Internal choices are given in some questions.
8.Support your answers with suitable examples wherever required.

SECTION A (OBJECTIVE TYPE QUESTIONS)


S.No​. ​Multiple choice questions Marks

1. Identify the​ method used to process food as little as possible in order to retain 1
the quality of fresh food.
a) Preserved foods
b) Minimally processed foods
c) Medical foods
d) Formulated foods

OR
The intergovernmental body formed to protect the health of the consumers and
facilitate food and agriculture trade.
a) CAC
b) ISO
c)ISI
d)WTO
2. A person who takes risk for converting a novel idea in to reality. 1
a) Businessman
b) Entrepreneur
c) Partner
d) Starter

3. In 1810, _________________ developed the method of canning. 1


a) Louis Pasteur
b) Napoleon Bonaparte
c) Nicolas Appert

d)Harvey Levenstein

4. The department responsible for maintaining cleanliness in the kitchen and 1


to wash the pots and pans used.
a) Assistant Waiter
b) Dish Washer
c)Lobby Manager
d) Kitchen Stewarding
OR
You got a job in hotel and your responsibility is maintain cleanliness,
maintenance of guestroom, corridors, staircase and floor pantries You must
be appointed as_____________________
a) Floor supervisor
b) Room attendant
c) Desk control supervisor
d) Assistant housekeeper
5. Nationwide communication campaign implemented in cooperation with 1
Indian Railways for generating awareness about HIV/AIDS.
a) Red Ribbon Express
b) Red Rose Express
c) Ribbon Red Express
d) Ribbon Rose Express
6. Match the following- 1
A. High risk food i) Lycopene
B. Medical food ii)Biscuit
C. Formulated food iii)lactose free milk
D. Functional food iv) protein rich food
Pick the correct option-
a) A iii, B i, C iv, D ii
b) A iv, B iii, C ii, D i
c) A iv, B i, C ii, D iii
d) A iii, B iv, C ii, D i
7. The organism that exists in the intestinal tract of animals, raw milk and eggs 1
which cause food poisoning or infection.
a) Salmonella
b) Lactobacillus rhamnosus
c)Staphylococcus aureus.
d)Lactobacillus reuteri

8. The diet that are adjusted to meet the medical needs of patient. 1
a) Balanced diet
b) Regular diet
c) Modified diet
d) Liquid diet

Fill in the blanks​-


9. ____________ is used to meet the demand for an interactive satellite -based 1
distance education system in India.

10. Adjustment of human and machine is ____________. 1


11. Identify the craft and name the state in which it is practiced. 1

OR
Identify the painting and name the state where it is popular.

12. Sita has to buy a silk saree for a marriage function. Which standardized mark 1
she should check on the saree.
13. Predict two benefits of campaign. 1

14. As per the NCF (2005) write two basic objectives of ECCE. 1
OR
List any two substitute child care.

SECTION B (CASE STUDY BASED QUESTIONS)


​Malnutrition affects all age groups but is more common especially among
children. Many consequences of malnutrition may be hidden. In every
society even in developed and industrialised countries suffers from
insufficient intake of macro nutrients. A large proportion of children (and
adults) suffers from micronutrient deficiencies in varying degrees of
severity. The micronutrients of most concern are iron, vitamin A, iodine,
folic acid, B12, calcium, vitamin D.
Micro nutrient deficiencies are referred
as-
Q15. a) Goitre 1
b) Hidden Hunger
c) Protein energy malnutrition
d) Anemia

Which vitamin deficiency is associated with night blindness?


Q16. a) Vitamin C 1
b) Vitamin D
c) Vitamin B 12
d) Vitamin A

When the weight of the child is not adequate relative to height, this is
Q17. termed as- 1
a) Underweight
b) Stunting
c) Wasting
d) Protein energy malnutrition

_____________ is caused by insufficient intake of the macro nutrients.


Q18. 1
a) Goitre
b) Iron deficiency anemia
c) Protein Energy Malnutrition
d) Vitamin A deficiency
Design is a term commonly used to describe the appearance and appeal
of any article. Elements of Design are the tools of the Art. These are
colour, texture, and line, shape or form. Colour is all around us in many
forms. The identity of the product is most often attributed to colour.
Colour reflects the season, events and the spirit of people. Colour is an
important part of Fashion. Colour can be seen in fabrics in various design
forms. Designers carefully choose fabric colours to make a definite
statement. Certain basic colour schemes are used as guide for combining
colours. Colour scheme are best studied with reference to the colour
wheel.
Lightness or darkness of a colour is described in terms of ______
Q19. a) Value 1
b) Intensity
c)Hue
d)Complementary

Yellow-orange, red-orange, and yellow-green are examples of_______.


a) Intermediate or Tertiary colour
Q20. b) Secondary colour
c) Primary colour 1
d) Triadic colour

When black colour is added to any other colour it is called as_____________


of a colour.
Q21. a) Shade
b) Tone
c) Tint
b) Hue

1
SECTION C
22. What are the aims of Kasturba Gandhi Balika Vidyalaya (KGBV)? 2
OR
Illustrate with an example which shows the importance of ergonomics in
work place .

23. Enlist any 4 nutrition programme operating in our country. 2

24. Write any 4 important service offered by staff in a front office. 2


OR

Elaborate on the four stages of Guest cycle.

25. What are the two aspects of care and maintenance of fabrics? 2

26. What are the essential features of Development Communication? 2

27. What are the services provided by ICDS to meet the needs of vulnerable 2
children?
28. Give any two ways each to create balance and rhythm in a frock of 5year old 3
girl.
29. Why are youth vulnerable? Give any two reasons to support your answer. 3
Name any two programmes operating in our country for youth.
OR
Describe any two programmes operating in our country for elderly and
describe about two skills need to develop to pursue a career in management
of institutions and programmes for children /youth/elderly.

30. Enumerate any four important knowledge and primary requirement needed 4
when preparing for a career in the field of care and maintenance of fabrics.
OR
How are the laundry in hospitals different from a hotel?

31. A food hazard can enter/come into the food at any stage of the food chain, 4
therefore, adequate control throughout the food chain is essential. Explain
three ways with two features of each to ensure food safety and quality. Also
state two importance of FSSA,2006.
32. Write any four principles given by NCF (2005) to be kept in mind while open 4
up an institution for children. List four skills an ECCE worker must have.

33. Elaborate on four knowledge and skills required by a professional, in the stage 4
where food as a material and food product development.
34. Discuss the various strategies that can be adopted to combat public 5
nutritional problems. Compare two advantages and disadvantages of each.

35. One major problem of consumer is lack of information of available products. 5


Explain four each responsibilities and rights of consumer in this context.
OR
Your friend wants to pursue career in consumer related field. Guide him/her
about five each skills and career opportunities of this field.

36. What do you understand by the term fashion merchandising? Describe 5


various levels of merchandising?
Answer key
1.b) Minimally processed foods
or
a) CAC
2. b) Entrepreneur
3.​ c)​ Nicolas Appert
4. d)​ ​Kitchen Stewarding
Or
a) Floor supervisor
5. a) Red Ribbon Express

6.a) A iv, B iii, C ii, D i


7. a)Salmonella
8.c) Modified diet
9.EDUSAT
10.Ergonomics
11.Bamboo craft of Assam
Or
Warli Painting of Maharashtra
12. Silk mark
13.a)Prepare for school (Any 2)
b)Holistic development of child
c)Providing support services for women and​ ​children
14.Creeche , Daycare
15. b)Hidden Hunger
16.​d) Vitamin A
17.c) Wasting
18. c)Protein Energy Mal nutrition
19. Value
20. Intermediate or Tertiary colours
21. Shade
22. to bring never enrolled and drop out student from rural
and urban areas.
OR
For a person who does office work sitting on a chair that
does not conform to ergonomic safety may cause back pai
which will effect the work and productivity.
23.a) ICDS (Any 4)
b) Nutrient Deficiency Control programme.
​c) Food Supplementation programmes like mid-day
meal programme.
d) Food security programmes.
e) Self employment and wage employment schemes.
24)a)Welcoming guest (Any 4)
b)Registering guest and allocating room
c)Preparing and settling bills
d)Porter services
e)Issuing room key to guests etc .
OR
a) Pre arrival stage
b) Arrival stage
c)Occupancy
d) Departure
25)a) keeping the material free of physical damage and
rectifying any damage that may have occurred during its
use.
b) Retaining or refreshing appearance in terms of removal
of dirt and stain and retaining the textural and visual
characteristics.
26) It is oriented to socio economic development and
happiness of the people and community at large.
b) It aims at giving information and educating the
community.
c)It is based on the audience characteristics and their
environment.
d) It combines suitable mass media and interpersonal
communication channel for greater impact.
27)a) Non formal pre school education to children aged 3-6
years
b) Immunization
c) Health check up
d) Referral services
e) Nutrition, health and hygiene education to mothers.
f) Supplementary Nutrition
28)Rhythm- repetition, radiation
Balance -formal ,informal
29) Peer pressure, unable to get support from family,
substance abuse, lack of knowledge and sexual and
reproductive health, livelihood and marriage followed by
raising a family.
Programmes operating for youth- The service scheme,
Nehru Yuva Kendra Sangathan, Scouts and Guides,
Promotion of Adventure. (Any Two)
OR
Programmes operating for Elderly-National Old age pension
scheme, Old age home, Respite care homes and continuous
care home for elders (AnyTwo)
Skills- People skills and administrative skills.
30.Primary requirement- knowledge of material, knowledge
of process involved, knowledge of chemical and reagents
used, working knowledge of machinery and its function
Knowledge – Proce of laundry ,stain removal,role of water ,
finishing treatments ,folding (Any 4)
OR
In Hospitals – Importance to hygiene, cleanliness,
disinfection, b) Ironing and pressing may not done on
perfection. c)repair and mending may not form apart of
service required d) quantum of work ,especially for bed
linen is more for hospitals compared to hotel.
In Hotels- aesthetic and final finish is more important, b)
final finishing of laundered goods are emphasised ,c) have
to care of guests personal laundry when required d)
quantum of work ,especially for bed linen is much lesser in
hotels.
31. Food safety and quality can be ensured through-a)
Good manufacturing practices (GMP) b) Good handling
practices c) Hazard Analysis Critical Control Point (HACCP)
Two importance of FSSA 2006 are- 1) To regulate storage
manufacture, distribution, sale and storage of food. 2)To
ensure availability of safe and wholesome food for human
consumption
32. Principles- Play as the basis of learning b) Art as the
basis of learning c) Mix of formal and informal interactions
d) Use of local materials ,arts and knowledge
Skills – a)An interest in children and their development b)
Capacity and motivation for interacting with children c)
Enthusiasm for activities d) Willingness and interest in
answering to children queries.
33.Food as a material- a) Seasonal availability of food stuffs
b)Nature and properties o food c) Nutritional content and
its analysis d) Food hygiene and food safety
Food product development – a) Knowledge of product
specification and testing it b) Assessment by sensory
methods c) packaging of marketable products d) Hazard
Analysis Critical Control Point (HACCP)
34. Strategies -a) Diet or food-based strategies b) Nutrient
based approach or medicinal approach
Diet or food-based strategies-Advantages : (Any Two) cost
effective , long term benefits ,no risk of overdose and
toxicity. Disadvantage- (Any two)time consuming, requires
change in eating behaviour, requires research participation
of food industry.
Nutrient based approach or medicinal approach -
Advantages: (Any Two) short term strategies, timely
,sustainability . Disadvantage: expensive ,narrow scope of
coverage.
35.Rights – Right to be informed ,Right to seek redressal,
Right to be heard, Right to choose (Any 4)
Responsibilities-( Any 4)(a)consumer should have
responsibility towards regularly updating the knowledge on
various laws and legislative provisions made by the
government. (b) consumer should be honest in all their
dealings and must pay for all their purchases. (c) while
purchasing ,should read all information given on label and
brochure. (d) to assured of quality he should buy products
with standardisation mark.
OR
Skills required – (Any 5) (a) knowledge about consumer
protection mechanism and redressal agencies (b) good
communication and interpersonal skills (c) empathetic and
understanding attitude (d) good listener (e) willingness to
help fellow customers (f) creative in developing
programmes
Career opportunities- (Any 5)(a)work in government
organization (b) work in voluntary consumer organizations
(c) work with market research organization (d) work with
consumer division of corporate house that deal consumer
complaints and suggestion. (e) work in National consumer
help line for consumer counselling.
36.Fashion merchandising – refers to the planning required
to have the right merchandise at the right time, at the right
place, at the right price and with right sales promotion, If all
these conditions are planned , one can achieve maximum
profit.
Levels of merchandising – (a)retail organisation
merchandising (b) buying agency merchandising (c) export
house merchandising.
________________________________________
MODEL EXAMINATION 2020 – 21
INFORMATICS PRACTICES (065)
Class XII

Max Marks: 70 Time: 3 hrs


General Instructions:

1. This question paper contains two parts A and B. Each part is compulsory.
2. Both Part A and Part B have choices.
3. Part-A has 2 sections:
a. Section – I is short answer questions, to be answered in one word or one line.
b. Section – II has two case studies questions. Each case
study has 4 case-based subparts. An examinee is to
attempt any 4 out of the 5 subparts.
4. Part - B is Descriptive Paper.
5. Part- B has three sections
a. Section-I is short answer questions of 2 marks each in which
two questions have internal options.
b. Section-II is long answer questions of 3 marks each in which
two questions have internal options.
c. Section-III is very long answer questions of 5 marks each in which one question
has

PART A
Section - I
Attempt any 15 questions from questions 1 to 21(1 mark each)

1. State whether True or False :


i. E –waste is hazardous if not handled carefully. _________
ii. Free software is the same as freeware. ______________
2. Fill in the blanks :
The command used to display graph is ______________
a. plt.show()
b.plt.plot()
c.plt.xlabel()
d.plt.title()
3. Write the output of the following SQL command.
select round(123.58,1);
a. ​123.6
b. ​124
c. ​123.5
d. ​123
4. Given a Pandas series called Sequences, the command which will display the last 3 rows
is .
a. print(Sequences.tail(3))
b. print(Sequences.Tail(3))
c. print(Sequences.tails(3)
d. print(Sequences.tail())
5. Given the following Series S1:

S1
A 10
B 20
C 30
D 40

Write the command to double the value in series and store in another series named as 2

6. Axes of a plot can be labelled using​ ​__________ and ____________ functions.


7. Web page constitutes the ​ ​_________________
8. Which method is used to access the vertical subset of a dataframe.?
i. Iterrows()
ii. Iteritems()
iii. Itertuples()
9. Which of the following is not a network device :
i. Repeater, hub, TCP, switch
10. Website is collection of .
11. The count() function in MySql is an example of .
i. Math function
ii. Text function
iii. Date Function
iv. Aggregate Function
12. Wi-Fi, infrared, and Bluetooth is an example network.
13. The command to install pandas is
14. I can share hardware
I can share software
I facilitate you
Who am i?
15. Which amongst the following is not used to surf on the network?
i. Internet explorer
ii. Firefox
iii. Avast
iv. Edge
16. Tricking people through authentic –looking emails or websites is called__________
17. Unsolicited commercial email is known as ____________
18. The​ ​command can be used to arrange data in some order in a table in SQL
19. Write the SQL command that will display the year from date ’2020-09-30’
20. _____________ is an example of a network of networks .
21. Online personal account, personal website is an example of____________
Section -II
Both the case study based questions (22 & 23 ) are compulsory. Attempt any four sub parts
from each question. Each sub question carries 1 mark.

22. Consider the following Data Frame ​df ​and answer ​any four out of five from ​(i) to (v)
Name Rollno English Hindi Maths Ssc Science
0 Anita 1 55 45 87 67 70
1 Sunita 2 67 65 90 87 56
2 Radha 3 78 76 98 90 78
3 Anis 4 98 87 78 45 87
4 Kaushal 5 45 58 90 69 98

a. Write statement to display name and rollno only.


b. The teacher needs to know the marks scored by the student with roll number 4. Help her to
identify the correct set of statement/s from the given options :

a. df1=df[df[‘rollno’]==4]
print(df1)
b. df1=df[rollno==4] print(df1)
c. df1=df[df.rollno=4] print(df1)
d. df1=df[df.rollno==4] print(df1)

c. Which of the following statement/s will delete column ‘Total’ from the dataframe?
i.df.pop(‘Total’) ii. del df[‘Total’] iii.del (df.Total) iv.df.del(‘Total’)
Choose the correct option:
a. both (i) and (ii)
b. only (ii) 
c. (i), (ii) and (iii) 
d. (i), (ii) and (iv)
d. Which of the following command will display the column labels of the DataFrame?
a. print(df.columns())
b. print(df.column())
c. (df.column)
d. print(df.columns)
e. Write command to add total column to dataframe named ‘total’ which stores the total of
all the subjects mentioned in the df.

23. Consider the table TRAVEL given below:

NO NAME TDATE KM CODE NOP


101 Janish Kin 2015–11–13 200 101 32
103 Vedika Sahai 2016–04–21 100 103 45
105 Tarun Ram 2016–03–23 350 102 42
102 John Fen 2016–02–13 90 102 40
107 Ahmed Khan 2015–01–10 75 104 2
104 Raveena 2016–05–28 80 105 4
i. Write query to give the output as:

NO NAME TDATE KM
101 Janish 2015-11- 200
Kin 13
105 Tarun 2016-03- 350
Ram 23

(A) Select * from travel where km>200;


(B) Select * from travel where km>=200;
(C) Select no,name, tdate,km from travel where km>=200;
(D) Select no,name, tdate,km from travel where km between 200 and 350;

ii. Write a query to display the maximum km from the travel table.
iii. Akhil has given the following command to arrange the data in ascending order of
date .
Select * from travel where order by date;
but he is not getting the desired result. Help him by writing the correct command.
a) Select * from travel where order by date;
b) Select * from travel order by date;
c) Select * from travel in ascending order;
d) Select date from travel order by date;
iv. Write the query to count the number of codes in each code type from the travel
table.?
i. select count(code) from travel ;
ii select code,count(code) from travel group by code;
iii. select code,count( distinct code) from travel;
iv. select code,count( distinct code) from travel group by code;

Choose the correct option:


a. Both (ii) and (iii)
b.Both (ii) and (iv) 
c.Both (i) and (iii) 
d.Only (ii)
v. Help Ritesh to write the command to display the name of the traveler whose travel
date is in the year 2016?
a. Select name,tdate from travel where year(tdate)=2016 ;
b. Select name,tdate from travel where tdate=2016;
c. Select name,tdate from travel where year(tdate)= =2016;
d. Select name,max(tdate) from travel ;

Part – B
Section – I(2 marks each)

24. Consider a given Series , S1:


100 500
101 500
102 500
103 500
104 500
Write a program in Python Pandas to create the series.
25. State any two differences between where and having clause in SQL. Give example
of each

OR
What is the difference between the order by and group by clause when used along with the
select ​statement. Explain with an example.
26. How are NULL values treated by aggregate functions in SQL. Give Example
27. Consider the following Series object, s
Apple 10
Mango 20
Banana 30
Orange 40
i.Write the command which will display only apples.
ii.Write the command to increase the price of all fruits by 10.

28. Kaushal writes the following commands with respect to a table student having fields,
name,roll no, marks and records of 5 students.
Command1 : Select count(marks) from student;
He gets the output as 4 for the command .
Explain the output with justification.

29. Consider the following SQL string: “Informatics”


a. Write commands to display “form”
b. Display the string in capital letters.

OR

Considering the same string “Informatics” Write SQL commands to display:


a) the position of the substring ‘matics’ in the string .
b) the last 4 letters of the string
30. Consider the following Data Frame, class1

Name Roll no English Hindi Maths Ssc Science

0 Anita 1 55 45 87 67 70

1 Sunita 2 67 65 90 87 56

2 Radha 3 78 76 98 90 78

3 Anis 4 98 87 78 45 87

4 Kaushal 5 45 58 90 69 98

Write code to display the data frame ‘class1’ row-wise.


31. Expand the following terms related to Computer Networks:
i. URL
ii. ISP 
iii. FTP 
iv. HTTP
32. List any two health hazards related to excessive use of Technology.
33. Following acts are considered :
● Posting any kind of humiliating content about the victim.
● Hacking the victim’s account.
● Sending or posting vulgar messages online.
● Threatening to commit acts of violence.
● Stalking by means of calls, messages, etc.
● Threats of child pornography.
What is the correct word for the above statements mentioned? What action an individual
must take if the above mentioned things happened ?

Section -II(3 marks each)

34. Consider two objects x and y. x is a list whereas y is a Series. Both have values 20,
40,90, 110.
What will be the output of the following statement considering that the above objects
have been created already
a. print (x*y)
This statement correct or not ? What will be the output. Give reason for your answer.
35. What do you mean by cyber security? How one can protect His/Her data.
OR

What do you understand by Net Etiquettes? Explain any two such etiquettes.
36. Consider the following graph . Write the code in python to draw a bar graph to show the
percentage of following students.
OR

Write code in python to show the data using histogram.


37. A relation product is given:
V_no Type Company Price Qty
AW125 Wagon Maruti 250000 25
J0083 Jeep Mahindra 4000000 15
S9090 SUV Mitsubishi 2500000 18
M0892 Mini Van Datsun 1500000 26
W9760 SUV Maruti 2500000 18
R2409 Mini Van Mahindra 350000 15
Write SQLcommands to:
i. Display the MINIMUM price of each type of vehicle having quantity
more than 20.
ii. Display the Count of the type of vehicles manufactured by each company.
iii. Display the type and total price((price*qty) of all the types of vehicles in
ascending order of type.
Section -III(5 Marks each)

38. Write a program in Python Pandas to create the following DataFrame Bank from a
Dictionary:
C_NO Name Balance
101 Sunita 9000
102 Manish Sharma 6500
103 Ajay Goel 70000
104 Aditya Thakur 800000
Perform the following operations on the DataFrame :
1)Add a new column interest in dataframe (interest rate is 5%)
2)Display the customers having balances more than 50000.

39. Write the SQL functions which will perform the following operations:
i. To display the day of the month of the current date .
ii. To display the length of string , “ hello “.
iii. To display the name of the day eg, Friday or Sunday from your date of birth, dob.
iv. To display the string in upper case ‘I love india’
v. To compute the 2​3​ using sql function.

OR
Consider a table SALES with the following data:

SNO SNAME SALARY BONUS DATE OF JOIN

A01 Beena Mehta 30000 45.23 29-10-2019

A02 K. L. Sahay 50000 25.34 13-03-2018

B03 Nisha 30000 35.00 18-03-2017


Thakkar

B04 Leela Yadav 80000 NULL 31-12-2018

C05 Gautam Gola 20000 NULL 23-01-1989

C06 Trapti Garg 70000 12.37 15-06-1987

D07 Neena 50000 27.89 18-03-1999


Sharma

Write SQL queries using SQL functions to perform the following operations:
i. Display salesman name and salary of salesman assuming the salary in the table is
for one month and display the salary for the complete year. .
ii. Display the first four characters of salesman names.
iii.
Display the name of the salesman in upper case.
iv.Display the year name for the date of join of salesman
v.Display the name of the weekday for the date of join of salesman whose year is
2019
40. KVS organization is setting up the network between the different wings. There are 4
Wings named as Science(S), Junior(J), Admin(A) and Hostel(H).
Distance between various wings in meters.
A TO S 100
A TO J 200
A TO H 400
S TO J 300
S TO H 100
J TO H 450
Number of computers in each wing
Senior 200
Admin 10
Junior 50
Hostel 20

i. Suggest a suitable Topology for networking the computer of all wings.


ii. Name the wing where the server is to be installed. Justify your answer
iii. Suggest the placement of Hub/Switch/Repeater in the network
iv. Mention an economic technology to provide internet accessibility to all wings
v. Suggest the type of Network LAN/MAN/WAN.
INFORMATICS PRACTICES (065)
Class XII(IP)
ANSWER KEY

SECTION A

1. i. True
ii.False
2. plt.show()
3. 123.6
4. print(Sequences.tail(3))
5. S2=s1*2
6. xlabel and ylabel
7. website
8. Iteritems()
9. TCP
10. Web pages
11. Aggregate function
12. PAN or LAN both are correct
13. Pip install pandas
14. Server
15. Avast
16. Hacking
17. Spam
18. Order by
19. Select year( ’2020-09-30’ )
20. Internet
21. Digital property
22. i. print(df[["Name","Rollno"]] )
ii. a) df1=df*df*‘rollno’+==4+
print(df1)
d) df1=df[df.rollno==4]
print(df1)
iii. both (i) and (ii)
iv. print(df.columns)
v. df["Total"]=df["English"]+df["Hindi"]+df["Maths"]+df["Ssc"]+df["Scie nce"]

23. i. (c) and (d) both are correct


ii. Select max(km) from travel;
iii. B
iv. only (ii)
v. Select name,tdate from travel where year(tdate)=2016
SECTION B

24. import pandas as pd1

s =pd1.Series(500,index=[100,101,102,103,104])

print(s)

25. (i) Where clause is used in sql when ever any condition is to be used for example select *
from std where rollno>10;
(ii) Having is used only along with group
(iii) Select * from std group by class having rollno>10;

OR

The order by clause is used to show the contents of a table/relation in a sorted manner with
respect to the column mentioned after the order by clause. The contents of the column can
be arranged in ascending or descending order.
The group by clause is used to group rows in a given column and then apply an aggregate
function eg max(), min() etc on the entire group.
26. null values are ignored in SQL if out of two rows 12, null one has null value the output of
sum would be 12 same in case of any aggregate function
27. i. print(s['Apple'])
ii. s=s+10
print(s)
28. This is because the column marks contains a NULL value and the aggregate functions do not
take into account NULL values.
Thus Command1 returns the total number of records in the table whereas Command2
returns the total number of non NULL values in the column marks
29. A. select substr("Informatics ", 3,4);
B. select upper(“Informatics”) OR

OR

a. select instr ("Informatics ", 'matics'));


b. select right ("Informatics ", 4);
30. for row_index,row in class1.iterrows():

print(row_index,row)

31. URL : Uniform Resource Locator a.


ISP Internet Service Provider
FTP File Transfer Protocol
HTTP :Hyper Text Transfer Protocol
32. The continuous use of devices like smartphones, computer desktop, laptops, head phones
etc cause a lot of health hazards if not addressed. These are:
a. Impact on bones and joints: wrong posture or long hours of sitting in an
uncomfortable position can cause muscle or bone injury.
b. Impact on hearing: using headphones or earphones for a prolonged time and on
high volume can cause hearing problems and in severe cases hearing impairments
c. Impact on eyes: This is the most common form of health hazard as prolonged hours
of screen time can lead to extreme strain in the eyes.
d. Sleep problem: Bright light from computer devices block a hormone called
melatonin which helps us sleep. Thus we can experience sleep disorders leading to
short sleep cycles.
33. We call this type of activity as cyber bullying
It must be reported to cyber cell.
34. The statement is correct
b.will give the output as:

0 10000

1 40000

2 90000

3 160000

dtype: int32

list is also like a series so they can be multiplied.

35. Cyber security is the combination of best processes and practices to ensure the security of
networks, computers, programs, data and information from attack, damage or unauthorized
access.

OR

Net Ettiquets refers to the proper manners and behaviour we need to exhibit while being
online.

These include :

1. No copyright violation: we should not use copyrighted materials without the permission of
the creator or owner. We should give proper credit to owners/creators of open source
content when using them.

2. Avoid cyber bullying: Avoid any insulting, degrading or intimidating online behaviour like
repeated posting of rumours, giving threats online, posting the victim’s personal
information, or comments aimed to publicly ridicule a victim.

36. import numpy as np


import matplotlib.pyplot as plt
year=[2014,2015,2016,2017,2018]
jnvpasspercentage=[90,92,94,95,97]
kvpasspercentage=[89,91,93,95,98]
plt.plot(year,jnvpasspercentage)
plt.plot(year,kvpasspercentage)
plt.xlabel('Year')
plt.ylabel('Passpercentage')
plt.title('JNVKVPASS%till2018')
plt.legend()
plt.show()

OR

import numpy as np
import matplotlib.pyplot as plt
plt.hist([1,11,21,31,41,51],bins=[0,10,20,30,40,50,60],weights=[10, 1,0,33,6,8])
plt.show()
37. a. select Type, avg(Price) from Vehicle group by Type having Qty>20;
b. select Company, count(Type) from Vehicle group by Company;
c. Select Type, Price*qty from Vehicle order by Type;
38. import pandas as pd
d1={'C_NO':[1,2,3,4], 'Name':["Sunita","Manish Sharma","Ajay Goel","Aditya Thakur"],
'Balance':[9000,6500,70000,800000], }
df=pd.DataFrame(d1)
print(df)
df['Interest'] = 0.05*df['Balance']
print(df[df['Balance']>50000])
39. (i) Select dayofmonth(date(now()));
(ii) Select len(“hello “) ;
(iii) Select dayname(‘date of birth any’);
(iv) Select upper (‘I love india’);
(v) Select mod(n1,n2);

OR

a) Select sname, salary*12 from Sales;


b) Select left(sname,4) from Sales;
c) Select upper(sname) from Sales;
d) Select year(DateofJoin) from Sales;
e) Select weekday(DateofJoin) from Sales where year(dateofjoin)=2019;
40. i) Star tolology
ii) Senior wing having maximum number of computers
iii) Switch in every wing and repeater between the wings.
(iv) Broadband
(v) LAN
MODEL EXAMINATION 2020 – 21
CLASS – XII
SUBJECT – Mathematics

Time Allowed : 3 HOURS MM: 80

General Instructions:
1. This question paper contains two parts A and B. Each part is compulsory. Part A carries 24 marks and Part B
carries 56 marks
2. Part-A has Objective Type Questions and Part -B has Descriptive Type Questions
3. Both Part A and Part B have choices.
Part – A:
1. It consists of two sections- I and II.
2. Section I comprises of 16 very short answers type questions.
3. Section II contains 2 case studies. Each case study comprises of 5 case-based MCQs. An examinee is to attempt
any 4 out of 5 MCQs.

Part – B:
1. It consists of three sections- III, IV and V.
2. Section III comprises of 10 questions of 2 marks each.
3. Section IV comprises of 7 questions of 3 marks each.
4. Section V comprises of 3 questions of 5 marks each.
5. Internal choice is provided in 3 questions of Section –III, 2 questions of Section IV and 3 questions of Section-V.
You have to attempt only one of the alternatives in all such questions.

Part A
SECTION- I
All questions are compulsory. In case of Internal choices attempt any one.
Question Nos. 1 to 16 are very short answer type.

1. Show that f:R R defined as f(x) = Sinx is neither one-one nor onto.
OR
If R is an equivalence relation defined in Set A= {1,2,3.....10} as R= {(a b):|a-b| is a multiple of 3}
write the equivalence class {1}
2. The relation R in the set of natural numbers N defined as R={(x,y)€ NxN: x+y=5}. Is relation R a
function?
3. Let f: R R be a function defined as f(x) =x2 + 3. Show that function is not surjective.
OR
How many relations are possible in Set A such that n(A)=2 ?
2 0 𝑇
4. Find the order of matrix [ −1 ] [ 2 ]
4 5

5. Let P and Q are two different matrices of order nX3 and nxp respectively. Then find the order of
matrix QTp.

0 2𝑏 −2
6. Matrix A= [ 3 1 3 ] is given to be symmetric, find values of a and b.
3𝑎 3 −1

7. Evaluate
OR
𝑎 1 𝜋
If ∫
0 1+4𝑥 2
. 𝑑𝑥 = then find the value of ‘a’ .
8

8. Find the area bounded by the line y=2x, the y-axis and the line y=3.
1
𝑑2 𝑦 𝑑𝑦 2 4
9. Find the order and degree of the differential equation = {𝑦 + (𝑑𝑥 ) }
𝑑𝑥 2
OR
𝑑𝑦
What is the integrating factor of 𝑥 − 𝑦 = 𝑥 4 − 3𝑥
𝑑𝑥
10. Evaluate 2𝑖̂ Χ (𝑗̂ Χ 𝑘̂) + 𝑗̂. (𝑖̂ Χ 𝑘̂ ) − 𝑘.
̂ (𝑖̂ Χ 𝑖̂)
11. Evaluate |2𝑏⃗ Χ a⃗| , if it is given that 𝑎 = 2 𝑖̂ -𝑗̂ + 3 𝑘̂ and 𝑏⃗ = 3𝑗̂ + 𝑘̂
12. Find the y- intercept cut by the plane 4x-3y+2z-9 = 0
13. Find the distance between parallel planes 𝑟 .(2𝑖̂ - 𝑗̂ + 3 𝑘̂ ) = 4 and 𝑟 .(6𝑖̂ -3 𝑗̂ + 9 𝑘̂ )+13 = 0
14. Write the vector equation of the following line:
𝑥−5 𝑦+4 6−𝑧
= =
3 7 2
15. Three dice are thrown at the same time, find the probability of getting three two’s if it is known
that sum of the numbers on three dice is six.
1 1
16. Probability of solving specific problem independently by A and B are and respectively. If both
2 3
try to solve the problem independently, find the probability that the problem is solved.
SECTION – II
Both the case study based questions are compulsory. Attempt any 4 sub parts from each
question. Each sub-part carries 1 mark.

17. These days competitive examinations are online, a student has to go to a particular place to
give examination at a given time. For this company has to make perfect arrangements and
students are expected to be well prepared. But as a human nature sometimes a student
guesses or copies or knows the answer to MCQ with four choices each. With the given
background answer the following.
1
i. If the possibility that a student makes a guess is and that he copies the
3
1
answer is . What is the probability that he knows the answer.
6
1 5 1
a. 1 b. c. d.
2 6 3
ii. If answer is correct, what is the probability that he guesses it?
1 1 1 1
a. b. c. d.
4 2 8 3
iii. What is the conditional probability that his answer is correct and he knew it?
1 1
a. b.0 c. 1 d.
2 4
iv. What is the probability of answering a question correctly assuming the probability
1
that he copied it is ?
8
11 1 29 5
a. b. c. d.
8 5 48 29
v. What is the probability that he copied it given his answer is correct?
24 1 4 6
a. b. c. d.
29 29 29 29
18. The government of a state which has mostly hilly area decided to have adventurous playground
on the top of hill having plane area and space for 10000 persons to sit at a time. After survey it
was decided to have rectangular playground with a semicircular parking at one end of
playground only as space is less. The total perimeter of the field is measured as 1000m as shown.
Based on the above information answer the following.
i. Looking at the figure(plan) the relation between x and y is
a. X+2y+πy= 100 c. 2X+2y+πy= 1000
b. X+2y+πy= 500 d. X+y+πy= 1000
ii. Area of sports ground in terms of x is
2 2
a. (1000x-2x2)m2 c. (500x-2x2)m2
𝜋+2 𝜋+2
1 1
b. (1000x-2x2)m2 d. (500x-2x2)m2
𝜋 𝜋
iii. The maximum area of sports ground is for x equal to
a. 500 m c. 100 m
b. 50 m d. 250m
iv. The government wants to maximise the area including parking area for this to
happen, value of y is

1000 500
a. m c. m
4+𝜋 4+𝜋
2000 750
b. m d. m
4+𝜋 4+𝜋

v. What is the maximum area of the sports field alone?


9000 25000
a. m2 c. m2
2+𝜋 2+𝜋
16000 100000
b. m2 d. m2
2+𝜋 2+𝜋

PART B
SECTION-III
All questions are compulsory. In case of internal choices attempt any one.

1 1
19. Find the value of cos( cos-1 ).
2 8
1 −1
20. If matrix A= [ ] and A2 =kA, then find the value of k
−1 1
OR
1 2 3 −7 −8 −9
Find the matrix X so tat X[ ]=[ ]
4 5 6 2 4 6

21. Find the value of k so that the function f defined by f(x)= {𝑘𝑥𝑐𝑜𝑠𝑥,
+ 1, 𝑖𝑓 𝑥 ≤ 𝜋
𝑖𝑓𝑥 > 𝜋
is continuous at

x=𝜋
2⁄ 2⁄
22. Find the equation of the tangent to the curve 𝑥 3 +𝑦 3 =1 at (1,1).
sin2 x
23. Evaluate ∫ 𝑐𝑜𝑠4 𝑥 . 𝑑𝑥
OR
1⁄ 𝑑𝑥
∫−1⁄2 1⁄
2 (1−𝑥 2 ) 2
24. Find the area (in square units) enclosed by the curve 𝑥 2 y =36, the x –axis and the lines x=6 and
x=9.
25. Solve: xy dy =(y+5)dx, given that y(5)=0.
26. If |𝑎| = 10, |𝑏⃗| = 2, and 𝑎 . 𝑏⃗ = 12, then find the value of |𝑎 Χ 𝑏⃗|
27. Find the vector equation of the line passing through the point A(1,2,-1) and parallel to the line
5x-25=14-7y=35z.
28. A random variable x has the following probability distribution.
x 0 1 2 3 4 5 6 7
p(x) 0 k 2k 2k 3k k2 2k2 7k2+k
Determine k.
OR
If p (A) = 3⁄5 and p(B) = 1⁄5, find p(A ∩ B) if A and B are independent events.

SECTION IV
Question number 29 to 35 carry 3 marks each.
29. Show that the relation R in the set of real numbers defined as R={(a, b):a≤ b2 } is neither
reflective nor symmetric nor transitive.
𝑑𝑦
30. Find ⁄𝑑𝑥 of 𝑦 𝑥 =𝑥 𝑦
𝑑𝑦
31. If x=a sin3t, y= a cos3t, then find ( ) 3𝜋
𝑑𝑥 𝑡=
4
OR
−1
If y=(tan 𝑥)2,Show that (𝑥 2 + 1)2 𝑦2 +2x(𝑥 2 + 1)𝑦1= 2

1
32. Find the interval(s) in which the function f(x)=x3 + 𝑥 3 , x≠0 is
i. Strictly increasing
ii. Strictly decreasing.
3𝑥−1
33. Find ∫ (𝑥−1)(𝑥−2)(𝑥−3) .dx
34. Find the area of the region bounded by y=√𝑥 and y=x.
OR
Find the area of the region bounded by the curve y2=4x and x2=4y.
𝑑𝑦
35. Solve: (1 + 𝑥 2 ) 𝑑𝑥 + 2𝑥𝑦 = 4𝑥 2
SECTION V

Question number 36 to 38 carry 5 marks each.

1 −1 0 2 2 −4
36. Given A=[2 3 4] and B=[−4 2 −4] verify that BA=6I, how we can use the result to find
0 1 2 2 −1 5
the values of x,y,z from given equations.
X-y=3, 2x+3y+4z= 17, y+2z=17

OR

3 1 2
If A=[3 2 −3] find A-1. Hence solve the system of equations.
2 0 −1
3x+3y+2z=1
X+2y=4
2x-3y-z=5
37. Find the shortest distance between the lines.
𝑟 = (1 + 𝜆)𝑖̂ + (2 − 𝜆)𝑗̂ +(𝜆 + 1)𝑘̂ ;
𝑟 = (2𝑖̂ − 𝑗̂ − 𝑘̂) + µ (2 𝑖̂ + 𝑗̂ + 2𝑘̂)
OR
Find the distance of the point (2,12,5) from the point of intersection of the line
𝑟 =2𝑖̂ − 4𝑗̂ − 2𝑘̂ +𝜆 (3 𝑖̂ + 4𝑗̂ + 2𝑘̂) and the plane 𝑟.(𝑖̂ − 2𝑗̂ − 𝑘̂ ) =0.

38. Solve the following linear programming problem graphically.


Maximise z=20x+40y
Subject to constraints
1.5+3y≤42
X+2y ≤ 28 and
3x+2y ≤ 24, x≥ 0, y≥ 0.
OR
𝜋⁄
Evaluate ∫0 4 𝑙𝑜𝑔(1 + 𝑡𝑎𝑛𝑥)𝑑𝑥 .
Model Question Paper 2020-21

Class XII

Biology
Time - 3 Hours Maximum marks -70

General Instructions:
(i) All questions are compulsory.
(ii) The question paper has four sections: Section A, Section B, Section C and Section D. There are 33
questions in the question paper.
(iii) Section–A has 14 questions of 1 mark each and 02 case-based questions. Section–B has 9 questions
of 2 marks each. Section–C has 5 questions of 3 marks each and Section–D has 3 questions of 5 marks
each.
(iv) There is no overall choice. However, internal choices have been provided in some questions. A
student has to attempt only one of the alternatives in such questions.
(v) Wherever necessary, neat and properly labeled diagrams should be drawn.

SECTION A

Questions Marks

1 Double fertilization is the characteristic feature of angiosperms. One of the 1


events is triple fusion.Name the other event.

2 Name the phenomenon in which plants produce seedless fruits without 1


fertilization .

3 Why are pollen grains preserved as fossils ? 1

4 The presence of HCG in urine is the indication of pregnancy. Which is the 1


source of HCG ?

5 In man, four types of blood groups are determined by three alleles of a gene. 1
Name the type of inheritance involved in blood groups .

6 Define a test cross. 1


7 Why DNA is considered as a better genetic material than RNA ? 1

8 Name the two genes that are present in Bt toxins that control cotton bollworms. 1

9 Name the pathogen from which T​1​ plasmid is taken to modify as cloning 1
vectors.

10 Define Allen’s rule. 1

11 Assertion​ : The whole chromatin is not functional.  1


Reason​ : Heterochromatin does not take part in transcription. 
a . Both Assertion and Reason are true and Reason is the correct explanation
of Assertion.
b. Both Assertion and Reason are true but Reason is not the correct
explanation of Assertion.
c. Assertion is true but Reason is false.
d . Assertion is false but Reason is true.
e. Both Assertion and Reason are false.

12 Assertion​ : DNA ligase plays an important role in rDNA technology  1


​Reason​ : DNA ligase acts as molecular scissors   
a . Both Assertion and Reason are true and Reason is the correct explanation
of Assertion.
b. Both Assertion and Reason are true but Reason is not the correct
explanation of Assertion.
c. Assertion is true but Reason is false.
d . Assertion is false but Reason is true.
e. Both Assertion and Reason are false.

13 Assertion​ :​Sacred groves are examples of Ex-situ conservation.  1


Reason​ : In Meghalaya ,the sacred groves are the last refuges for a large 
number of rare and threatened plants .  
a . Both Assertion and Reason are true and Reason is the correct explanation
of Assertion.
b. Both Assertion and Reason are true but Reason is not the correct
explanation of Assertion.
c. Assertion is true but Reason is false.
d . Assertion is false but Reason is true.
e. Both Assertion and Reason are false.
14 Assertion​ :Interspecific interaction arises from the interaction of  1

populations of two different species . 


​Reason​ : Ammensalism is a type of interspecific interaction in which both 
the species are harmed . 
a . Both Assertion and Reason are true and Reason is the correct explanation
of Assertion.
b. Both Assertion and Reason are true but Reason is not the correct
explanation of Assertion.
c. Assertion is true but Reason is false.
d . Assertion is false but Reason is true.
e. Both Assertion and Reason are false.

15 Read the following and answer the four questions from 15(i) to 15(iv) given
below: 4

HIV causes AIDS and interferes with the body's ability to fight infections.

The virus can be transmitted through contact with infected blood, semen or
vaginal fluids.

Within a few weeks of HIV infection, flu-like symptoms such as fever, sore
throat and fatigue can occur. Then the disease is usually asymptomatic until it
progresses to AIDS. AIDS symptoms include weight loss, fever or night sweats,
fatigue and recurrent infections.

No cure exists for AIDS, but strict adherence to antiretroviral regimens (ARVs)
can dramatically slow the disease's progress as well as prevent secondary
infections and complications.

i HIV parasitizes ____ 


a. Y-helper cells
b. T-helper cells
c. K-helper cells
d. None of the above
ii How many stages are there in HIV infection?
a) One
b) Two
c) Three
d) Four

iii In which country AIDS was first clinically observed?


a) India
b) The United States
c) Norway
d) Pakistan

iv ​Which of the following enzyme required for viral replication plays a critical role
in the pathogenesis of HIV infection?
a) RNA polymerase
b) DNA polymerase
c) RNA polymerase II
d) Reverse transcriptase

16 Read the following and answer the four questions from 16(i) to 16(iv) given  4
below: 

Our immune system is essential for our survival. Without an immune system, 
our bodies would be open to attack from bacteria, viruses, parasites, and more. 
It is our immune system that keeps us healthy as we drift through a sea of 
pathogens.This vast network of cells and tissues is constantly on the lookout 
for invaders, and once an enemy is spotted, a complex attack is mounted. The 
immune system is spread throughout the body and involves many types of 
cells, organs, proteins, and tissues. Crucially, it can distinguish our tissue from 
foreign tissue — self from non-self. Dead and faulty cells are also recognized 
and cleared away by the immune system.If the immune system encounters a 
pathogen, for instance, a bacterium, virus, or parasite, it mounts a so-called 
immune response. 
i What is the response during the first attack ?
a. More antibodies are produced .
b. Less antibody is produced.
c. No antibodies are produced.
d. None of the above

ii What is the response during the second attack ?


a. More antibodies are produced .
b. Less antibody is produced.
c. No antibodies are produced.
d. None of the above

iii Allergic response needs a mediator and will look after that job.Who am i ?

a. IgA
b. IgM
c. IgE
d. Ig D

iv A patient is bitten by a dog with confirmed rabies infection. After treating the
bite wound, the physician injects the patient with antibodies that are specific for
the rabies virus to prevent the development of an active infection. This is an
example of:

a. Artificial active immunity


b. Natural passive immunity
c. Artificial passive immunity
d. Natural active immunity
SECTION B

17 Sex of the baby is determined by the father, not by the mother. Substantiate 2

18 Polypeptide chains of two haemoglobin molecules are shown below. One of 2


the chains shows an abnormality. Observe the diagram and answer the
following questions

a) Which of the polypeptide chains in haemoglobin is abnormal leading to a


disease?
b) What is the reason for this abnormality?

19 Read the statements below and identify the mode of interaction between the 2
species.
a.Human liver fluke feed on snail
b. Lice on humans
c. Orchid attached to a tree
d.Mycorrhizal association of fungi and roots of higher plants.

20 a) Name the technique used for the separation of DNA fragments . 2


b) How is the separated DNA visualized?
​ OR
Denaturation, Annealing and Extension are three steps of a process used for
gene amplification
a) Name the process.
b) Name the organism from which the DNA polymerase for this process is
extracted.

21 There are many features required to facilitate successful cloning into a vector. 2
Write shortly any two such features required by a vector.

22 While studying nucleotide sequence. Raj found the following sequence which 2
can be recognized by some enzymes
5′- GAATTC – 3′
3′- CTTAAG – 5′
a) Give salient features of this sequence.
b) Write the name of enzymes which recognize such sequences​.

23 ​Homeostasis is accomplished by artificial means. Explain any two ways by 2


which other living organisms cope with the situation.
24 ​The “Evil Quartet” is the nickname used to describe the causes of 2
biodiversity losses. Explain the reason leading to accelerated rates of
extinction of flora and fauna.
​ OR
Philosophically or spiritually, we need to realize that every plant or animal
species has an intrinsic value and we have a moral duty to protect them.
Write down the protective measures.

25 Explain any two levels of biodiversity. 2

SECTION C

26 The graph shown below shows the levels of LH and FSH at various stages of 3
menstrual cycle

a) Name the source of LH and FSH.


b) The level of LH is maximum during the middle day of the cycle. Mention its
effect.
c) Note the function of LH in males.

27 The first child of a couple is affected with phenylketonuria. During the second 3
pregnancy they visited a genetic counsellor and he prepared a pedigree chart
of their family
.a) What is pedigree analysis?
b) Draw the symbols for
i) Affected female.
ii) Sex unspecified.

28 Define flocs. 3
Name two groups of organisms which constitute ‘flocs’. Write their influence
on the level of BOD during biological treatment of sewage.

29 A farmer noticed reduction in tobacco yield due to root damage by 3


nematodes,
a) What is your suggestion to prevent this infestation?
b) Explain the process.
30 3

Given above is the bar diagram showing age structure of three different
populations. Observe the diagram carefully and answer the following
questions.
a) Select the stable population.
b) Compare the nature of population growth in A and C

SECTION D

31 Write down the difference between Oogenesis and spermatogenesis. 5

32 Explain Lac operon Model 5

33 a.What is sewage? 5
b.In which way can sewage be harmful to us.
c.What is the key difference between primary and secondary treatment?
Model question paper 2020-21
Biology
Time -3 hours ​ Answer key ​ Maximum scores- 70

SECTION A Marks

1 Syngamy 1

2 Parthenogenesis 1

3 Sporopollenin 1

4 Placenta 1

5 Multiple allelism 1

6. Crossing of F​1 ​hybrid with its recessive parent. 1

7 The 2-OH group present at the nucleotide in RNA is a reactive group  1


and makes RNA labile and easily degradable. 

8 cry I Ac ,cry II Ab 1

9 Agrobacterium tumifaciens

10 Mammals from colder climates generally have shorter ears and 1


limbs to minimise heat loss.

11 a .Both Assertion and Reason are true and Reason is the correct 1
explanation of Assertion.

12 c. Assertion is true but Reason is false. 1


13 d . Assertion is false but Reason is true. 1

14 c. Assertion is true but Reason is false. 1

15 4

i b.T-helper cells 

ii c.three

iii b) The United States

iv d) Reverse transcriptase

16 4

i b.Less antibody is produced.

ii a.More antibodies are produced .

iii IgE

iv d.Natural active immunity

SECTION B

17 ● ovum fertilised with a sperm carrying X- chromosome the zygote 2


develops into a female (XX) .
● The fertilisation of ovum with a Y-chromosome carrying sperm
results into a male offspring.

18 a) A 2
b) Substitution of glutamic acid by valine in the 6th position of polypeptide
chain.

19 a.predation or parasitism ½ x4=2


b. parasitism
c. commensalism
d.mutualism
20 a) Gel electrophoresis 2
b) Ethidium bromide stain
OR
a) Polymerase Chain Reaction (PCR)
b) Thermus aquaticus

21 ● Origin of replication (ori): replication starts 2


● Selectable marker: helps to identify transformants and non
transformants.
● Cloning sites (any two)

22 a) Palindromic sequence 2
b) Restriction endonuclease enzyme

23 Hibernation – winter sleeping 2


Aestivation – Summer sleeping
Migration – Moving into more suitable area
Diapause – inactive in adverse condition ( any two)

24 1)Habitat loss and fragmentation 2


2) Over-exploitation
3) Alien species invasions
4) Co-extinction (any two)
OR
In Situ conservation – Biosphere reserve, National park, WildLife
Sanctuary etc.
Ex Situ conservation – Zoo, Botanical gardens, seed bank etc.

25 1. Genetic diversity – A single species might show high diversity at the 2


genetic level over its distributional range
2. Species diversity – Diversity at species level
3. Ecological diversity – Diversity at ecosystem level

SECTION -D

26 a) Pituitary gland 3
b) Helps for ovulation
c) Stimulates the synthesis of hormones (androgens)

27 a) It is the study of inheritance of a trait for several generations of a 3


family.

b) ,

28 The groups of organisms that constitute mesh. 3


Bacteria and fungi.
They consume the organic matter thereby reduce BOD.

29 a) RNA interference (RNAi) 3


b) The best method used to prevent the attack of nematode is RNA
interference (RNAi). It involves silencing of a specific mRNA of nematode.
Here the complementary dsRNA molecule that binds to and prevents
translation of the mRNA (silencing).

30 a) B- stable population 3
b)A- expanding positive growth. Pre reproductive age people is more than
post reproductive age, B - declining negative growth.post reproductive
age is more than pre reproductive .

SECTION D

31 . 5
spermatogenesis oogenesis

Occur in testis In ovary

Gamete- sperm Gamete- ovum

Begins at puberty Begins at embryonic stage

Four gametes are produced One gamete is produced

Limited growth phase Very elaborate growth phase

32 In lac operon,lactose is the inducer.Inducer binds with repressor protein but 5


repressor cannot bind to op-erator gene,the free operator gene induces the
RNA polymerase to bind with promoter and initiates tran-scription Three
structural genes synthesise three mRNAs to produce enzymes.

33 a.The waste water with large amount of organic matters ,microbes (1) 5
b.cause communicable disease(1)
c.Primary sewage treatment - physical removal of impurities through
sedimentation and filtration(1)
Secondary sewage treatment - role of microbes, reduction in BOD(2)
Physical Education
Maximum marks: 70
Time Allowed: 3hours
General Instructions
● The question paper consists of 30 questions and all are compulsory.
● Question 1-12 carries 01 mark each and are Multiple Choice Questions.
● Questions 13-16carry 02 marks each and shall not exceed 40-60 words.
● Questions 17-26 carry 03 marks each and shall not exceed 80-100 words.
● Questions 27 - 30 carry 05 marks each and shall not exceed 150-200 words.

Section A
1. Name the type of tournament in which the defeated team gets eliminated
and does not have another chance to play.

a. Bye

b. League

C. Fixture

d. Knockout

2. Which food item has carbohydrates and fats?

a Tomato and Almond

b. Bread and butter


c Rice and Pules

d. Potato and Tomato

3. Blood pressure is the force of blood in the arteries. When this pressure
becomes abnormally high then it is called -----------

a. Heart Attack

b. Blood sugar

c Hypertension

d. Cardiac arrest

OR

According to Asthanagyog Asana lies in what place -

a. Second

b. Third

C. First

d. Fifth

4. SPD's expended form is

a. Sensory Processing Disorder

b. Sensory processing department

c Special police department

d. Special processing Disorder


5. Which development is motor development?

a. Sense Organs

b. Postural deformity

c. Disorder development

d. Bones & muscles

6. Rikli and Jones senior citizen test was developed in

a. 2000

b. 2002

C. 2001

d. 1990

7. The volume of blood flow to muscle tissues increases the most during

a. Reading

b. Eating

c Exercise

d. Walking

8. Our -------- are located on the back of our leg, below our knee.

a. calf muscles
b. Ankle muscles

Shoulder muscles

d. Knee muscles

9 The word personality is derived from the Latin word

a. Endomorphy

b. Perjona

c. Persona

d. Person

10. The ballistic method is related to

a. Strength development

b. Coordination development

C Speed development

d. Flexibility development

11. If the BMI of a person is 28. It is considered as

a. Obese

b. Normal

C. Underweight

d. Overweight
12. The damage to muscles, ligaments and tendons is called

a. Physical Injury

b. Fracture

C. Soft Tissue Injury

d. Bone Injury

13. Identify the asanas given below and write the names.

A-------------------------------

B----------------------------

C---------------------------

D--------------------------------
14. Identify the common postural deformities and give their names.

A------------------------

B--------------------------

C---------------------

D-------------------
15. Why the word 'differently-abled' is used in place of disabled nowadays?

16. What do you mean by Projectile & Power?

OR

What is Kinesiology?

17. Your grandmother is an active lady. She says she has much better
fitness as compared with you because she takes a balanced diet, does
regular exercise and often sits in the sunshine.

Based on this case, answer the following questions.

i, What is included in a balanced diet?

a. Carbohydrates

b. Vitamins

c Proteins

d. All of these

ii. Which is not a direct benefit of a balanced diet?

a. Makes us energetic

b. Proper functioning of organs

C Increases age

d. Improves metabolism

iii. Which vitamin do we get from sunshine?


a. Vitamin A

b. Vitamin B

C Vitamin C

d. Vitamin D

18. Roshni, a girl of 14, faces difficulty in breathing. She usually gets a
cough at night and sometimes early in the morning. She gets tired soon
and also feels pain in her hands, feet, and shoulders. So, she takes an
appointment with a doctor who finds out the disease at an early stage.

Based on the above case, answer the following questions.

i, What do you think about which disease Roshni is suffering from?

a. Diabetes

b. Hypertension

c Asthma

d. Obesity

ii, What could be the reason behind the disease?

a. Pollution

b. Respiratory Disorder

c Genetic Factor

d. All of these

iii. Which asana would be effective to cure the disease which Roshni had?
a. Tad asana

b. Parvatasana

c Vajrasana

d. Savasana

19. Difference between the sagittal plane and horizontal plane.

OR

What do you mean by axis? Discuss various types of axes.

20. Explain the test items of the Physical Fitness Test.

21. How are sports injuries classified?

22. Discuss the sports participation of women in India.

23. What do you mean by Flexibility?

24. Discuss the causes of ODD.

25. What are the various factors affecting projectile trajectory?

26. Write a short note on vitamins.

OR

Write briefly about protein as an essential component of the diet.

27. Explain in detail the role of physical activities in the development of


personality. Or what is the role of sports in personality development?

28. Recall the adaptive effects that take place in our body after engaging in
exercise for a longer period.

OR
What do you understand by Coordinative ability? Discuss different types of
coordinative abilities.

29. What is a league tournament? Draw a fixture of 10 teams using the


round method.

OR

What do you mean by league match or round-robin tournament?

30. Discuss in detail about the Rockport fitness walking Test.


Physical Education
Maximum marks: 70
Time Allowed: 3hours
General Instructions
● The question paper consists of 30 questions and all are compulsory.
● Question 1-12 carries 01 mark each and are Multiple Choice Questions.
● Questions 13-16carry 02 marks each and shall not exceed 40-60 words.
● Questions 17-26 carry 03 marks each and shall not exceed 80-100 words.
● Questions 27 - 30 carry 05 marks each and shall not exceed 150-200 words.

Section A
1. Name the type of tournament in which the defeated team gets eliminated
and does not have another chance to play.

a. Bye

b. League

C. Fixture

d. Knockout

ANS: Knockout

2. Which food item has carbohydrates and fats?


a Tomato and Almond

b. Bread and butter

c Rice and Pules

d. Potato and Tomato

ANS: Bread and butter

3. Blood pressure is the force of blood in the arteries. When this pressure
becomes abnormally high then it is called -----------

a. Heart Attack

b. Blood sugar

c Hypertension

d. Cardiac arrest

ANS: Hypertension

OR

According to Asthanagyog Asana lies in what place -

a. Second

b. Third

C. First

d. Fifth

ANS: Third
4. SPD's expended form is

a. Sensory Processing Disorder

b. Sensory processing department

c Special police department

d. Special processing Disorder

ANS: Sensory Processing Disorder

5. Which development is motor development?

a. Sense Organs

b. Postural deformity

c. Disorder development

d. Bones & muscles

ANS: Bones & muscles

6. Rikli and Jones senior citizen test was developed in

a. 2000

b. 2002

C. 2001

d. 1990

ANS: 2001
7. The volume of blood flow to muscle tissues increases the most during

a. Reading

b. Eating

c Exercise

d. Walking

ANS: Exercise

8. Our -------- are located on the back of our leg, below our knee.

a. calf muscles

b. Ankle muscles

Shoulder muscles

d. Knee muscles

ANS: calf muscles

9 The word personality is derived from the Latin word

a. Endomorphy

b. Perjona
c. Persona

d. Person
ANS: Persona

10. The ballistic method is related to

a. Strength development

b. Coordination development

C Speed development

d. Flexibility development

ANS: Flexibility development

11. If the BMI of a person is 28. It is considered as

a. Obese

b. Normal

C. Underweight

d. Overweight

ANS: Overweight

12. The damage to muscles, ligaments and tendons is called

a. Physical Injury
b. Fracture

C. Soft Tissue Injury

d. Bone Injury

ANS: Soft Tissue Injury

13. Identify the asanas given below and write the names.

A. SHALABHASANA

B. CHAKRASANA

C.MATSYASANA

D.BHUJANGASANA

14. Identify the common postural deformities and give their names.
A.SCOLIOSIS

B: FLAT FOOT

C: KNOCK KNEE

D: BOWLEGS
15. Why the word 'differently-abled' is used in place of disabled nowadays?

ANS: The problem arises when the term "differently-abled" is used to refer to an
individual "disabled" person. It is borderline cutesy and it diminishes the actual
experiences of disabled people. It suggests that the term disability should be
uncomfortable and therefore should be avoided.

16. What do you mean by Projectile & Power?

ANS: Projectile: A projectile is an object on which the only force acting is gravity.
Power: Power is the rate at which work is done (measured in watts (W), in other words,
the work is done per second.

OR

What is Kinesiology?

ANS: Kinesiology is a branch of physiology that studies mechanics and anatomy in


relation to human movement.

17. Your grandmother is an active lady. She says she has much better
fitness as compared with you because she takes a balanced diet, does
regular exercise and often sits in the sunshine.

Based on this case, answer the following questions.

i, What is included in a balanced diet?

a. Carbohydrates

b. Vitamins

c Proteins
d. All of these

ii. Which is not a direct benefit of a balanced diet?

a. Makes us energetic

b. Proper functioning of organs

C Increases age

d. Improves metabolism

iii. Which vitamin do we get from sunshine?

a. Vitamin A

b. Vitamin B

C Vitamin C

d. Vitamin D

ANS: All of these

Exp-Balanced diet comprises of carbohydrates, Vitamins, Proteins, Minerals, Fats,

Etc.

ii (c) Increases age

Exp- Balanced diet increases our age only indirectly by improving our immune system
and making us less prone to diseases.

iii. (d) Vitamin D

Exp- Sunlight is a free source of vitamin D.


18. Roshni, a girl of 14, faces difficulty in breathing. She usually gets a
cough at night and sometimes early in the morning. She gets tired soon
and also feels pain in her hands, feet, and shoulders. So, she takes an
appointment with a doctor who finds out the disease at an early stage.

Based on the above case, answer the following questions.

i, What do you think about which disease Roshni is suffering from?

a. Diabetes

b. Hypertension

c Asthma

d. Obesity

ii, What could be the reason behind the disease?

a. Pollution

b. Respiratory Disorder

c Genetic Factor

d. All of these

iii. Which asana would be effective to cure the disease which Roshni had?

a. Tad asana

b. Parvatasana

c Vajrasana

d. Savasana
ANS: i (c) Asthma

Exp- Asthma is a lung disease in which the airways get blocked or become a

Narrow causing difficulty in breathing.

ii (d) All of these

Exp- Asthma can be caused due to Air Pollution, respiratory infection, Genetic

Factors, etc.

iii. (b) Parvatasana

Exp- Parvatasana is very effective in curing Asthma.

19. Difference between the sagittal plane and horizontal plane.

OR

What do you mean by axis? Discuss various types of axes.


ANS: A sagittal plane is a vertical plane passing from the rear to the front, dividing the
body into left and right halves. It is also known as the anteroposterior plane. Most of the
sports and exercise movements that are two dimensional, such as running, long
jumping and somersault takes place in this plane. Transverse or Horizontal plane: The
transverse plane divides the body into top and bottom halves. In fact, it divides the body
into upper and lower sections. This plane lies horizontally that why it is also called a
horizontal plane. Movements along this plane can include an ice skating spin or rotation
to play a tennis shot.
OR
An axis is a straight line around which an object rotates. Movements at the joints of the
human musculoskeletal system are mainly rotational and take place about a line
perpendicular to the plane in which they occur. This line is known as the axis of rotation.

There are the following types of axes of rotation:

● Sagittal axis: The sagittal axis passes horizontally from posterior to anterior. It is
Formed by the intersection of the sagittal and transverse plane. The sagittal axis b.
Frontal axis: The frontal axis passes horizontally from left to right. It is formed by passes
from front to back.

● Frontal axis. Vertical axis. Frontal plane is perpendicular to the ground and
divides the body into the front or anterior and back or posterior part. Axis of the
frontal plane is the sagittal axis. One of the examples of movement on the frontal
plane could be shoulder abduction and addiction.

● Vertical axis: The vertical axis passes vertically from inferior to superior. It passes
straight through the top of the head down between feet. It is formed by the
intersection of the sagittal and frontal plane. It is also known as the longitudinal
axis. It is the longest axis.

20. Explain the test items of the Motor Fitness Test.


ANS:50 M Standing Start,
600 M Run/Walk,
Sit & Reach,
Partial Curl Up,
Push Ups (Boys),
Modified Push Ups (Girls),
Standing Broad Jump,
Agility – 4x10 M Shuttle

21. How are sports injuries classified?

ANS: Sports injuries are classified in various ways. The classification can be based on
the time taken for the tissues to become injured, the tissue type affected and seventy of
the injury.

These are detailed below

1. Soft tissue Injury

● Sprain

● Strain

● Contusion

● Abrasion

● Incision

2. Bone Injury
● Stress Fracture

● Green Stick Fracture

● Comminuted fracture

● Transverse Fracture

● Impacted Fracture

3. Joint Injury

● Dislocation of joint.

22. Discuss the sports participation of women in India.


ANS: Sport is an area where gender inequality is strongly evident. The problem Is more
sociopsychological than anything else. The first Indian women to participate in the
Olympics was in 1952. In 1975, the Government of India instituted the national sports
festival for women sports. The sports authority of India was set up in 1984 with the

Objective of broadcasting of sports and nurturing of talented children in different age


groups by providing the infrastructure, education, coaching facilities and other related
facilities. Indian female sportspersons, such as Anju Bobby George, PT Usha, Sania
Mirza, Anjali Bhagwat, Saina Nehwal have made a mark for themselves in the world of
sports and are gaining respect.

23. What do you mean by Flexibility?


ANS: It is the ability of the Joint to move to maximum range. The flexibility of an
individual varies from joint to joint due to many reasons like the structure of joint,
attachment of ligaments and tendon of joint, surrounding muscle etc. It is of two types
an Active (to do the movement without external help and Passive (to the movement with
internal help) flexibility.

24. Discuss the causes of ODD.


ANS: The various causes of Oppositional Defiant Disorder are as follows:

a. Biological or Genetic factors: Children are more susceptible of developing ODD if


they have a parent with a history of ADHD or ODD.
b. Physical factors: the presence of ODD traits has been linked to the existence of
abnormal amounts of some brain chemicals. These brain chemicals, known as
neurotransmitters, keep the brain chemicals themselves balance properly.

c Psychological factors: Children may develop ODD if they don't have good relations
with parents or have neglectful parents or have the inability to develop a social
relationships.

25. What are the various factors affecting projectile trajectory?


ANS: The angle of Projection.

● Initial Velocity.

● Gravity.

● Air resistance.

● Projection of height relevant to the landing surface.

● Spin.

26. Write a short note on vitamins.


ANS: Vitamins are complex compounds of carbon and essential for the normal
functioning of body. It is important for the metabolism of fats and carbohydrates and
helps to repair and maintenance of various tissues. Vitamins are natural substances
found in plants and animals and known as Essential nutrients for human beings. Human
body uses these substances to stay healthy and support its many functions. There are
two types of vitamins: water-soluble and fat-soluble. Deficiencies of vitamins and
minerals may be caused by disease states such as malabsorption.

OR

Write briefly about protein as an essential component of the diet.


ANS: Proteins are the basic structure of all living cells. These are complex organic
compounds. Protein is a chain of amino acids that contain carbon, oxygen, hydrogen,
and nitrogen.

These are two types of proteins:

(i) Non-essential proteins

(ii) Essential proteins


All meat and other animal products are sources of proteins. The best sources are eggs.

Milk, meat, poultry, milk products, beans etc.

Functions of Proteins:

1. The cells of muscles and ligaments are maintained with protein.

2. Proteins are needed for the growth and development of children.

3. Proteins are the main components of muscles, organs, and glands.

27. Explain in detail the role of physical activities in the development of


personality. Or what is the role of sports in personality development?

ANS: Physical activities and sports play an important role in the development of the
personality of an individual. These activities help in shaping up the personality of an
Individual such as:

1. Physical appearance: One of the primary aspects of one's personality is his

Physical appearance. Both boys and girls are very much concerned about how they
look. Physical activities are conducive to the growth and development of the

Physique. Workouts in the gym are becoming a must for all the youngsters of today. So,
this develops their outer personality which creates a good impression.

2. Social interactions: Physical activities and sports provide opportunities of interaction


between athletes coming from different regions, speaking different languages, different
caste and religion. This helps an Individual to develop a multi-dimensional personality.
Moral Values through sports is responsible for the development of sound and ideal
character, a very essential attribute of Personality.

3. Analytic thinking. This mental exercise enhance the intellectual abilities of the
participants and broaden their mental horizon. It is well said that physique is of no use if
not governed by analytic thinking, analyzing and Disciplined and assertive: In sports one
learns to make sincere efforts, which reflect positively in the development of an
individual's personality. Code of discipline is the fundamental learning of any physical
activity and effective participation in sports.
4. Disciplined and assertive: In sports, one learns to make sincere efforts, which reflect
positively in the development of an individual's personality. Code of discipline is a
fundamental learning of any physical activity and effective participation in sports. 5. Well
balanced Individual: Physical activities also provide recreation which go a long way In
producing perfectly happy, satisfied and balanced individual having a pleasing and
energetic personality, having zest for life experiences, Physical activities and sports are
the basic needs of human beings which help in the development of well-balanced
personality,

28. Recall the adaptive effects that take place in our body after engaging in
exercise for a longer period.
ANS: The adaptive effects that take place in our cardiovascular system after engaging
in exercise for a longer period are:

1. Increase in heart size we cannot do the exercise on our heart directly, but when we
perform any exercise regularly, our heart size increases. Exercising develops the
muscles of the heart.

2 Increase In heart rate generally an adult has a heart rate of 72 beats per minute while
resting, but when he exercises, his heart rate increases as per the intensity and duration
of the exercise.

3An increase in stroke volume is the quantity of blood which the heart pumps out in a
single stroke. Due to the heart's size increases, the stroke volume increases

4 The decrease in cholesterol level Regular exercise reduces the cholesterol level in our
blood, which has a direct link with blood pressure.

5. Increase in number and efficiency of capillaries Regular exercise increases the


number of capillaries and their efficiency.

OR
What do you understand by Coordinative ability? Discuss different types of
coordinative abilities.
ANS: Coordinative abilities are those abilities that enable an individual to do various
related activities accurately and efficiently. Coordinative abilities mainly depend on the
Central Nervous System.

Types:

● Orientation ability

● Coupling ability
● Reaction ability:

a simple reaction ability

b. Complex reaction Ability

● Balance Ability

29. What is a league tournament? Draw a fixture of 10 teams using the


round method.
ANS: The league tournament is one in which if it is a single league tournament then
each team plays with every other team once, if it is a double league tournament then
each team plays with every team twice. There are two methods of drawing fixture:

a) Staircase method:

Here, no. of teams (n) = 10

No. of matches = n(n-1)/2 = 45

b) Cyclic method:

Here, no. of teams (n) = 10

No. of matches = n (n-1)/2 = 45 No. of rounds = no. of teams - 1= 10-1 = 9

OR
What do you mean by league match or round-robin tournament?

League: These are also called Round robin tournament.

There are two types of league tournaments:

1. Single league tournament: In this type of tournament, every team shall play once with
every other team. The total number of matches in a single league n(n-1) 2

For example, if 10 teams are competing, the total number of matches to be played shall
be n (n-1) 10(10-1) 2 2 = 10(9) 2

2. Double league tournament: In this type of tournament, every team shall play twice
with every other team. The total number of matches shall be n (n - 1), e.g. 10 teams are
competing for the total number of matches: n (n-1) - 10 (10 – 1) = 10 (9) = 90 matches.

30. Discuss in detail about the Rockport fitness walking Test.


ANS:

Rockport Fitness Walking Test : This test is very good to measure the Cardio
Respiratory fitness of the Individual. Thus the objective of this test is to monitor the
development of the athlete's maximum Cardio-Respiratory ability (VO2).

Requirements of Test: (1) Running track (200 m or 400 m), (ii) Stopwatch, (ii) An

Assistant/Helper.

Administration of Test:

● Choose a windless day to conduct the test.

● Record your weight in pounds (lbs)

● Walk one mile (1609 mt) as fast as possible.

● Record the time to complete the one-mile walk.

● Immediately on finishing the walk record your heart rate (beats per minute).
vi Determine your Maximum Cardio-Respiratory ability (VO2) from the
calculation
Given below.

Calculation Procedure:

Analysis of the result is done by comparing It with the result of the previous test. It
is expected that appropriate training between each test should be done to show
Improvement.

The formula used to calculate

VO2 Max is : 132-853 - (0-0769 x weight) - (0-3877* Age) (6-315 Gender)-(3-2649

Time) - (0-1565 x Heart rate)

Where

a. Weight is in pounds (lbs).

b. Gender: Male = 1 and Female = 0

c Time is expressed in minutes and seconds.

d. Heart rate is in beats/minute

e Age in years
MODEL EXAMINATION
CHEMISTRY THEORY
MM : 70
CLASS: XII Time: 3 Hours
General Instructions. Read the following instructions carefully.
a) There are 33 questions in this question paper. All questions are compulsory.
b) Section A: Q. No. 1 to 2 are case-based questions having four MCQs or Reason
Assertion type based on given passage each carrying 1 mark.
c) Section A: Question 3 to 16 are MCQs and Reason Assertion type questions
carrying 1 mark each
d) Section B: Q. No. 17 to 25 are short answer questions and carry 2 marks each.
e) Section C: Q. No. 26 to 30 are short answer questions and carry 3 marks each.
f) Section D: Q. No. 31 to 33 are long answer questions carrying 5 marks each.
g) There is no overall choice. However, internal choices have been provided.
h) Use of calculators and log tables is not permitted.
SECTION A (OBJECTIVE TYPE)
1. Read the passage given below and answer the following questions: (1x4=4)
Manoj is an obedient student but some of his classmates were very mischievous. One day
in the chemistry laboratory Manoj saw some students remove the label from two bottles
containing methanol and dimethyl ether. Seeing this Manoj approach the teacher and
informed about the incident.
1. The following questions are multiple choice questions. Choose the most
appropriate answer:
i. Which chemical is used to identify methanol and dimethyl ether if the teacher asked to
perform the chemical test?
a) PCC
b) Na
c) FeCl3
d) HCl
ii. The carbonyl compound formed when methanol gets oxidised using the acidified
potassiumdichromate and can also be obtained by ozonolysis of:
a) Methane
b) Ethene
c) Pent-2-ene
d) But-2-ene
OR
Which of the following method is used for preparation of symmetrical and
unsymmetrical ethers?
a) Williamson’s synthesis
b) Riemer-Tiemann reaction
c) Kolbe’s reaction
d) None of the above

1
iii. An organic compound A reacts with CH3MgBr followed by hydrolysis gives B on
dehydration under mild conditions gives propene. Compounds A and B are
respectively.
a) Ethanol, Ethanal
b) Propanol, Propanal
c) Ethanal, Ethanol
d) Ethanal, Propan-2-ol
iv. Ethers are treated with an aqueous solutions of A in order to remove peroxides from it.
Identify the A from the following options.
a) KI
b) Br2
c) KCNS
d) Na2S2O3
2. Read the passage given below and answer the following questions:
(1x4=4)
During experimental work in chemistry lab Raman is inserting a glass tube in a rubber
cork, he got a cut in his finger and started bleeding. A student of the class noticed it and
brought a bottle of ferric chloride solution, while another student brought a bottle of
potassium chloride solution. On sensing some trouble, the chemistry teacher
immediately rushed to the students and asked the reason. On knowing the problem she
advised the student to apply ferric chloride solution on the cut and then go to the medical
room of the school. When a few drops of ferric chloride were applied on the cut, the
bleeding was arrested.
In these questions (Q. No 5-8 , a statement of assertion followed by a statement of
reason is given. Choose the correct answer out of the following choices.
a) Assertion and reason both are correct statements and reason is correct explanation for
assertion.
b) Assertion and reason both are correct statements but reason is not correct explanation
for assertion.
c) Assertion is correct statement but reason is wrong statement.
d) Assertion is wrong statement but reason is correct statement.
(i) Assertion: FeCl3 is more preferred than KCl. Reason: Greater the valency more will
be the coagulation.
(ii) Assertion: FeCl3 is better than Fe(OH)3. Reason: Oppositely charged ions causes
coagulation.
(iii) Assertion: The degree of coagulation is inversely proportional to the valency of
effective ions used. Reason: Hardy Schulze rule.
(iv) Assertion: Coagulating value depends on concentration of electrolyte. Reason:
Smaller the quantity required higher will be the coagulating power of ions.
OR
Assertion: Lyophilic sols are coagulated using electrolyte like alcohol. Reason: It is
a reversible sol.
Following questions (No. 3 -11) are multiple choice questions carrying 1 mark each:

2
3. Electrode potential of any electrode depends on
a) Nature of Metal
b) Temperature of solution
c) Molarity of solution
d) All of these.
4. Which one of the following amino acid has phenyl -OH group?
a) Lysine
b) Arginine
c) Proline
d) Tyrosine
OR
Nucleic acids are the polymers of
a) Nucleosides
b) Nucleotides
c) Bases
d) Sugars
5. Molality of an aqueous solutions of urea is 4.44mol/kg. In solution mole fraction of urea is
a) 0.074
b) 0.00133
c) 0.008
6. The magnetic moment is associated with its spin angular momentum and orbital angular
momentum. Spin only magnetic moment value of Cr3+ ion is
a) 2.87 BM
b) 3.87BM
c) 3.47 BM
d) 3.57 BM
OR
Which of the following has abnormally low value of third ionization enthalpy?
a) Lanthanum
b) Gadolinium
c) Lutetium
d) All of the above
7. Amongst the following, the strongest base in aqueous medium is…
a) CH3NH3
b) NCCH2NH2
c) (CH3)2NH
d) C6H5NHCH3
OR
-NH2 group in the aniline is activating group and proceed reaction at
a) Only para position
b) Only ortho position
c) Meta position
d) Both (a) and (b)

3
8. Which of the following complex ions is diamagnetic in nature?
a) [CoF6]3-
b) [NiCl4]2-
c) [Ni(CN)4]2-
d) [CuCl2]2-
OR
The CFSE for octahedral [CoCl6]4- is 18,000 cm-1. The CFSE for tetrahedral [CoCl4]2- will
be
a) 18,000 cm-1
b) 16,000 cm-1
c) 8,000 cm-1
d) 20,000 cm-1
9. Generally, transition elements from coloured salts due to the presence of unpaired
electrons. Which of the following compounds will be coloured in solid state?
a) Ag2SO4
b) CuF2
c) ZnF2
d) Cu2Cl2
10. Select the by-product formed in the reaction ROH+PCl5→ RCl+A Here A is
a) POCl3+HCl
b) H3PO3+ No other by-product
c) H3PO3+HCl
d) POCl3+ No other by-product
11. Calculate the edge length of a crystal where the radius of cation and anion are 95pm and
181pm respectively
a) 276pm
b) 552pm
c) 795pm
d) 368pm
In the following questions (Q. No. 12 - 16) a statement of assertion followed by a
statement of reason is given. Choose the correct answer out of the following choices.
a) Assertion and reason both are correct statements and reason is correct explanation for
assertion.
b) Assertion and reason both are correct statements but reason is not correct explanation
for assertion.
c) Assertion is correct statement but reason is wrong statement.
d) Assertion is wrong statement but reason is correct statement.
12. Assertion: All naturally occurring Amino Acid except glycine are optically active.
Reason: Most naturally amino acid have L-configuration.
13. Assertion: SO2 is more powerful reducing agent in alkaline medium.
Reason: SO2 react with H2S in presence of moisture and sulphur is precipitated.

4
14. Assertion : Perfectly ideal solution is possible with respect to binary solutions of two
liquids.
Reason : No two substances can have exactly the same nature of intermolecular forces
and also the same magnitude.
OR
Assertion: When methyl alcohol is added to water, boiling point of water increases.
Reason: When a non-volatile solute added to a volatile solvent elevation in boiling point is
observed.
15. Assertion: The α-H atom in carbonyl compounds is less acidic.
Reason: The anion formed after the loss of α-H atom is resonance stabilized.
16. Assertion: Addition reaction of water to but-1-ene in acidic medium yields butan-2-ol.
Reason: Addition of water in acidic medium proceeds through the formation of secondary
carbocation.
SECTION B
The following questions, Q.No 17 – 25 are short answer type and carry 2 marks each
17. Carry out the following conversions in not more than 2 steps
i. Aniline to bromobenzene
ii. Prop-1-ene to 1-flouropropane
OR
i. 2-bromopropane to 1-bromopropane
ii. Ethanol to Propanenitrile
18. Nalorphene(C19H21NO3) similar to morphine is used to combat withdrawal symptoms in
narcotic users.Dose of Nalorphene, generally given is 1.5mg. Calculate the mass of
1.5 x 10-3m aqueous solution required for the above dose.
19. Name the following co-oridation entities and describe there structures.
i. [Fe(CN)6]4-
ii. [Cr(NH3)4Cl2]+
OR
Describe for the following complex ions, the type of hybridization, shape and magnetic
property:
i. [Fe(H2O)6]2+
ii. [NiCl4]2-
20. For a reaction A+B→P, the rate law is given by, r=k[A] ½ [B]2

i. What is the order of this reaction?


ii. A first order reaction is found to have a rate constant k=5.5 x 10-14 s-1. Find
the half life of the reaction.

5
OR
The rate constant for a first order reaction is 60s-1. How much time will it take to reduce
the initial concentration of the reactants to its 1/16th value?
21. During nuclear explosion, one of the products is 90Sr with half-life on 28.1 years. If 1µg of
90
Sr was absorbed in the bones of newly born baby instead of calcium, how much of it will
remain after 10 years and 60 years if it is not lost metabolically?
22. Give mechanism of preparation of ethoxyethane from ethanol.
23. Draw the structures of the following molecules:
i) ClF3 ii)XeF6 iii) H2S2O7 iv)XeOF4
24. What happens when:
i. Ethyl chloride treated with aq.KOH
ii. Methyl chloride treated with KCN
25. Atoms of a compound O form ccp lattice and those of the element X occupy 1/8th of
tetrahedral voids and Y occupy ½ of all octahedral voids. What is the simplest formula of
the compound formed by the elements X, Y and O?
SECTION C
Q.No 26 -30 are Short Answer Type II carrying 3 mark each.
26. Give reasons for the following:
i. The enthalpies of atomization of transition elements are high.
ii. The E- value for the Mn3+/Mn2+ couple is much more positive than that of Cr3+/Cr2+
iii. Zn2+ salts are colourless.
OR
i. Zr and Hf have almost identical radii
ii. The E- value for copper is positive
iii. All though +3 oxidation state is the characteristic oxidation state of lanthanoids but
Ce shows +4 state also.
27. Arrange the following in increasing order of property specified:
i. Aniline, p-nitroaniline, p-toluidine (basic strength)
ii. C2H5NH2, (C2H5)2NH, C6H5NH2(solubility in water)
iii. C2H5OH, CH3CHO, CH3COOH(boiling point)
OR
i. Give a chemical test to distinguish between N-methylaniline and N,N-dimethyl
aniline.
ii. Acetophenone and benzophenone

6
iii. C2H5NH2, (C2H5)2NH, (C2H5)3N arrange in the increasing order of basic strength in
aqueous solution.
28. Niobium crystallizes in bcc lattice with density 8.55gm/cm3 calculate the radius of the
metal atom where the atomic mass of Nb is 93gmol-1.
29. i. Differentiate between globular and fibrous proteins.
ii. Where does the water present in the egg go after boiling the egg?
30. Account for the following

i. Acidic character increases from HF to HI


ii. NH3 possess high boiling point
iii. Bond enthalpy of F2 is lower than Cl2

SECTION D
Q.No 31 to 33 are long answer type carrying 5 marks each.
30. Write the structures A, B, C, D and E from the following reactions.

OR
An organic compound with molecular formula C9 H10O forms 2,4-DNP derivative, reduces
Tollen’s reagent and undergoes Cannizzaro reaction. On vigorous oxidation, it gives 1,2-
benzenedicarboxylic acid. Identify the compound.
31. Account for the following
i. Halogens are coloured
ii. Dioxygen is gas but sulphur is a solid
iii. An amorphous solid A burns in air to form a gas B which turns lime water milky.
The gas is also produced as a by-product during roasting of sulphide KMnO4
solution and reduces Fe3+ to Fe2+. Identity the solid A and the gas B and write the
reactions involved.
OR
i. Phosphorous has a greater tendency for catenation than Nitrogen.
ii. Why do noble gases have very low boiling points.
X2 is a greenish yellow gas with pungent smell and used in purification of water.
On dissolving water, it gives a solution which turns blue litmus red. When it is
passed through NaBr solution Br2, is obtained.

7
a) Identify the gas.
b) What are products obtained when X2 reacts with ammonia? Give chemical
equations.
c) What happens when X2 reacts with cold and dilute NaOH solution? Write
chemical equation and give the name of reaction.
32. The conductivity of sodium chloride at 298 K has been determined at different
concentrations and the results are given below:
Concentration/M 0.001 0.010 0.020 0.050 0.100
102 x K/Sm1 1.237 11.85 23.15 55.53 106.74
i. Calculate Λm for all concentrations and draw a graph between Λm and c1/2. Find the
value of Λm.
ii. Why is alternating current used for measuring resistance of an electrolytic
solutions.
OR
0
i. A voltaic cell is set up at 25 C with the following half cells:
Al/Al3+(0.001 M) and Ni/Ni2+ (0.50 M)
Calculate the cell voltage [E0 Ni2+/Ni= -0.25 V, E0 Al3+/Al= -1.66 V]
ii. Can absolute electrode potential of an electrode be measured?

8
MARKING SCHEME

CHEMISTRY-XII

ANSWER KEY:

1.(i) b (ii) b or a (iii) d (iv) a

2.(i) a (ii) a (iii) d (iv) a or a

3. d

4. d or b

5. a

6. b

7. c or d

8. c or c

9. b

10. a

11. b

12. c

13. b

14. c or d

15. d

16. a

17.(i)Aniline to benzenediazoniumchloride(nitrousacid) and then to


bromobenzene(Cu/HBr).

(ii) propene to 1-bromopropane(HBr) and then to flouropropane(AgF)

18. molarmass=311g/mol,1.5mol nalorphene=467mg,required=3.21g

19. (i) 6,octahedral,inner orbital (ii) 6,octahedral,inner orbital

Or
(i)Sp3d2,octahedral, paramagnetic,(ii) sp3d,tetrahedral,para

20. (i) 5/2 order (ii) 444s

or

t= 4.62 x 10-2s

21. After 10 years [R] = 0.7814μg,After 60 years [R]= 0.2278μg

22. Refer ncert text.(page no: 337)

23. t-shaped (ii) Distorted octahedral (iv) Squarepyramidal

24. (i) Ethanol (ii) Ethylcyanide

25. XY2O4

26.(i)Strong metallic bond due to greater unpaired electrons.

(ii) Due to much larger 3rd IP of Mn as Mn2+ is very stable on account of d5

(iii) Due to completely filled d-subshell hence no d-d transition.

27.(i) p-nitoaniline<aniline<p-toluidine

(ii) C6H5NH2<(C2H5)2NH<C2H5NH2 (iii) Acid<alcohol<aldehyde

Or

(i)Hintsberg test.

N-methylaniline + benzenesulphonylchloride gives precipitate and insoluble in alkali.

But N,N-dimethylaniline + benzenesulphonylchloride gives no precipitate.

(ii) Iodoform test. Acetophenone gives yellow precipitate.

(iii) C2H5NH2<(C2H5)3N<(C2H5)2NH.

28. a= 330.5pm r= 143pm.

29.(i)Fibrous: insoluble in water, chemically inactive,linear in shape,stable to moderate


temperature.

Globular: Soluble in water, chemically active, spherical shape, unstable.

(ii) The water soluble globular protein egg gets denatured and get coagulated into hard and
rubbery insoluble mass.
30.(i) Increase from HF to HI due increased bond length cleavage of H-bond more easier.

(ii) Due intermolecular H-bonding.

(iii) Small size of F2 cause repulsion so easy cleavage hence bondenthalpy lowers than Cl2.

31. A=Acetophenone B=Ethylbenzene C= Benzoic acid D= Sodium benzoate E= CHI3.

32. (i) Due to absorbtion of energy in visible light by their molecules for excitation of outer
electronsof higher energy levels.

(ii) Dioxygen small size and high electronegativity and form multiple p𝜋-p𝜋 bond.Sulphur
bigger size and lower electronegativity prefer S-S single bonds.

(iii) Sulphur, sulphurdioxide.

Or

(i)Due to larger size and lower electronegativity, P usually form P-P single bonds and hence
exist as tetrahedral P4 molecules.

(ii) Noble gases being monoatomic have no interatomic forces except weak dispersion
forces and therefore, they are liquefied at very low temperatures. Hence, they have low
boiling points.

(iii) a. Cl2

b. 8NH3 + Cl2 → 6NH4Cl + N2 ; NH3 + 3Cl2 → NCl3 + 3HCl

c. 2NaOH + Cl2 → NaCl + NaOCl + H2O

33. (i) Λm=123.70Scm2mol-1,118.50Scm2mol-1,115.75Scm2mol-1110.06Scm2mol-1,106.74


Scm2mol-1

C1/2 = 0.0,0.0316,0100,0.141,0.316

(ii) AC is used to prevent electrolysis so that concentration of ions in the solution remains
constant.

Or

(i)E0cell+ 1.41V, Ecell=1.46V

Anode: [Al(s) → Al3+(aq) + 3e] x 2

Cathode: [Ni2+(aq) + 2e → Ni(s)] x 3

(ii) No, only the difference in the potential between electrodes can be measured.This is due
to the reason that oxidation or reduction cannot occur alone.So,when we measure
electrode potential we have to take a reference electrode.
.
MODEL EXAMINATION 2020 – 21
CLASS – XII
SUBJECT – ​PHYSICS

Time Allowed : 3 HOURS MM: 70

General Instructions:

(1) All questions are compulsory. There are 33 questions in all.

(2) This question paper has five sections: Section A, Section B, Section C, Section D and
Section E.

(3) Section A contains ten very short answer questions and four assertion reasoning MCQs
of 1 mark each, Section B has two case based questions of 4 marks each, Section C
contains nine short answer questions of 2 marks each, Section D contains five short answer
questions of 3 marks each and Section E contains three long answer questions of 5 marks
each.

(4) There is no overall choice. However internal choice is provided. You have to attempt
only one of the choices in such questions.

Section – A

All questions are compulsory. In case of internal choices, attempt any one of
them.

1. Two identical balls having equal positive charge ‘q’ coulombs are suspended by two
insulating strings of equal length. What would be the effect on the force when a
plastic sheet is inserted between the two?
2. If the length of a conductor is doubled by stretching it keeping potential difference
constant, by what factor does the drift speed of the electron change?
3. Why are infra-red radiations referred to as heat waves?
OR
The frequency of oscillation of the electric vector of a certain electromagnetic wave is
5x10​14​ Hz. What is the frequency of oscillation of the corresponding magnetic field
vector ? Which part of the electromagnetic spectrum does it belong?
4. The radii of curvature of both the surfaces of a lens are equal. If one of the surfaces
is made plane by grinding, how will the focal length and power of the lens change?
5. The variation of potential difference ‘V’ with length l in case of two potentiometer
wires P and Q is shown. Which one of these will you prefer for comparing emfs of two
primary cells? Why?

6. A square loop of side 10 cm and resistance 0.5 Ω is placed vertically in the east-west
plane. A uniform magnetic field of 0.10 T is set up across the plane in the north-east
direction. The magnetic field is decreased to zero in 0.70 s at a steady rate.
Determine the magnitudes of induced emf during this time-interval.
OR
In a series LCR circuit, the voltage across an inductor, a capacitor and a resistor are
30 volt, 30 volt and 60 volt respectively. What is the phase difference between the
applied voltage and current in the circuit?
7. When light travels from a rarer to a denser medium, the speed decreases. Does the
reduction in speed imply a reduction in the energy carried by the light wave?
OR
If the path difference produced due to interference of light coming out of two slits for
yellow colour of light at a point on the screen be 3λ/2, what will be the colour of the
fringe at the point?Give reason also.
8. A proton and an α – particle are accelerated, using the same potential difference.
How are the de-Broglie wavelengths λ​p​ and λ​α​ related to each other?
9. (i)Name the type of diode whose characteristic curve is shown.
(ii)What does the point P in the given figure represent?

10. How does the angle of minimum deviation of a glass prism of refractive index 1.5
change, if it is immersed in a liquid of refractive index of 1.3?
For question numbers 11, 12, 13 and 14, two statements are given-one
labelled Assertion (A) and the other labelled Reason (R). Select the correct
answer to these questions from the codes (a), (b), (c) and (d) as given
below.
a) Both A and R are true and R is the correct explanation of A
b) Both A and R are true but R is NOT the correct explanation of A
c) A is true but R is false
d) A is false and R is also false
11. Assertion(A):When the temperature of a conductor is increased, its resistance
increases.
Reason(R): Free electrons start colliding faster.
12. Assertion(A): In Young’s experiment, the fringe width for dark fringes is same as
that for white fringes.
Reason(R): In Young’s double slit experiment performed with a source of white light,
only black and bright fringes are observed.
13. Assertion(A): When a negative (retarding) potential is given to the collector plate
with respect to the emitter plate and make it increasingly negative gradually, the
photocurrent is found to decrease rapidly and it drops to zero at stopping potential.
Reason(R): All the photoelectrons emitted from the metal do not have the same
energy.
14. Assertion(A): 1 amu =931.5MeV
Reason(R): The density of nucleus is a constant, independent of mass number, for all
nuclei.

Section – B

Questions 15 and 16 are Case Study based questions and are compulsory. Attempt
any 4 sub parts from each question. Each question carries 1 mark.

15. Semiconductors
Semiconductors are the basic building block of modern electronics, including
transistors, pn junction diodes, rectifiers, photodiodes, solar cells, LEDs, digital and
analog integrated circuits. Semiconductors have crystalline structure and contains
very few electrons at room temperature. Its resistivity lies between that of
conductors and insulators A pure semiconductor is called as intrinsic semiconductor.
The semiconductor with suitable impurity atom added to it is called extrinsic
semiconductor which is of two types- n-type and p-type semiconductors.

(i)An n-type semiconductor is

a. Positively charged
b. Negatively charged
c. Uncharged
d. Uncharged at 0 K but charged at higher temperatures

(ii)When an impurity is doped into an intrinsic semiconductor, the conductivity of the


semiconductor
a. Decreases
b. Increases
c. Remain the same
d. Becomes zero

(iii)The Fermi level in p-type semiconductor lies

a. Half way in the energy gap


b. Above the valence band in energy gap
c. Below the conduction band in the energy gap
d. Any of the above

(iv)Which of the following statements is wrong?

a. Electrons are the majority carriers in n-type semiconductors


b. Impurity atoms are called donor in p-type semiconductors
c. Number of free electrons is equal to number of holes in intrinsic semiconductors
d. The difference in the resistivity of C, Si and Ge depends upon the energy gap

(v)The depletion layer in the p-n junction arises due to

a. Drift of holes
b. Drift of electrons
c. Diffusion of charge carriers
d. Migration of impurity ions

16.Hydrogen spectrum
Electrons in an atom or a molecule absorb energy and get excited, they jump from a
lower energy level to a higher energy level and they emit radiation when they come
back to their original states. This phenomenon accounts for the emission spectrum
through hydrogen also, better known as hydrogen emission spectra.

(i)If 13.6eV of energy is needed to ionize the hydrogen atom, then energy needed to
remove electron from n=2 is ---

a. 10.2eV
b. 0eV
c. 3.4eV
d. 6.8eV
(ii)The longest wavelength in Balmer series is

a. H​α
b. H​β
c. Hγ
d. H​δ

(iii)The hydrogen spectrum which lies in the ultraviolet region is

a. Brackett
b. Paschen
c. Lyman
d. Balmer

(iv)On moving up the energy states of H-like atoms, the energy difference between
consecutive energy states

a. Increase
b. Decrease
c. Remain the same
d. May increase or decrease

(v)The Bohr atom model

a. Assumes that the angular momentum of electrons is quantized


b. Uses Einstein’s photoelectric equation
c. Predicts continuous emission spectrum for atoms
d. None of these

Section – C

All questions are compulsory. In case of internal choices, attempt anyone.

17. What is a light emitting diode (LED)? Mention two important advantages of LEDs
over conventional lamps.
18. A charge having magnitude Q is divided into two parts q and (Q-q). If the two parts
exert a maximum force of repulsion on each other, then find the ratio of q/Q.
OR
Derive the expression for the electric potential at a point distant r from a point
charge q.
19. A cell of emf ε and internal resistance r is connected across a variable resistance R.
Plot a graph showing the variation of terminal voltage V of the cell verses the current
I. Using the graph, show how the emf of the cell and its internal resistance be
determined.
20. A charge ‘q’ moving along the X-axis with a velocity v→ is subjected to a uniform

magnetic field B acting along the Z-axis as it crosses the origin O. (i) Trace its
trajectory. (ii) Does the charge gain kinetic energy as it enters the magnetic field?
Justify your answer.
21. A series LCR circuit with L = 4.0 H, C = 100 mF and R = 60 Ω is connected to a
variable frequency 240 V source. Calculate: (i) the angular frequency of the source
which derives the circuit at resonance; (ii) the current at the resonating frequency.
OR
A circular copper disc 10 cm in radius rotates at a speed of 2π rad/s about an axis
through its centre and perpendicular to the disc. A uniform magnetic field of 0.2T
acts perpendicular to the disc. (i) Calculate the potential difference developed
between the axis of the disc and the rim. (ii) What is the induced current if the
resistance of the disc is 2Ω ?
22. A wire AB is carrying a current of 12Aand is lying on the table. Another wire CD,
carrying a current of 5A, is arranged just above AB at a height of 1mm. What should
be the weight per unit length of this wire so that CD remains suspended at its
position? Indicate the direction of current in CD and the nature of force between the
two wires.
23. (i)The graph shows the variation of stopping potential with frequency of incident
radiation for two photosensitive metals A and B. Which one of the two has higher
work function? Justify your answer.

(ii)Give any two properties of photon.


24. (i)State Lenz’s rule.
(ii) A rectangular loop and a circular loop are moving out of a uniform magnetic field
region to a field-free region with a constant velocity v. In which loop do you expect
the induced emf to be constant during the passage out of the field region? The field
is normal to the loops.
OR
(i)Define mutual inductance.
(ii) How is the mutual inductance of a pair of coils affected when : (a) Separation
between the coils is increased. (b) The number of turns of each coil is increased?
25. Which constituent radiation of the electromagnetic spectrum is used? (i) To
photograph internal parts of human body. (ii) For air aircraft navigation

Section –D

All questions are compulsory. In case of internal choices, attempt any one.
26. A storage battery is of emf 8V and internal resistance 0.5 ohm is being charged by
d.c supply of 120 V using a resistor of 15.5 ohm
a) Draw the circuit diagram.
b) Calculate the potential difference across the battery.
c) What is the purpose of having series resistance in this circuit?
27. For a single slit of width a, the first minimum of the diffraction pattern of a
monochromatic light of wavelength λ occurs at an angle of λ/a. At the same angle of
λ/a, we get a maximum for two narrow slits separated by a distance a. Explain.
OR
Three light rays red (R), green (G) and blue (B) are incident on a right angled prism
‘abc’ at face ‘ab’. The refractive indices of the material of the prism for red, green
and blue wavelengths are 1.39, 1.44 and 1.47 respectively. Out of the three which
colour ray will emerge out of face ‘ac’? Justify your answer. Trace the path of these
rays after passing through face ‘ab’.

28. (i) If both the number of protons and the number of neutrons are conserved in each
nuclear reaction, in what way is mass converted into energy (or vice-versa) in a
nuclear reaction?
(ii) Two nuclei have mass numbers in the ratio 1 : 8. What is the ratio of their
nuclear radii?
(iii) Draw a graph showing variation of potential energy of a pair of nucleon as a
function of their separation indicate the region in which the nuclear force is (a)
Attractive (b) Repulsive?
29. Define angle of dip and angle of declination. Deduce the relation connecting angle of
dip and horizontal component of earth’s total magnetic field with the horizontal
direction
30. Draw a schematic diagram of a reflecting telescope (Cassegrain). Write two
important advantages that the reflecting telescope has over a refracting type.

Section – E

All questions are compulsory. In case of internal choices, attempt any one.
31. A thin convex lens having two surfaces of radii of curvature R​1​ and R​2​ is made of a
material of refractive index n​2​. It is kept in a medium of refractive index n​1​. Derive,
with the help of a ray diagram, the lens maker formula when a point object placed
on the principal axis in front of the radius of curvature R​1​ produces an image I on the
other side of the lens.
OR
Describe Young’s double slit experiment to produce interference pattern due to a
monochromatic source of light. Deduce the expression for the fringe width. What
happens to fringe width if the experiment is conducted in water?
32. (a) Define electric flux. Write its S.I. units.
(b) Using Gauss’s law, prove that the electric field at a point due to a uniformly
charged infinite plane sheet is independent of the distance from it.
OR
(i)Define dipolemoment. Give its unit
(ii)Two identical parallel plate (air) capacitors C​1​ and C​2​ have capacitances C each.
The between their plates is now filled with dielectrics as shown. If the two capacitors
still have equal capacitance, obtain the relation between dielectric constants K, K​1
and K​2 .​

33. State Faraday’s law of electromagnetic induction.


Figure shows a rectangular conductor PQRS in which the conductor PQ is free to
move in a uniform magnetic field B perpendicular to the plane of the paper. The field
extends from x = 0 to x = b and is zero for x > b. Assume that only the arm PQ
possesses resistance r. When the arm PQ is pulled outward from x = 0 to x = 2b and
is then moved backward to x = 0 with constant speed v, obtain the expressions for
the flux and the induced emf. Sketch the variations of these quantities with distance
0 ≤ x ≤ 2b.

OR
Draw a schematic diagram of a step-up transformer. Explain its working principle.
Deduce the expression for the secondary to primary voltage in terms of the number
of turns in the two coils. In an ideal transformer, how is this ratio related to the
currents in the two coils? How is the transformer used in large scale transmission
and distribution of electrical energy over long distances?
CBSE MODEL EXAMINATION PAPER PHYSICS(2020-’21)
Marking scheme and answer key
Q.No Value point Marks
1 Force will decrease. 1
F= F​0​/K
2 Drift velocity will be halved 1
V​d​ =(eE/m)τ= =(eV/lm)τ
3 Infrared waves are produced by hot bodies and molecules, so are referred to as heat 1
waves.
OR
5x10​14 ​Hz, visible region.
4 Focal length gets doubled. 1/2
Power is halved. 1/2
5 Potentiometer ‘Q’ will be preferred 1/2
potential gradient is less, sensitivity is more 1/2
10−3
6 ε = ∆Φ ∆t = √2×0.7 = 1.0 mV
1
OR
tan Φ = ( V​L​ - V​C​ ) / V​R​ , Φ = 0
7 No. Energy carried by a wave depends on the amplitude of the wave, not on the speed 1
of wave propagation
OR
The point will be a dark fringe. As it satisfies the condition for minima or destructive
interference
8 λ= h 1
√2mqV
λp √mα q α

λα = √mp q p = √(4mp * 2e)/(mp * e) = 8

9 (i)solar cell 1/2


(ii)Point P represents some positive voltage on solar cell with zero current through solar
cell. 1/2
10 µ​ge​ = 1.5 /1.3 = 1.15 1/2
as µ decreases so δm decrease 1/2
11 (a) 1
12 (c) 1
13 (a) 1
14 (b) 1
15 (i)c 4
(ii)b
(iii)b
(iv)b
(v)c
16 (i)c
(ii)a
(iii)c
(iv)b
(v)a
17 A light emitting diode is simply a forward biased p-n junction which emits spontaneous 1+1
light radiation. When forward bias is applied, the electron and holes at the junction
recombine and energy released is emitted in the form of light.
The advantages of LEDs are: (i) Low operational voltage and less power. (ii) Fast action
with no warm up time. (iii) Emitted light is nearly monochromatic radiation. (iv) They
have long life.
18
1/2
1/2

q/Q=1/2 1

OR

Derivation(NCERT page 54)


2
19 Since V =ε-Ir
1

1/2
When I=0, emfε= V
Maximum current is drawn from the cell when terminal voltage is 0.
So , 0=ε-I​max​r
1/2
Internal resistance, r= ε/I​max
20 (i) 1
(i)
(ii)Magnetic Lorentz Force is perpendicular to velocity , so work done by the magnetic 1
force on charge is zero; so charge does not gain kinetic energy on entering the magnetic
field.
21 1+1

OR

22 1

Direction of current in CD is opposite to that of in AB and Nature of force is repulsive. 1


23 (i)Metal A has higher work function as it has higher threshold frequency 1+1
(ii) In interaction of radiation with matter, radiation behaves as if it is made up of
particles called photons. (ii) Each photon has energy E (=hν) and momentum p (= h
ν/c), and speed c, the speed of light.
24 (i) The polarity of induced emf is such that it tends to produce a current which opposes 1
the change in magnetic flux that produced it.
(ii) The induced emf is expected to be constant only in the case of the rectangular loop. 1
In the case of circular loop, the rate of change of area of the loop during its passage out
of the field region is not constant, hence induced emf will vary accordingly
OR
(i)Mutual inductance is numerically equal to the emf induced in secondary coil if the
rate of change of current in the primary coil is 1A/s. 1
(ii)(a)decrease
(b)increase 1/2
1/2
25 (i) X -Rays 1
(ii) (ii) Microwaves 1

26 a)Circuit diagram 1
b) Applying correct formula 1/2
And caluation of p.d=11.5V 1/2
c) Series resistor limits the current drawn from source 1
27 1/2

1/2

1
OR
1/2

1.5

28 (i) The difference in these binding energies appears as energy released or absorbed in a 1
nuclear reaction
1

(ii)
(iii)page 445(NCERT) 1
29 Definition 1+1
2

30 Diagram 2
Advantages : (i) It is free from chromatic aberration. (ii) Its resolving power is greater 1
than refracting telescope due to larger aperture of mirror.
31 Diagram 2
Refraction at both surfaces 1
Derivation 2
OR
Diagram 1
Positions of maxima and minima 1.5
Bandwidth derivation 1.5
In water, bandwidth reduces to 1/n times 1
32 (i)Definition and unit 1.5
(ii)Statement of Gauss’s law 1
Derivation 2.5
OR
(i)definition ,unit 1.5

3.5
(ii)
33 (i)Statement 1
Expressions for flux and induced emf 2
Graph 2
OR
Diagram 1
Principle 1
Derivation 2
Use of transformer 1
CBSE Class 12 PHYSICS (2020-’21)

BLUE PRINT
Unit VSA CS SA1 SA2 LA Total

Electrostatics 1(1) 2(1) 5(1)


16
Current Electricity 1(3)RA 2(1) 3(1)

Magnetic Effects of Current and 2(2) 3(1)


Magnetism

17
Electromagnetic Induction and 1(1) 2(2) 5(1)
Alternating Current

Electromagnetic Waves 1(1) 2(1)


18
Optics 1(4)RA 3(2) 5(1)

Dual Nature of Radiation and Matter 1(2)RA 2(1)


12
Atoms and Nuclei 1(1)RA 4(1) 3(1)

Semiconductor electronics 1(1) 4(1) 2(1)


7
Total 10(10) 8(2) 18(9) 15(5) 15(3) 70(33)
4(4)
Special Thanks to

Ms. Anoja Department of English


Ms. Lijo Elizabeth Abraham Department of Physics
Mr. Vysakh G Department of Chemistry
Ms. Archana Department of Biology
Ms. Remya Sarah George Department of Informatics Practices
Mr. Nithin S Department of Mathematics
Ms. Deepthi V Department of Home Science
Ms. Lekshmi B M Department of Commerce
Mr. Kuthubudheen Department of Economics
Mr. Arun Viswanadh Department of Physical Education.

Co-ordination and compilation


Ms. Annie Francis
Vice Principal, The Oxford School Trivandrum

With Best Compliments From :

Shibu S Dr. Abdul Salam Biju S Pillai Dr. Jayanthi


President Gen. Secretary Treasurer Patron

You might also like